Закрыть

Закон ома и его практическое применение: Закон Ома и его применение

Содержание

Закон Ома и его применение

Закон Ома выражает зависимость между напряжением U, током I и сопротивлением R для участка цепи, не содержащего ЭДС:

U = I ∙ R 

где     U – напряжение, в вольтах;

I – сила тока, в амперах;

R – сопротивление, в омах.

  

Три составляющие закона Ома

 

Для участка цепи, содержащего ЭДС, закон Ома выражает зависимость между ЭДС источника тока E, сопротивлением нагрузки Rн, током I и внутренним сопротивлением r0 источника тока:

I = E / (Rн + r0)

 

Напряжение на зажимах источника тока U определяется по формуле:

U = E

Ir0 = IRн

 

Диаграмма, помогающая запомнить закон Ома. Нужно закрыть искомую величину, и два других символа дадут формулу для её вычисления:

 

В зависимости от сопротивления нагрузки Rн существуют три режима работы:

 

режим короткого замыкания при Rн = 0

Iк.з. = Imax = E / r0

 

режим холостого хода при Rн = ∞

Uх.х. = Umax = E

 

режим согласованной нагрузки при Rн = r0

I = Iк.з. / 2

U = Uх.х. / 2

Р = Uх.х. ∙ Iк.з. / 4

 

         В последнем случае источник тока отдает в нагрузку максимально возможную мощность. Если сопротивление нагрузки состоит из нескольких резисторов, то справедливы следующие соотношении:

 

при последовательном соединении резисторов R1 и R2:

U1 / U2 = R1 / R2

U / U2 = R1 + R2 / R2

U2 = U ∙ R2 / (R1 + R2)

U1 = U

R1 / (R1 + R2) 

где     U – подведенное напряжение;

U1 и U2 – падение напряжения на резисторах R1 и R2;

 

при параллельном соединении резисторов R1 и R2:

I1 / I2 = R2 / R1

 

         Подключение резисторов параллельно или последовательно измерительному прибору позволяет расширить пределы измерений. Можно показать, что расширение пределов измерения вольтметра достигается включением последовательно с ним добавочного резистора Rдоб. Если верхний предел измерения вольтметра

Uв, а необходимый предел измерения Uн > Uв, то включение Rдоб = Rп ∙ (Uн / Uв – 1) позволяет отсчитывать максимально напряжение Uн. В приведенном выражении Rп – сопротивление прибора, равное Rп = Uв / Iв, где Iв – ток прибора при подведении к нему напряжения Uв.

         Расширение предела измерения амперметра достигается параллельным подключением к нему дополнительного резистора (шунта). Если верхний предел измерения тока амперметра Iв, а необходимый предел измерения Iн > Iв, то сопротивление шунта:

Rш = R

п / (Iн / Iв) – 1

 

         Сопротивление вольтметра можно определить следующим способом. Измерить вольтметром напряжение на зажимах источника напряжения E и, отметив показания вольтметра, включить последовательно с ним такой добавочный резистор, при котором показание вольтметра уменьшится вдвое, т.е. при равенстве сопротивлений вольтметра и добавочного резистора.

         На этом же принципе основана и обратная задача определения величины неизвестного сопротивления с помощью вольтметра.

Закон ома и его практическое применение

Закон Ома, основанный на опытах, представляет собой в электротехнике основной закон, который устанавливает связь силы электрического тока с сопротивлением и напряжением.

Появление смартфонов, гаджетов, бытовых приборов и прочей электротехники коренным образом изменило облик современного человека. Приложены огромные усилия, направленные на исследование физических закономерностей для улучшения старой и создания новой техники. Одной из таких зависимостей является закон Ома.

Закон Ома – полученный экспериментальным путём (эмпирический) закон, который устанавливает связь силы тока в проводнике с напряжением на концах проводника и его сопротивлением, был открыт в 1826 году немецким физиком-экспериментатором Георгом Омом.

Строгая формулировка закона Ома может быть записана так: сила тока в проводнике прямо пропорциональна напряжению на его концах (разности потенциалов) и обратно пропорциональна сопротивлению этого проводника.

Формула закона Ома записывается в следующем виде:

U – электрическое напряжение (разность потенциалов), единица измерения напряжения- вольт [В];

Согласно закону Ома, увеличение напряжения, например, в два раза при фиксированном сопротивлении проводника, приведёт к увеличению силы тока также в два раза

И напротив, уменьшение тока в два раза при фиксированном напряжении будет означать, что сопротивление увеличилось в два раза.

Рассмотрим простейший случай применения закона Ома. Пусть дан некоторый проводник сопротивлением 3 Ом под напряжением 12 В. Тогда, по определению закона Ома, по данному проводнику течет ток равный:

Существует мнемоническое правило для запоминания этого закона, которое можно назвать треугольник Ома. Изобразим все три характеристики (напряжение, сила тока и сопротивление) в виде треугольника. В вершине которого находится напряжение, в нижней левой части – сила тока, а в правой – сопротивление.

Правило работы такое: закрываем пальцем величину в треугольнике, которую нужно найти, тогда две оставшиеся дадут верную формулу для поиска закрытой.

Где и когда можно применять закон Ома?

Закон Ома в упомянутой форме справедлив в достаточно широких пределах для металлов. Он выполняется до тех пор, пока металл не начнет плавиться. Менее широкий диапазон применения у растворов (расплавов) электролитов и в сильно ионизированных газах (плазме).

Работая с электрическими схемами, иногда требуется определять падение напряжения на определенном элементе. Если это будет резистор с известной величиной сопротивления (она проставляется на корпусе), а также известен проходящий через него ток, узнать напряжение можно с помощью формулы Ома, не подключая вольтметр.

Значение Закона Ома

Закон Ома определяет силу тока в электрической цепи при заданном напряжении и известном сопротивлении.

Он позволяет рассчитать тепловые, химические и магнитные действия тока, так как они зависят от силы тока.

Закон Ома является чрезвычайно полезным в технике(электронной/электрической), поскольку он касается трех основных электрических величин: тока, напряжения и сопротивления. Он показывает, как эти три величины являются взаимозависимыми на макроскопическом уровне.

Если бы было можно охарактеризовать закон Ома простыми словами, то наглядно это выглядело бы так:

Из закона Ома вытекает, что замыкать обычную осветительную сеть проводником малого сопротивления опасно. Сила тока окажется настолько большой, что это может иметь тяжелые последствия.

Принцип работы одного из основополагающих законов электротехники хочется начать объяснять с аллегории — показа небольшого карикатурного изображения 1 из трех человечков под именами «Напряжение U», «Сопротивление R» и «Ток I».

На нем видно, что «Ток» пытается пролезть через сужение в трубе, которое «Сопротивление» усердно затягивает. В то же время «Напряжение» прилагает максимально возможное усилие для прохождения, проталкивания «Тока».

Этот рисунок напоминает, что электрический ток — это упорядоченное движение заряженных частиц в определенной среде. Передвижение их возможно под действием приложенной внешней энергии, создающей разность потенциалов — напряжение. Однако, внутренние силы проводников и элементов схемы уменьшают величину тока, оказывают сопротивление его перемещению.

Рассмотрим простую схему 2, поясняющую действие закона Ома для участка электрической цепи постоянного тока.

В качестве источника напряжения U используем аккумуляторную батарею, которую подключим к сопротивлению R толстыми и одновременно короткими проводами в точках А и В. Допустим, что провода не влияют на величину прохождения тока I к резистору R.

Формула (1) выражает соотношения между сопротивлением (омы), напряжением (вольты) и током (амперы).

Ее называют законом Ома для участка цепи. Кружок под формулой облегчает ее запоминание и пользование для выражения каждого из составляющих параметров U, R или I (U расположено сверху над черточкой, а R и I — снизу).

Если надо определить один из них, то мысленно закрываем его и работаем с двумя оставшимися, выполняя арифметические действия. Когда величины расположены на одной строчке, то их перемножаем. А в случае расположения их на разных уровнях выполняем деление верхнего на нижний.

Эти соотношения показаны на формулах 2 и 3 рисунка 3 ниже.

В этой схеме для измерения тока используется амперметр, который соединен последовательно с нагрузкой R, а напряжения — вольтметр, подключенный параллельно точкам 1 и 2 резистора. Учитывая конструктивные особенности приборов, допустим, что амперметр не влияет на величину тока в схеме, а вольтметр — напряжения.

Определение сопротивления с помощью закона Ома

Пользуясь показаниями приборов (U=12 В, I=2,5 А) можно по формуле 1 определить величину сопротивления R=12/2,5=4,8 Ом.

На практике этот принцип заложен в работу измерительных приборов — омметров, определяющих активное сопротивление различных электрических устройств. Поскольку они могут быть настроены на замеры различных диапазонов величин, то их соответственно подразделяют на микроомметры и миллиомметры, работающие с малыми сопротивлениями и тера-, гиго- и мегаомметры — измеряющие очень большие значения.

Для конкретных условий эксплуатации их выпускают:

Принцип работы омметра

Для выполнения замеров обычно используются магнитоэлектрические приборы, хотя в последнее время широко внедряются электронные (как аналоговые, так и цифровые).

В омметре магнитоэлектрической системы используется токоограничивающий резистор R, пропускающий через себя только миллиамперы и чувствительная измерительная головка (миллиамперметр). Она реагирует на протекание малых токов через прибор за счет взаимодействия двух электромагнитных полей от постоянного магнита N-S и поля, создаваемого током, проходящим через обмотку катушки 1 с токопроводящей пружинкой 2.

В результате взаимодействия сил магнитных полей происходит отклонение стрелки прибора на определенный угол. Шкала головки для облегчения работы сразу проградуирована в омах. При этом используется выражение сопротивления через ток по формуле 3.

У омметра для обеспечения точных замеров должно поддерживаться стабилизированное значение подаваемого напряжения от батареи питания. С этой целью применяется калибровка посредством использования добавочного регулировочного резистора R рег. С его помощью до начала измерения на схему ограничивается подача излишнего напряжения от источника, выставляется строго стабильная, нормируемая величина.

Определение напряжения с помощью закона Ома

Во время работ с электрическими схемами бывают случаи, когда необходимо узнать падение напряжения на каком-то элементе, например, резисторе, а известно его сопротивление, которое обычно маркируется на корпусе, и проходящий сквозь него ток. Для этого не обязательно подключать вольтметр, а достаточно воспользоваться расчетами по формуле 2.

В нашем случае для рисунка 3 проведем расчеты: U=2,5·4,8 =12 В.

Определение тока с помощью закона Ома

Этот случай описывает формула 3. Его используют для расчета нагрузок в электрических схемах, выбора сечений проводников, кабелей, предохранителей или защитных автоматов.

В нашем примере расчет выглядит так: I=12/4,8=2,5 А.

Этот способ в электротехнике используют для исключения работы определенных элементов из схемы без их демонтажа. Для этого на ненужном резисторе замыкают накоротко проводником входящую и отходящую клеммы (на рисунке 1 и 2) — шунтируют.

В результате ток схемы выбирает для себя путь с меньшим сопротивлением через шунт и резко возрастает, а напряжение зашунтированного элемента падает до нуля.

Этот режим является частным случаем шунтирования и, в общем-то, показан на рисунке выше, когда закоротка устанавливается на выходные клеммы источника. При его возникновении создаются очень опасные большие токи, способные поражать людей и сжигать не защищенное электрооборудование.

Для борьбы со случайно возникающими замыканиями в электрической сети используют защиты. На них выставляют такие уставки, которые не мешают работать схеме в нормальном режиме. Они отключают питание только при аварийных случаях.

Например, если ребенок по неосторожности всунет в домашнюю розетку проволоку, то правильно настроенный автоматический выключатель вводного квартирного щита практически моментально отключит электроснабжение.

Все, что описано выше, относится к закону Ома для участка цепи постоянного тока, а не полной схемы, где процессов может быть значительно больше. Следует представлять, что это только небольшая часть применения его в электротехнике.

Закономерности, выявленные знаменитым ученым Георгом Симоном Омом между током, напряжением и сопротивлением по-разному описываются в различных средах и цепях переменного тока: однофазных и трехфазных.

Вот основные формулы, выражающие соотношения электрических параметров в металлических проводниках.

Более сложные формулы для проведения специальных расчетов закона Ома на практике.

Как видим, исследования, которые провел гениальный ученый Георг Симон Ом, имеют огромное значение даже в наше время бурного развития электротехники и автоматики.

В природе существует два основных вида материалов, проводящие ток и непроводящие (диэлектрики). Отличаются эти материалы наличием условий для перемещения в них электрического тока (электронов).

Из токопроводящих материалов (медь, алюминий, графит, и многие другие), делают электрические проводники, в них электроны не связаны и могут свободно перемещаться.

В диэлектриках электроны привязаны к атомам намертво, поэтому ток в них течь не может. Из них делают изоляцию для проводов, детали электроприборов.

Для того чтобы электроны начали перемещаться в проводнике (по участку цепи пошел ток), им нужно создать условия. Для этого в начале участка цепи должен быть избыток электронов, а в конце – недостаток. Для создания таких условий используют источники напряжения – аккумуляторы, батарейки, электростанции.

Формула Закона Ома

В 1827 году Георг Симон Ом открыл закон силы электрического тока. Его именем назвали Закон и единицу измерения величины сопротивления. Смысл закона в следующем.

Чем толще труба и больше давление воды в водопроводе (с увеличением диаметра трубы уменьшается сопротивление воде) – тем больше потечет воды. Если представить, что вода это электроны (электрический ток), то, чем толще провод и больше напряжение (с увеличением сечения провода уменьшается сопротивление току) – тем больший ток будет протекать по участку цепи.

Сила тока, протекающая по электрической цепи, прямо пропорциональна приложенному напряжению и обратно пропорциональна величине сопротивления цепи.

где I – сила тока, измеряется в амперах и обозначается буквой А; U – напряжение, измеряется в вольтах и обозначается буквой В; R – сопротивление, измеряется в омах и обозначается .

Если известны напряжение питания U и сопротивление электроприбора R, то с помощью вышеприведенной формулы, воспользовавшись онлайн калькулятором, легко определить силу протекающего по цепи тока I.

Онлайн калькулятор для определения силы тока
Напряжение, В:
Сопротивление, Ом:

С помощью закона Ома рассчитываются электрические параметры электропроводки, нагревательных элементов, всех радиоэлементов современной электронной аппаратуры, будь то компьютер, телевизор или сотовый телефон.

Применение закона Ома на практике

На практике часто приходится определять не силу тока I, а величину сопротивления R. Преобразовав формулу Закона Ома, можно рассчитать величину сопротивления R, зная протекающий ток I и величину напряжения U.

Онлайн калькулятор для определения величины сопротивления
Напряжение, В:
Величина тока, А:

Величину сопротивления может понадобится рассчитать, например, при изготовлении блока нагрузок для проверки блока питания компьютера. На корпусе блока питания компьютера обычно есть табличка, в которой приведен максимальный ток нагрузки по каждому напряжению. Достаточно в поля калькулятора ввести данные величины напряжения и максимальный ток нагрузки и в результате вычисления получим величину сопротивления нагрузки для данного напряжения. Например, для напряжения +5 В при максимальной величине тока 20 А, сопротивление нагрузки составит 0,25 Ом.

Формула Закона Джоуля-Ленца

Величину резистора для изготовления блока нагрузки для блока питания компьютера мы рассчитали, но нужно еще определить какой резистор должен быть мощности? Тут поможет другой закон физики, который, независимо друг от друга открыли одновременно два ученых физика. В 1841 году Джеймс Джоуль, а в 1842 году Эмиль Ленц. Этот закон и назвали в их честь – Закон Джоуля-Ленца.

Потребляемая нагрузкой мощность прямо пропорциональна приложенной величине напряжения и протекающей силе тока. Другими словами, при изменении величины напряжения и тока будет пропорционально будет изменяться и потребляемая мощность.

где P – мощность, измеряется в ваттах и обозначается Вт; U – напряжение, измеряется в вольтах и обозначается буквой В; I – сила ток, измеряется в амперах и обозначается буквой А.

Зная напряжения питания и силу тока, потребляемую электроприбором, можно по формуле определить, какую он потребляет мощность. Достаточно ввести данные в окошки ниже приведенного онлайн калькулятора.

Онлайн калькулятор для определения потребляемой мощности
Напряжение, В:
Сила тока, А:

Закон Джоуля-Ленца позволяет также узнать силу тока, потребляемую электроприбором зная его мощность и напряжение питания. Величина потребляемого тока необходима, например, для выбора сечения провода при прокладке электропроводки или для расчета номинала.

Онлайн калькулятор для определения силы тока в зависимости от потребляемой мощности
Потребляемая мощность, Вт:
Напряжение питания, В:

Например, рассчитаем потребляемый ток стиральной машины. По паспорту потребляемая мощность составляет 2200 Вт, напряжение в бытовой электросети составляет 220 В. Подставляем данные в окошки калькулятора, получаем, что стиральная машина потребляет ток величиной 10 А.

Еще один пример, Вы решили в автомобиле установить дополнительную фару или усилитель звука. Зная потребляемую мощность устанавливаемого электроприбора легко рассчитать потребляемый ток и правильно подобрать сечение провода для подключения к электропроводке автомобиля. Допустим, дополнительная фара потребляет мощность 100 Вт (мощность установленной в фару лампочки), бортовое напряжение сети автомобиля 12 В. Подставляем значения мощности и напряжения в окошки калькулятора, получаем, что величина потребляемого тока составит 8,33 А.

Разобравшись всего в двух простейших формулах, Вы легко сможете рассчитать текущие по проводам токи, потребляемую мощность любых электроприборов – практически начнете разбираться в основах электротехники.

Преобразованные формулы Закона Ома и Джоуля-Ленца

Встретил в Интернете картинку в виде круглой таблички, в которой удачно размещены формулы Закона Ома и Джоуля-Ленца и варианты математического преобразования формул. Табличка представляет собой не связанные между собой четыре сектора и очень удобна для практического применения

По таблице легко выбрать формулу для расчета требуемого параметра электрической цепи по двум другим известным. Например, нужно определить ток потребления изделием по известной мощности и напряжению питающей сети. По таблице в секторе тока видим, что для расчета подойдет формула I=P/U.

А если понадобится определить напряжение питающей сети U по величине потребляемой мощности P и величине тока I, то можно воспользоваться формулой левого нижнего сектора, подойдет формула U=P/I.

Подставляемые в формулы величины должны быть выражены в амперах, вольтах, ваттах или Омах.

Как использовать закон Ома: инструкция для чайников с примерами | ASUTPP

Я обычно не использую много математики, когда занимаюсь электроникой. Но закон Ома — редкое исключение:)

Закон был выведен Георгом Омом и основан на взаимосвязи напряжения, тока и сопротивления:

Рис. 1. Иллюстрация связи сопротивления (Ohm), тока (Amp) и напряжения (Volt)

Рис. 1. Иллюстрация связи сопротивления (Ohm), тока (Amp) и напряжения (Volt)

Посмотрите на рисунок выше и посмотрите, имеет ли для вас смысл:

  • Если вы увеличиваете напряжение в цепи, при неизменном сопротивлении, вы получаете больше тока.
  • Если вы увеличиваете сопротивление в цепи, в то время как напряжение остается тем же, вы получаете меньший ток.

Закон Ома — это способ описания взаимосвязи между напряжением, сопротивлением и током с использованием математики: V = R*I, где

  • V — напряжение;
  • I — ток;
  • R — является символом сопротивления.

Вы видоизменить формулу и получить R = V / I или I = V / R. Пока у вас есть две переменные, вы можете вычислить последнюю.

Закон треугольника Ома

Вы можете использовать этот треугольник, чтобы запомнить закон Ома:

Рис. 2. Треугольник закона Ома

Рис. 2. Треугольник закона Ома

Как использовать это:

Используйте свою руку, чтобы покрыть переменную, которое вы хотите найти. Если оставшиеся буквы расположены друг над другом, это значит, разделить верхнюю с нижней. Если они рядом друг с другом, это значит, умножить одно на другое.

Пример: нужно найти напряжение

Рис. 3. Закрываем рукой напряжение

Рис. 3. Закрываем рукой напряжение

Закрываем V в треугольнике, затем смотрим на R и I. I и R рядом друг с другом (на одной горизонтальной линии), поэтому вам нужно их умножить. Это означает, что вы получите:

V = I * R

Все просто!

Пример: Нужно найти сопротивление

Рис. 4. Закрываем сопротивление

Рис. 4. Закрываем сопротивление

Положите руку на R. Затем вы увидите, что V над I. Это означает, что вы должны разделить V на I:

R = V / I

Пример: нужно найти силу тока

Рис. 5. Закрываем ток

Рис. 5. Закрываем ток

Поместите руку над I. Затем вы увидите V над R, что означает деление V на R:

I = V / R

Практические примеры использования закона Ома

Пример 1

Лучший способ научить его использовать на своем примере.

Ниже приведена очень простая схема с аккумулятором и резистором. Аккумулятор представляет собой источник напряжения на 12 вольт, а сопротивление резистора составляет 600 Ом. Сколько тока протекает по цепи?

Рис. 6. Пример 1

Рис. 6. Пример 1

Чтобы найти величину тока, вы можете использовать треугольник выше к формуле для тока: I = V / R. Теперь вы можете рассчитать ток, используя напряжение и сопротивление:

I = 12 В / 600 Ом
I = 0,02 А = 20 мА (миллиампер)

Таким образом, ток в цепи составляет 20 мА.

Пример 2

Давайте попробуем другой пример.

Ниже у нас есть схема с резистором и аккумулятором снова. Но на этот раз мы не знаем напряжение батареи. Вместо этого мы представляем, что мы измерили ток в цепи и обнаружили, что он составляет 3 мА (миллиампер).

Рис. 7. Пример 2

Рис. 7. Пример 2

Сопротивление резистора составляет 600 Ом. Какое напряжение батареи? Используя треугольник Ома (рис. 3) получаем:

V = RI
V = 600 Ом * 3 мА
V = 1,8 В

Поэтому напряжение аккумулятора должно быть 1,8 В.

Более подробное о законе Ома вы можете почитать в моей статье на сайте: https://www.asutpp.ru/zakon-oma-dlya-uchastka-cepi.html

Закон ома и его практическое применение

Где и когда можно применять закон Ома?

Закон Ома в упомянутой форме справедлив в достаточно широких пределах для металлов. Он выполняется до тех пор, пока металл не начнет плавиться. Менее широкий диапазон применения у растворов (расплавов) электролитов и в сильно ионизированных газах (плазме).

Работая с электрическими схемами, иногда требуется определять падение напряжения на определенном элементе. Если это будет резистор с известной величиной сопротивления (она проставляется на корпусе), а также известен проходящий через него ток, узнать напряжение можно с помощью формулы Ома, не подключая вольтметр.

Про закон Ома в популярном изложении

Электрический ток и опасное напряжение невозможно услышать (за исключением гудящих высоковольтных линий и электроустановок). Токоведущие части, находящиеся под напряжением, ничем не отличаются по внешнему виду.

Невозможно узнать их и по запаху, и повышенной температурой в штатных режимах работы они не отличаются. Но включаем в безмолвную и тихую розетку пылесос, щелкаем выключателем – и энергия словно берется из ниоткуда, сама по себе, материализуясь в виде шума и компрессии внутри бытового прибора.

Опять же, если мы воткнем в разъемы розетки два гвоздя и возьмемся за них, то буквально всем своим телом ощутим реальность и объективность существования электрического тока. Делать это, конечно, настоятельно не рекомендуется.

Но примеры с пылесосом и гвоздями наглядно демонстрируют нам, что изучение и понимание основных законов электротехники способствует безопасности при обращении с бытовым электричеством, а также устранению суеверных предубеждений, связанных с электрическим током и напряжением.

Итак, рассмотрим один, самый ценный закон электротехники, который полезно знать. И попытаемся сделать это в как можно более популярной форме.

Открытие закона Ома

В 1827 г. немецкий физик Георг Симон Ом сформулировал закон, связывающий величины электрического тока, электродвижущей силы батареи и сопротивления простой электрической цепи, составленной из батареи и соединяющих ее полюса последовательно включенных разнородных проводников. Кроме того, он обнаружил, что различные вещества оказывают электрическому току различное сопротивление.

Ом экспериментально установил, что в последовательной цепи, составленной из нескольких участков с проводниками разного сопротивления, ток во всех участках одинаков, различна только разность потенциалов на проводниках, которую Ом назвал «падением напряжения».

Открытие закона Ома было очень важным этапом исследований электрических и магнитных явлений, имевших большое практическое значение. Закон Ома и открытые в дальнейшем законы Кирхгофа впервые дали возможность производить расчеты электрических цепей и легли в основу зародившейся электротехники.

Виды законов Ома

1. Дифференциальная форма записи закона Ома

Самый главный закон электротехники – это, конечно, закон Ома. О его существовании знают даже люди, не имеющие отношения к электротехнике. Но между тем вопрос «А знаешь ли ты закон Ома?» в технических ВУЗах является ловушкой для зарвавшихся и самонадеянных школяров. Товарищ, разумеется, отвечает, что закон Ома знает отлично, и тогда к нему обращаются с просьбой привести этот закон в дифференциальной форме. Тут-то и выясняется, что школяру или первокурснику еще учиться и учиться.

Однако дифференциальная форма записи закона Ома на практике почти неприменима. Она отражает зависимость между плотностью тока и напряженностью поля:

где G – это проводимость цепи; Е – напряженность электрического тока.

Все это – попытки выразить электрический ток, принимая во внимание только физические свойства материала проводника, без учета его геометрических параметров (длина, диаметр и тому подобное). Дифференциальная форма записи закона Ома – это чистая теория, знание ее в быту совершенно не требуется.

2. Интегральная форма записи закона Ома для участка цепи

Иное дело – интегральная форма записи. Она тоже имеет несколько разновидностей. Самой популярной из них является закон Ома для участка цепи: I=U/R

Говоря по-другому, ток в участке цепи всегда тем выше, чем больше приложенное к этому участку напряжение и чем меньше сопротивление этого участка.

Вот этот «вид» закона Ома просто обязателен к запоминанию для всех, кому хоть иногда приходится иметь дело с электричеством. Благо, и зависимость-то совсем простая. Ведь напряжение в сети можно считать неизменным.

Для розетки оно равно 220 вольт. Поэтому получается, что ток в цепи зависит только от сопротивления цепи, подключаемой к розетке. Отсюда простая мораль: за этим сопротивлением надо следить.

Короткие замыкания, которые у всех на слуху, случаются именно по причине низкого сопротивления внешней цепи. Предположим, что из-за неправильного соединения проводов в ответвительной коробке фазный и нулевой провода оказались напрямую соединены между собой. Тогда сопротивление участка цепи резко снизится практически до нуля, а ток так же резко возрастет до очень большой величины.

Если электропроводка выполнена правильно, то сработает автоматический выключатель, а если его нет, или он неисправен или подобран неправильно, то провод не справится с возросшим током, нагреется, расплавится и, возможно, вызовет пожар.

Но бывает, что приборы, включенные в розетку и отработавшие уже далеко не один час, становятся причиной короткого замыкания. Типичный случай – вентилятор, обмотки двигателя которого подверглись перегреву из-за заклинивания лопастей.

Изоляция обмоток двигателя не рассчитана на серьезный нагрев, она быстро приходит в негодность. В результате появляются межвитковые короткие замыкания, которые снижают сопротивление и, в соответствии с законом Ома, также ведут к увеличению тока.

Повышенный ток, в свою очередь, приводит изоляцию обмоток в полную негодность, и наступает уже не межвитковое, а самое настоящее, полноценное короткое замыкание. Ток идет помимо обмоток, сразу из фазного в нулевой провод. Правда, все сказанное может случиться только с совсем простым и дешевым вентилятором, не оборудованным тепловой защитой.

Шпаргалка по закону Ома для участка цепи:

Закон Ома для переменного тока

Надо отметить, что приведенная запись закона Ома описывает участок цепи с постоянным напряжением. В сетях переменного напряжения существует дополнительное реактивное сопротивление, а полное сопротивление приобретает значение квадратного корня из суммы квадратов активного и реактивного сопротивления.

Закон Ома для участка цепи переменного тока принимает вид: I=U/Z,

где Z – полное сопротивление цепи.

Но большое реактивное сопротивление свойственно, прежде всего, мощным электрическим машинам и силовой преобразовательной технике. Внутреннее электрическое сопротивление бытовых приборов и светильников практически полностью является активным. Поэтому в быту для расчетов можно пользоваться самой простой формой записи закона Ома: I=U/R.

3. Интегральная форма записи для полной цепи

Раз есть форма записи закона для участка цепи, то существует и закон Ома для полной цепи: I=E/(r+R).

Здесь r – внутреннее сопротивление источника ЭДС сети, а R – полное сопротивление самой цепи.

За физической моделью для иллюстрации этого подвида закона Ома далеко ходить не надо – это бортовая электрическая сеть автомобиля, аккумулятор в которой является источником ЭДС.

Нельзя считать, что сопротивление аккумулятора равно абсолютному нулю, поэтому даже при прямом замыкании между его клеммами (отсутствии сопротивления R) ток вырастет не до бесконечности, а просто до высокого значения.

Однако этого высокого значения, конечно, хватит для того, чтобы вызвать расплавление проводов и возгорание обшивки авто. Поэтому электрические цепи автомобилей защищают от короткого замыкания при помощи предохранителей.

Такой защиты может оказаться недостаточно, если замыкание произойдет до блока предохранителей относительно аккумулятора, или если вовсе один из предохранителей заменен на кусок медной проволоки. Тогда спасение только в одном – необходимо как можно быстрее разорвать цепь полностью, откинув «массу», то есть минусовую клемму.

4. Интегральная форма записи закона Ома для участка цепи, содержащего источник ЭДС

Следует упомянуть и о том, что есть и еще одна разновидность закона Ома – для участка цепи, содержащего источник ЭДС:

Здесь U – это разность потенциалов в начале и в окончании рассматриваемого участка цепи. Знак перед величиной ЭДС зависит от направленности ее относительно напряжения.

Воспользоваться законом Ома для участка цепи нередко приходится при определении параметров цепи, когда часть схемы недоступна для детального изучения и не интересует нас.

Допустим, она скрыта неразъемными деталями корпуса. В оставшейся схеме имеется источник ЭДС и элементы с известным сопротивлением. Тогда, замерив напряжение на входе неизвестной части схемы, можно вычислить ток, а после этого – и сопротивление неизвестного элемента.

Выводы

Таким образом, мы можем увидеть, что «простой» закон Ома далеко не так прост, как кому-то, возможно, казалось. Зная все формы интегральной записи законов Ома, можно понять и легко запомнить многие требования электробезопасности, а также приобрести уверенность в обращении с электричеством.

1.1 Закон Ома — Законы постоянного тока

Родился 16 марта 1787 г. в городе Эрлангене. В 1811 г. окончил Эрлангенский университет. Работал преподавателем математики и физики в различных гимназиях. В 1833 г. стал профессором Нюрнбергской высшей политехнической школы и вскоре был назначен её ректором.

С 1849 по 1852 г. — ректор Мюнхенского университета. Изучая связь электричества с магнетизмом, Ом в 1826 г. открыл один из важнейших законов — количественный закон цепи электрического тока. Учёный воспользовался методом французского инженера и физика Ш. О. Кулона, но несколько изменил его. Над проволокой с током он поместил магнитную стрелку, подвешенную на нити. При закручивании она удерживала стрелку в равновесии, а углом кручения измеряласьсила тока.
В этом эксперименте Ом установил, что:
1) сила тока постоянна в различных участках цепи;
2) сила тока убывает с увеличением длины провода и с уменьшением площади его поперечного сечения.

Физик также обнаружил ряд веществ, которые увеличивают сопротивление: в их числе серебро, свинец, медь, золото, цинк, олово, платина, палладий, железо.

Главный труд Ома — «Гальваническая цепь, разработанная математически» (1826 г.).

В 1827 г. учёный ввёл понятия «электродвижущая сила», «падение напряжения», «проводимость».

Помимо электричества Ом занимался акустикой, оптикой, кристаллооптикой. Он высказал мысль о сложном составе звука и экспериментально установил, что человеческое ухо воспринимает как простой тон лишь тот звук, который вызван простым синусоидальным колебанием. Остальные звуки воспринимаются как основной тон и добавочные обертоны. Открытие получило название акустического закона Ома.

Умер 7 июля 1854 г. в Мюнхене.

Закон Ома

Текстовое поле

Сила тока I, текущего по однородному металлическому проводнику (т. е. проводнику, в котором не действуют сторонние силы), пропорционально напряжению U на концах проводника:
I = U/R   (1)

где R — электрическое сопротивление проводника.
Уравнение (1) выражает закон Ома для участка цепи (не содержащего источника тока): сила тока в проводнике прямо пропорциональна приложенному напряжению и обратно пропорционально сопротивлению проводника.
Участок цепи, в котором не действуют э.д.с. (сторонние силы) называют однородным участком цепи, поэтому эта формулировка закона Ома справедлива для однородного участка цепи.
Теперь рассмотрим неоднородный участок цепи, где действующую э.д.с. на участке 1 — 2 обозначим через Ε12, а приложенную на концах участка разность потенциалов — через φ1 — φ2.
Если ток проходит по неподвижным проводникам, образующим участок 1-2, то работа A12 всех сил (сторонних и электростатических), совершаемая над носителями тока, по закону сохранения и превращения энергии равна теплоте, выделяющейся на участке. Работа сил, совершаемая при перемещении заряда Q0 на участке 1- 2:
A12 = Q0E12 + Q0(φ1 — φ2) (2)
Э.д.с. E12, как и сила тока I, — величина скалярная. Её необходимо брать либо с положительным, либо с отрицательным знаком в зависимости от знака работы, совершаемой сторонними силами. Если е.д.с. способствует движению положительных зарядов в выбранном направлении (в направлении 1-2), то E12 > 0. Если э.д.с. препятствует движению положительных зарядов в данном направлении, то E12 < 0.
За время t в проводнике выделяется теплота:
Q =I2Rt = IR(It) = IRQ0 (3)
Из формул (2) и (3) получим:
IR = (φ1 — φ2) + E12 (4)
Откуда
I = (φ1 — φ2 + E12)/R (5)
Выражение ,(4) или (5) представляет собой закон Ома для неоднородного участка цепи в интегральной форме, который является обобщённым законом Ома.
Если на данном участке цепи источник тока отсутствует (E12 = 0), то из (5) приходим к закону Ома для однородного участка цепи
I = (φ1 — φ2)/R = U / R
Если же электрическая цепь замкнута, то выбранные точки 1 и 2 совпадают, φ1 = φ2; тогда из (5) получаем закон Ома для замкнутой цепи:
I =E /R,
где E — э.д.с., действующая в цепи, R — суммарное сопротивление всей цепи. В общем случае R = r + R1, где r — внутреннее сопротивление источника тока, R1 — сопротивление внешней цепи. Поэтому закон Ома для замкнутой цепи будет иметь вид:
I = E / (r+R1).
Если цепь разомкнута, в ней ток отсутствует (I = 0), то из закона Ома (4) получим, что (φ1 — φ2) = E12 , т.е. э.д.с., действующая в разомкнутой цепи, равна разности потенциалов на её концах. Следовательно, для того чтобы найти э.д.с. источника тока, надо измерить разность потенциалов на его клеммах при разомкнутой цепи.

Закон ома наблюдение.

Применение закона ома на практике

В 1826 году немецкий ученый Георг Ом совершил открытие и описал
эмпирический закон о соотношении между собой таких показателей как сила тока, напряжение и особенности проводника в цепи. Впоследствии, по имени ученого он стал называться закон Ома.

В дальнейшем выяснилось, что эти особенности не что иное, как сопротивление проводника, возникающее в процессе его контакта с электричеством. Это внешнее сопротивление (R). Есть также внутреннее сопротивление (r), характерное для источника тока.

Закон Ома для участка цепи

Согласно обобщенному закону Ома для некоторого участка цепи, сила тока на участке цепи прямо пропорциональна напряжению на концах участка и обратно пропорциональна сопротивлению.

Где U – напряжение концов участка,I– сила тока, R– сопротивление проводника.

Беря во внимание вышеприведенную формулу, есть возможность найти неизвестные значенияUиR, сделав несложные математические операции.

Данные выше формулы справедливы лишь когда сеть испытывает на себе одно сопротивление.

Закон Ома для замкнутой цепи

Сила тока полной цепи равна ЭДС, деленной на сумму сопротивлений однородного и неоднородного участков цепи.

Замкнутая сеть имеет одновременно сопротивления внутреннего и внешнего характера. Поэтому формулы отношения будут уже другими.

Где E – электродвижущая сила (ЭДС), R- внешнее сопротивление источника, r-внутреннее сопротивление источника.

Закон Ома для неоднородного участка цепи

Замкнутая электрическая сеть содержит участки линейного и нелинейного характера. Участки, не имеющие источника тока и не зависящие от стороннего воздействия являются линейными, а участки, содержащие источник – нелинейными.

Закон Ома для участка сети однородного характера был изложен выше. Закон на нелинейном участке будет иметь следующий вид:

I = U/ R = f1 – f2 + E/ R

Где f1 – f2 – разница потенциалов на конечных точках рассматриваемого участка сети

R – общее сопротивление нелинейного участка цепи

ЭДС нелинейного участка цепи бывает больше нуля или меньше. Если направление движения тока, идущего из источника с движением тока в электрической сети, совпадают, будет преобладать движение зарядов положительного характера и ЭДС будет положительная. В случае же совпадения направлений, в сети будет увеличено движение отрицательных зарядов, создаваемых ЭДС.

Закон Ома для переменного тока

При имеющейся в сети емкости или инертности, необходимо учитывать при проводимых вычислениях, что они выдают свое сопротивление, от действия которого ток приобретает переменный характер.

Закон Ома для переменного тока выглядит так:

где Z – сопротивление по всей длине электрической сети. Его еще называют импеданс. Импеданс составляют сопротивления активного и реактивного характера.

Закон Ома не является основным научным законом, а лишь эмпирическим отношением, причем в некоторых условиях оно может не соблюдаться:

  • Когда сеть обладает высокой частотой, электромагнитное поле меняется с большой скоростью, и при расчетах необходимо учитывать инертность носителей заряда;
  • В условиях низкой температуры с веществами, которые обладают сверхпроводимостью;
  • Когда проводник сильно нагревается проходящим напряжением, отношение тока к напряжению становится переменным и может не соответствовать общему закону;
  • При нахождении под высоким напряжением проводника или диэлектрика;
  • В светодиодных лампах;
  • В полупроводниках и полупроводниковых приборах.

В свою очередь элементы и проводники, соблюдающие закон Ома, называются омическими.

Закон Ома может дать объяснение некоторым явлениям природы. Например, когда мы видим птиц, сидящих на высоковольтных проводах, у нас возникает вопрос – почему на них не действует электрический ток? Объясняется это довольно просто. Птицы, сидя на проводах, представляют собой своеобразные проводники. Большая часть напряжения приходится на промежутки между птицами, а та доля, что приходится на сами «проводники» не представляет для них опасности.

Но это правило работает лишь при единичном соприкосновении. Если птица заденет клювом или крылом провод или телеграфный столб, она неминуемо погибнет от огромного количества напряжения, которое несут в себе эти участки. Такие случаи происходят повсеместно. Поэтому в целях безопасности в некоторых населенных пунктах установлены специальные приспособления, защищающие птиц от опасного напряжения. На таких насестах птицы находятся в полной безопасности.

Закон Ома также широко применятся на практике. Электричество смертельно опасно для человека при одном лишь касании к оголенному проводу. Но в некоторых случаях сопротивление человеческого тела может быть разным.

Так, например, сухая и неповрежденная кожа обладает большим сопротивлением к воздействию электричества нежели рана или кожа, покрытая потом. В следствие переутомления, нервного напряжения и опьянения, даже при небольшом напряжении тока человек может получить сильный удар током.

В среднем, сопротивление тела человека – 700 Ом, значит, для человека является безопасным напряжение в 35 В. Работая с большим напряжением, специалисты используют .

Отправить свою хорошую работу в базу знаний просто. Используйте форму, расположенную ниже

Студенты, аспиранты, молодые ученые, использующие базу знаний в своей учебе и работе, будут вам очень благодарны.

Размещено на http://www.allbest.ru/

МИНИСТЕРСТВО ОБРАЗОВАНИЯ РЕСПУБЛИКИ БЕЛАРУСЬ

Кафедра естественнонаучных дисциплин

Реферат

Закон Ома

Выполнил:

Иванов М. А.

Введение

1. Общий вид закона Ома

2. История открытия закона Ома, краткая биография ученого

3. Виды законов Ома

4. Первые исследования сопротивления проводников

5. Электрические измерения

Заключение

Литература, другие источники информации

Введение

Явления, связанные с электричеством были замечены в древнем Китае, Индии и древней Греции за несколько столетий до начала нашей эры. Около 600 года до н.э., как гласят сохранившиеся предания, древнегреческому философу Фалесу Милетскому было известно свойство янтаря, натертого об шерсть, притягивать легкие предметы. Кстати словом “ электрон” древние греки называли янтарь. От него же пошло и слово “электричество”. Но греки всего лишь наблюдали явления электричества, но не могли объяснить.

XIX век был полон открытий связанных с электричеством. Одно открытие порождало целую цепь открытий в течении нескольких десятилетий. Электричество из предмета исследования начало превращаться в предмет потребления. Началось его широкое внедрение в различные области производства. Были изобретены и созданы электрические двигатели, генераторы, телефон, телеграф, радио. Начинается внедрение электричества в медицину.

Напряжение, сила тока и сопротивление — физические величины, характеризующие явления, происходящие в электрических цепях. Эти величины связаны между собой. Эту связь впервые изучил немецкий физик 0м. Закон Ома был открыт в 1826 .

1. Общий вид закона Ома

Закон Ома звучит так: Сила тока на участке цепи прямо пропорциональна напряжению на этом участке (при заданном сопротивлении) и обратно пропорциональна сопротивлению участка (при заданном напряжении): I = U / R, из формулы следует, что U = IЧR и R = U / I. Так как сопротивление данного проводника не зависит ни от напряжения, ни от силы тока, то последнюю формулу надо читать так: сопротивление данного проводника равно отношению напряжения на его концах к силе протекающего по нему тока. В электрических цепях чаще всего проводники (потребители электрической энергии) соединяются последовательно (например, лампочки в елочных гирляндах) и параллельно (например, домашние электроприборы).

При последовательном соединении сила тока в обоих проводниках (лампочках) одинакова: I = I1 = I2, напряжение на концах рассматриваемого участка цепи складывается из напряжения на первой и второй лампочках: U = U1 + U2. Общее сопротивление участка равно сумме сопротивлений лампочек R = R1 + R2.

При параллельном соединении резисторов напряжение на участке цепи и на концах резисторов одинаково: U = U1 = U2. сила тока в неразветвленной части цепи равна сумме сил токов в отдельных резисторах: I = I1 + I2. Общее сопротивление участка меньше сопротивления каждого резистора.

Если сопротивления резисторов одинаковы (R1 = R2) то общее сопротивление участка Если в цепь включено параллельно три и более резисторов, то общее сопротивление может быть —

найдено по формуле: 1/R = 1/R1 + 1/R2 + … + 1/RN. Параллельно соединяются сетевые потребители, которые рассчитаны на напряжение, равное напряжению сети.

Итак, Закон Ома устанавливает зависимость между силой тока I в проводнике и разностью потенциалов (напряжением) U между двумя фиксированными точками (сечениями) этого проводника:

Коэффициент пропорциональности R , зависящий от геометрических и электрических свойств проводника и от температуры, называется омическим сопротивлением или просто сопротивлением данного участка проводника.

2. История открытия закона Ома, краткая биография ученого

Георг Симон Ом родился 16 марта 1787 года в Эрлангене, в семье потомственного слесаря. После окончания школы Георг поступил в городскую гимназию. Гимназия Эрлангена курировалась университетом. Занятия в гимназии вели четыре профессора. Георг, закончив гимназию, весной 1805 года приступил к изучению математики, физики и философии на философском факультете Эрлангенского университета.

Проучившись три семестра, он принял приглашение занять место учителя математики в частной школе швейцарского городка Готтштадта.

В 1811 году он возвращается в Эрланген, заканчивает университет и получает степень доктора философии. Сразу же по окончании университета ему была предложена должность приват-доцента кафедры математики этого же университета.

В 1812 году Ом был назначен учителем математики и физики школы в Бамберге. В 1817 году он публикует свою первую печатную работу, посвященную методике преподавания «Наиболее оптимальный вариант преподавания геометрии в подготовительных классах». Ом занялся исследованиями электричества. В основу своего электроизмерительного прибора Ом заложил конструкцию крутильных весов Кулона. Результаты своих исследований Ом оформил в виде статьи под названием «Предварительное сообщение о законе, по которому металлы проводят контактное электричество». Статья была опубликована в 1825 году в «Журнале физики и химии», издаваемом Швейггером. Однако выражение, найденное и опубликованное Омом, оказалось неверным, что стало одной из причин его длительного непризнания. Приняв все меры предосторожности, заранее устранив все предполагаемые источники ошибок, Ом приступил к новым измерениям.

Появляется в свет его знаменитая статья «Определение закона, по которому металлы проводят контактное электричество, вместе с наброском теории вольтаического аппарата и мультипликатора Швейггера», вышедшая в 1826 году в «Журнале физики и химии».

В мае 1827 года «Теоретические исследования электрических цепей» объемом в 245 страниц, в которых содержались теперь уже теоретические рассуждения Ома по электрическим цепям. В этой работе ученый предложил характеризовать электрические свойства проводника его сопротивлением и ввел этот термин в научный обиход. Ом нашел более простую формулу для закона участка электрической цепи, не содержащего ЭДС: «Величина тока в гальванической цепи прямо пропорциональна сумме всех напряжений и обратно пропорциональна сумме приведенных длин. При этом общая приведенная длина определяется как сумма всех отдельных приведенных длин для однородных участков, имеющих различную проводимость и различное поперечное сечение».

В 1829 году появляется его статья «Экспериментальное исследование работы электромагнитного мультипликатора», в которой были заложены основы теории электроизмерительных приборов. Здесь же Ом предложил единицу сопротивления, в качестве которой он выбрал сопротивление медной проволоки длиной 1 фут и поперечным сечением в 1 квадратную линию.

В 1830 году появляется новое исследование Ома «Попытка создания приближенной теории униполярной проводимости». Только в 1841 году работа Ома была переведена на английский язык, в 1847 году — на итальянский, в 1860 году — на французский.

16 февраля 1833 года, через семь лет после выхода из печати статьи, в которой было опубликовано его открытие, Ому предложили место профессора физики во вновь организованной политехнической школе Нюрнберга. Ученый приступает к исследованиям в области акустики. Результаты своих акустических исследований Ом сформулировал в виде закона, получившего впоследствии название акустического закона Ома.

Раньше всех из зарубежных ученых закон Ома признали русские физики Ленц и Якоби. Они помогли и его международному признанию. При участии русских физиков, 5 мая 1842 года Лондонское Королевское общество наградило Ома золотой медалью и избрало своим членом.

В 1845 году его избирают действительным членом Баварской академии наук. В 1849 году ученого приглашают в Мюнхенский университет на должность экстраординарного профессора. В этом же году он назначается хранителем государственного собрания физико-математических приборов с одновременным чтением лекций по физике и математике. В 1852 году Ом получил должность ординарного профессора. Ом скончался 6 июля 1854 года. В 1881 году на электротехническом съезде в Париже ученые единогласно утвердили название единицы сопротивления — 1 Ом.

3. Виды законов Ома

Существует несколько видов закона Ома.

Закон Ома для однородного участка цепи (не содержащего источника тока): сила тока в проводнике прямо пропорциональна приложенному напряжению и обратно пропорциональна сопротивлению проводника:

Закон Ома для полной цепи — сила тока в цепи пропорциональна действующей в цепи ЭДС и обратно пропорциональна сумме сопротивлений цепи и внутреннего сопротивления источника.

где I — сила тока

E — электродвижущая сила

R — внешнее сопротивление цепи (т.е. сопротивление той

части цепи, которая находится за пределами источника ЭДС)

ЭДС — работа сторонних сил (т.е. сил неэлектрического происхождения) по перемещению заряда в цепи отнесенная к величине этого заряда.

Единицы измерения:

ЭДС — вольты

Ток — амперы

Сопротивления (R и r) — омы

Применяя основной закон электрической цепи (закон Ома), можно объяснить многие природные явления, которые на первый взгляд кажутся загадочными и парадоксальными. Например, всем известно, что любой контакт человека с электрическими проводами, находящимися под напряжением, является смертельно опасным. Всего лишь одно прикосновение к оборвавшемуся проводу высоковольтной линии способно убить электрическим током человека или животное. Но в то же время, мы постоянно видим, как птицы спокойно усаживаются на высоковольтные провода электропередач, и ничто не угрожает жизни этих живых существ. Тогда как же найти объяснение такому парадоксу?

А объясняется подобное явление довольно просто, если представить, что находящаяся на электрическом проводе птица — это один из участков электрической сети, сопротивление второго значительно превышает сопротивление другого участка той же цепи (то есть небольшого промежутка между лапками птицы). Следовательно, сила электрического тока, воздействующая на первый участок цепи, то есть на тело птицы, будет совершенно безопасной для неё. Однако полная безопасность гарантирована ей только при соприкосновении с участком высоковольтного провода. Но стоит только птице, усевшейся на линию электропередач, задеть крылом или клювом провод или какой-либо предмет, находящийся вблизи от провода (например, телеграфный столб), то птица неминуемо погибнет. Ведь столб непосредственно связан с землёй, и поток электрических зарядов, переходя на тело птицы, способен мгновенно убить её, стремительно двигаясь по направлению к земле. К сожалению, по этой причине в городах гибнет немало птиц.

Для защиты пернатых от губительного воздействия электричества зарубежными учеными были разработаны специальные устройства — насесты для птиц, изолированные от электрического тока. Такие приспособления размещали на высоковольтных линиях электропередач. Птицы, усаживаясь на изолированный насест, могут без всякого риска для жизни прикасаться клювом, крыльями или хвостом к проводам, столбам или кронштейнам. Наибольшим сопротивлением обладает поверхность верхнего, так называемого рогового слоя кожи человека. Сопротивление сухой и неповреждённой кожи может достигать 40 000 — 100 000 Ом. Роговой слой кожи очень незначителен, всего 0,05 — 0,2 мм. и легко пробивается напряжением 250 В. При этом сопротивление уменьшается в сто раз и падает тем скорее, чем дольше действует на тело человека ток. Резко, до 800 — 1000 Ом, уменьшают сопротивление тела человека повышенная потливость кожного покрова, переутомление, нервное возбуждение, опьянение. Этим объясняется, что порой даже небольшое напряжение может вызвать поражение электрическим током. Если, например, сопротивление тела человека равно 700 Ом, то опасным будет напряжение всего в 35 В. Именно поэтому, например, специалисты-электрики даже при работе с напряжением 36 В применяют изолирующие защитные средства — резиновые перчатки или инструмент с изолированными ручками.

Закон Ома выглядит настолько просто, что трудности, которые пришлось преодолеть при его установлении, упускают из виду и забывают. Закон Ома нелегко проверить, и его нельзя рассматривать как очевидную истину; действительно, для многих материалов он не выполняется.

В чем же все-таки заключаются эти трудности? Разве нельзя проверить, что дает изменение числа элементов вольтова столба, определяя ток при разном числе элементов?

Дело в том, что, когда мы берем разное число элементов, мы меняем всю цепь, т.к. дополнительные элементы имеют и дополнительное сопротивление. Поэтому необходимо найти способ изменять напряжение, не меняя самой батареи. Кроме того, разный по величине ток нагревает проволоку до развой температуры, и этот эффект тоже может влиять на силу тока. Ом (1787—1854) преодолел эти трудности, воспользовавшись явлением термоэлектричества, которое открыл Зеебек (1770—1831) в 1822 г.

Таким образом, Ом показал, что ток пропорционален напряжению и обратно пропорционален полному сопротивлению цепи. Это был простой результат для сложного эксперимента. Так по крайней мере должно казаться нам сейчас.

Современники Ома, в особенности его соотечественники, полагали иначе: возможно, именно простота закона Ома вызывала у них подозрение. Ом столкнулся с затруднениями в cлужебной карьере, испытывал нужду; особенно угнетало Ома то, что не признавались его труды. К чести Великобритании, и в особенности Королевского общества, нужно сказать, что работа Ома получила там заслуженное признание. Ом входит в число тех великих людей, имена которых часто встречаются написанными с маленькой буквы: название «ом» было присвоено единице сопротивления.

4. Первые исследования сопротивления проводников

Что такое проводник? Это чисто пассивная составная часть электрической цепи, отвечали первые исследователи. Заниматься его исследованием — значит попросту ломать себе голову над ненужными загадками, т.к. только источник тока представляет собой активный элемент.

Такой взгляд на вещи объясняет нам, почему ученые, по крайней мере до 1840 г., почти не проявляли интереса к тем немногим работам, которые проводились в этом направлении.

Так, на втором съезде итальянских ученых, состоявшемся в Турине в 1840 г. (первый собирался в Пизе в 1839 г. и приобрел даже некое политическое значение), выступая в прениях по докладу, представленному Марианини, Де ла Рив утверждал, что проводимость большинства жидкостей не является абсолютной, «а скорее относительной и изменяется с изменением силы тока». А ведь закон Ома был опубликован за 15 лет до этого!

Среди тех немногих ученых, которые первыми стали заниматься вопросом проводимости проводников после изобретения гальванометра, был Стефано Марианини (1790—1866).

К своему открытию он пришел случайно, изучая напряжение батарей. Он заметил, что с увеличением числа элементов вольтова столба электромагнитное воздействие на стрелку не увеличивается заметным образом. Это заставило Марианини сразу же подумать, что каждый вольтов элемент представляет собой препятствие для прохождения тока. Он делал опыты с парами «активными» и «неактивными» (т. е. состоящими из двух медных пластинок, разделенных влажной прокладкой) и опытным путем нашел отношение, в котором современный читатель узнает частный случай закона Ома, когда сопротивление внешней цепи не принимается во внимание, как это и было в опыте Марианини.

Георг Симон Ом (1789—1854) признавал заслуги Марианини, хотя его труды и не оказали Ому непосредственной помощи в работе. Ом вдохновлялся в своих исследованиях работой («Аналитическая теория тепла», Париж, 1822 г.) Жана Батиста Фурье (1768—1830)—одной из самых значительных научных работ всех времен, очень быстро получившей известность и высокую оценку среди математиков и физиков того времени. Ому пришла мысль, что механизм «теплового потока», о котором говорит Фурье, можно уподобить электрическому току в проводнике. И подобно тому как в теории Фурье тепловой поток между двумя телами или между двумя точками одного и того же тела объясняется разницей температур, точно так же Ом объясняет разницей «электроскопических сил» в двух точках проводника возникновение электрического тока между ними.

Придерживаясь такой аналогии, Ом начал свои экспериментальные исследования с определения относительных величин проводимости различных проводников. Применив метод, который стал теперь классическим, он подключал последовательно между двумя точками цепи тонкие проводники из различных материалов одинакового диаметра и изменял их длину так, чтобы получалась определенная величина тока. Первые результаты, которые ему удалось получить, сегодня кажутся довольно скромными. закон ом электрический гальванометр

Историки поражаются, например, тем, что по измерениям Ома серебро обладает меньшей проводимостью, чем медь и золото, и снисходительно принимают данное впоследствии самим Омом объяснение, согласно которому опыт проводился с серебряной проволокой, покрытой слоем масла, и это вводило в заблуждение относительно точного значения диаметра.

В то время имелось множество источников ошибок при проведении опытов (недостаточная чистота металлов, трудность калибровки проволоки, трудность точных измерений и т. п.). Важнейшим же источником ошибок была поляризация батарей. Постоянные (химические) элементы тогда еще не были известны, так что за время, необходимое для измерений, электродвижущая сила элемента существенно менялась. Именно эти причины, вызывавшие ошибки, привели к тому, что Ом на основании своих опытов пришел к логарифмическому закону зависимости силы тока от сопротивления проводника, включенного между двумя точками цепи. После опубликования первой статьи Ома Поггендорф посоветовал ему отказаться от химических элементов и воспользоваться лучше термопарой медь — висмут, незадолго до этого введенной Зеебеком.

Ом прислушался к этому совету и повторил свои опыты, собрав установку с термоэлектрической батареей, во внешнюю цепь которой включались последовательно восемь медных проволок одинакового диаметра, но разной длины. Силу тока он измерял с помощью своего рода крутильных весов, образуемых магнитной стрелкой, подвешенной на металлической нити. Когда ток, параллельный стрелке, отклонял ее, Ом закручивал нить, на которой она была подвешена, пока стрелка не оказывалась в своем обычном положении;

сила тока считалась пропорциональной углу, на который закручивалась нить. Ом пришел к выводу, что результаты опытов, проведенных с восемью различными проволоками, «могут быть выражены очень хорошо уравнением

где X означает интенсивность магнитного действия проводника, длина которого равна х, а а и b — константы, зависящие соответственно от возбуждающей силы и от сопротивления остальных частей цепи».

Условия опыта менялись: заменялись сопротивления и термоэлектрические пары, но результаты все равно сводились к приведенной выше формуле, которая очень просто переходит в известную нам, если X заменить силой тока, a —электродвижущей силой и b+x,—общим сопротивлением цепи.

Получив эту формулу, Ом пользуется ею для изучения действия мультипликатора Швейггера на отклонение стрелки и для изучения тока, который проходит во внешней цепи батареи элементов, в зависимости от того, как они соединены — последовательно или параллельно. Таким образом он объясняет (как это делается теперь в учебниках), чем определяется внешний ток батареи,— вопрос, который был довольно темным для первых исследователей. Ом надеялся, что его экспериментальные работы откроют ему путь в университет, чего он так желал. Однако статьи прошли незамеченными. Тогда он оставил место преподавателя в кельнской гимназии и отправился в Берлин, чтобы теоретически осмыслить полученные результаты. В 1827 г. в Берлине он опубликовал свой главный труд «Die galvanische Kette, mathe-matisch bearbeitet» («Гальваническая цепь, разработанная математически»).

Эта теория, при разработке которой он вдохновлялся, как мы уже указывали, аналитической теорией теплоты Фурье, вводит понятия и точные определения электродвижущей силы, или «электроскопической силы», как ее называет Ом, электропроводности (Starke der Leitung) и силы тока. Выразив выведенный им закон в дифференциальной форме, приводимой современными авторами, Ом записывает его и в конечных величинах для частных случаев конкретных электрических цепей, из которых особенно важна термоэлектрическая цепь. Исходя из этого, он формулирует известные законы изменения электрического напряжения вдоль цепи.

Но теоретические исследования Ома также остались незамеченными, а если кто-нибудь и писал о них, то лишь для того, чтобы, высмеять «болезненную фантазию, единственной целью которой является стремление принизить достоинство природы». И лишь лет десять спустя его гениальные работы постепенно начали пользоваться должным признанием: в

Германии их оценили Поггендорф и Фехнер, в России — Ленц, в Англии — Уитстон, в Америке — Генри, в Италии — Маттеуччи.

Одновременно с опытами Ома во Франции проводил свои опыты А. Беккерель, а в Англии — Барлоу. Опыты первого особенно замечательны введением дифференциального гальванометра с двойной обмоткой рамки и применением «нулевого» метода измерения. Опыты же Барлоу стоит упомянуть потому, что они экспериментально подтвердили постоянство силы тока во всей цепи. Этот вывод был проверен и распространен на внутренний ток батареи Фехнером в 1831 г., обобщен в 1851 г. Рудольфом Кольраушем

(180Э—1858) на жидкие проводники, а затем еще раз подтвержден тщательными опытами Густава Нидмана (1826—1899).

5. Электрические измерения

Беккерель применил дифференциальный гальванометр для сравнения электрических сопротивлений. На основе проведенных им исследований он сформулировал известный закон зависимости сопротивления проводника от его длины и сечения. Эти работы были продолжены Пуйе и описаны им в последующих изданиях его известных «Elements de

physique experimentale» («Основы экспериментальной физики»), первое издание которых появилось в 1827 г. Сопротивления определялись методом сравнения.

Уже в 1825 г. Марианини показал, что в разветвляющихся цепях электрический ток распределяется по всем проводникам независимо от того, из какого материала они сделаны, вопреки утверждению Вольты, который полагал, что если одна ветвь цепи образуется металлическим проводником, а остальные — жидкими, то весь ток должен проходить по металлическому проводнику. Араго и Пуйе популяризировали во Франции наблюдения Марианини. Не зная еще закона Ома, Пуйе в 1837 г. воспользовался этими наблюдениями и законами Беккереля, чтобы показать, что проводимость цепи, эквивалентной двум

разветвленным цепям, равна сумме проводимостей обеих цепей. Этой работой Пуйе положил начало изучению разветвленных цепей. Пуйе установил для них целый ряд терминов,

которые живы и до сих пор, и некоторые частные законы, обобщенные Кирхгофом в 1845 г. в его известных «принципах»..

Самый большой толчок для проведения электрических измерений, и в частности измерений сопротивления, был дан возросшими потребностями техники, и в первую очередь проблемами, возникшими с появлением электрического телеграфа. Впервые мысль об использовании электричества для передачи сигналов на расстояние родилась еще в XVIII веке. Вольта описал проект телеграфа, а Ампер еще в 1820 г. предлагал использовать электромагнитные явления для передачи сигналов. Идея Ампера была подхвачена многими учеными и техниками: в 1833 г. Гаусс и Вебер построили в Геттингене простейшую телеграфную линию, соединявшую астрономическую обсерваторию и физическую лабораторию. Но практическое применение телеграф получил благодаря американцу Самуэлу Морзе (1791—1872), которому в 1832 г. пришла удачная мысль создать телеграфный алфавит, состоящий всего из двух знаков. После многочисленных попыток Морзе в 1835 г. наконец удалось построить частным образом первую грубую модель телеграфа в Нью-Йоркском университете. В 1839 г. была проведена экспериментальная

линия между Вашингтоном и Балтиморой, а в 1844 г. возникла организованная Морзе первая американская компания по коммерческой эксплуатации нового изобретения. Это было также первое практическое применение результатов научных изысканий в области электричества.

В Англии изучением и усовершенствованием телеграфа занялся Чарльз Уитстон (1802—1875), бывший мастер по изготовлению музыкальных инструментов. Понимая важность

измерений сопротивления, Уитстон стал искать наиболее простые и точные методы таких измерений. Бывший в то время в ходу метод сравнения, как мы видели, давал ненадежные результаты, главным образом из-за отсутствия стабильных источников питания. Уже в 1840 г. Уитстон нашел способ измерения сопротивления независимо от постоянства электродвижущей силы и показал свое устройство Якоби. Однако статья, в которой это устройство описано и которую вполне можно назвать первой работой в области электротехники, появилась лишь в 1843 г. В этой статье дано описание знаменитого «мостика», названного затем в честь Уитстона. Фактически такое устройство было описано —

еще в 1833 г. Гюнтером Кристи и независимо от него в 1840 г. Марианини; оба они предлагали метод сведения к нулю, но их теоретические объяснения, при которых не учитывался закон Ома, оставляли желать лучшего.

Уитстон же был поклонником Ома и очень хорошо знал его закон, так что данная им теория «мостика Уитстона» ничем не отличается от приводимой сейчас в учебниках. Кроме того, Уитстон, чтобы можно было быстро и удобно изменять сопротивление одной стороны мостика для получения нулевой силы тока в гальванометре, включенном в диагональное плечо мостика, сконструировал три типа реостатов (само это слово было предложено им по

аналогии с «реофором», введенным Ампером, в подражание которому Пекле ввел также термин «реометр»). Первый тип реостата с подвижной скобкой, применяемый и сейчас, был создан Уитстоном по аналогии со схожим приспособлением, применявшимся Якоби в 1841 г. Второй тип реостата имел вид деревянного цилиндра, вокруг которого была намотана часть подключенного в цепь провода, который легко перематывался с деревянного цилиндра на бронзовый. Третий тип реостата был похож на «магазин сопротивлений», который Эрнст

Вернер Сименс (1816—1892), ученый и промышленник, в 1860 г. улучшил и широко распространил. «Мостик Уитстона» дал возможность измерять электродвижущие силы и сопротивления.

Создание подводного телеграфа, пожалуй, еще более, нежели воздушного телеграфа, потребовало разработки методов электрических измерений. Опыты с подводным телеграфом начались еще в 1837 г., и одной из первых проблем, которую предстояло разрешить, было определение скорости распространения тока. Еще в 1834 г. Уитстон с помощью вращающихся зеркал, о чем мы уже упоминали в гл. 8, произвел первые измерения этой скорости, но полученные им результаты противоречили результатам Латимера Кларка, а последние в свою очередь не соответствовали более поздним исследованиям других ученых.

В 1855 г. Уильям Томсон (получивший впоследствии титул лорда Кельвина) объяснил причину всех этих расхождений. Согласно Томсону, скорость тока в проводнике не имеет определенной величины. Подобно тому как скорость распространения тепла в стержне зависит от материала, так и скорость тока в проводнике зависит от произведения его сопротивления на электрическую емкость. Следуя этой своей теории, которая в»»его времена

подверглась ожесточенной критике, Томсон занялся проблемами, связанными с подводным телеграфом.

Первый трансатлантический кабель, соединивший Англию и Америку, функционировал около месяца, но затем испортился. Томсон рассчитал новый кабель, провел многочисленные измерения сопротивления и емкости, придумал новые передающие аппараты, из коих следует упомянуть астатический отражательный гальванометр, замененный «сифонным регистратором» его же изобретения. Наконец, в 1866 г. новый трансатлантический кабель успешно вступил в действие. Созданию этого первого большого электротехнического сооружения сопутствовала разработка системы единиц электрических и магнитных измерений.

Основа электромагнитной метрики была заложена Карлом Фридрихом Гауссом (1777—1855) в его знаменитой статье «Intensitas vis magneticae terrestris ad mensuram absolutam revocata» («Величина силы земного магнетизма в абсолютных мерах»), опубликованной в 1832 г. Гаусс заметил, что различные магнитные единицы измерения несоотносимы между

собой, по крайней мере в большей своей части, и поэтому предложил систему абсолютных единиц, основанную на трех основных единицах механики: секунде (единице времени), миллиметре (единице длины) и миллиграмме (единице массы). Через них он выразил все остальные физические единицы и придумал ряд измерительных приборов, в частности магнетометр для измерения в абсолютных единицах земного магнетизма. Работу Гаусса продолжил Вебер, который построил много собственных приборов и приборов, задуманных еще Гауссом. Постепенно, особенно благодаря работам Максвелла, проводившимся в созданной Британской ассоциацией специальной комиссии по измерениям, которая издавала ежегодные отчеты с 1861 по 1867 г., возникла идея создать единые системы мер, в частности систему электромагнитных и электростатических мер.

Мысли о создании таких абсолютных систем единиц были подробно изложены в историческом отчете за 1873 г. второй комиссии Британской ассоциации. Созванный в Париже в 1881 г. Международный конгресс впервые установил международные единицы измерения, присвоив каждой из них название в честь какого-нибудь великого физика. Большая часть этих названий сохраняется до сих пор: вольт, ом, ампер, джоуль и т. д. После

многих перипетий в 1935 г. была введена международная система Джорджи, или MKSQ, которая принимает за основные единицы метр, килограмм-массу, секунду и ом.

С «системами» единиц связаны «формулы размерностей», примененные впервые Фурье в его аналитической теории тепла (1822 г.) и распространенные Максвеллом, которым и установлены применяемые в них обозначения. Метрология прошлого века, основывавшаяся на стремлении объяснить все явления с помощью механических моделей, придавала большое значение формулам размерностей, в которых она хотела видеть не больше и не меньше как ключ к тайнам природы. При этом выдвигался ряд утверждений почти догматического характера. Так, чуть ли не обязательным догматом было требование, чтобы основных величин было непременно три. Но к концу века начали понимать, что формулы размерностей — это чистая условность, вследствие чего интерес к теориям размерностей стал постепенно падать.

Заключение

О значении исследований Ома хорошо сказал профессор физики Мюнхенского университета Е. Ломмель при открытии памятника ученому в 1895 году:

«Открытие Ома было ярким факелом, осветившим ту область электричества, которая до него была окутана мраком. Ом указал единственно правильный путь через непроходимый лес непонятных фактов. Замечательные успехи в развитии электротехники, за которыми мы с удивлением наблюдали в последние десятилетия, могли быть достигнуты только на основе открытия Ома. Лишь тот в состоянии господствовать над силами природы и управлять ими, кто сумеет разгадать законы природы, Ом вырвал у природы так долго скрываемую ею тайну и передал ее в руки современников».

Список используемых источников

Дорфман Я. Г. Всемирная история физики . М., 1979 Ом Г. Определение закона, по которому металлы проводят контактное электричество. — В кн.: Классики физической науки. М., 1989

Энциклопедия Сто человек. Которые изменили мир. Ом.

Прохоров А. М. Физический энциклопедический словарь, М., 1983

Орир Дж. Физика , т. 2. М., 1981

Джанколи Д. Физика , т. 2. М., 1989

http://www.portal-slovo.ru/

http://www.polarcom.ru/~vvtsv/s_doc9c.html)

Размещено на Allbest.ru

Подобные документы

    История открытия Исааком Ньютоном «Закона всемирного тяготения», события, предшествующие данному открытию. Суть и границы применения закона. Формулировка законов Кеплера и их применение к движению планет, их естественных и искусственных спутников.

    презентация , добавлен 25.07.2010

    Изучение движения тела под действием постоянной силы. Уравнение гармонического осциллятора. Описание колебания математического маятника. Движение планет вокруг Солнца. Решение дифференциального уравнения. Применение закона Кеплера, второго закона Ньютона.

    реферат , добавлен 24.08.2015

    История открытия закона всемирного тяготения. Иоган Кеплер как один из первооткрывателей закона движения планет вокруг солнца. Сущность и особенности эксперимента Кавендиша. Анализ теории силы взаимного притяжения. Основные границы применимости закона.

    презентация , добавлен 29.03.2011

    Изучение «Закона Архимеда», проведение опытов по определению архимедовой силы. Вывод формул для нахождения массы вытесненной жидкости и расчета плотности. Применение «Закона Архимеда» для жидкостей и газов. Методическая разработка урока по данной теме.

    конспект урока , добавлен 27.09.2010

    Биографические сведения о Ньютоне — великом английском физике, математике и астрономе, его труды. Исследования и открытия ученого, эксперименты по оптике и теории цвета. Первый вывод Ньютоном скорости звука в газе, основанный на законе Бойля-Мариотта.

    презентация , добавлен 26.08.2015

    Изучение причины магнитной аномалии. Методы определения горизонтальной составляющей напряженности магнитного поля Земли. Применение закона Био-Савара-Лапласа. Определение причины поворота стрелки после подачи напряжения на катушку тангенс–гальванометра.

    контрольная работа , добавлен 25.06.2015

    Описание основных законов Ньютона. Характеристика первого закона о сохранении телом состояния покоя или равномерного движения при скомпенсированных действиях на него других тел. Принципы закона ускорения тела. Особенности инерционных систем отсчета.

    презентация , добавлен 16.12.2014

    Законы движения планет Кеплера, их краткая характеристика. История открытия Закона всемирного тяготения И. Ньютоном. Попытки создания модели Вселенной. Движение тел под действием силы тяжести. Гравитационные силы притяжения. Искусственные спутники Земли.

    реферат , добавлен 25.07.2010

    Проверка справедливости соотношений при параллельном соединении резисторов и первого закона Кирхгофа. Особенности сопротивления приемников. Методика расчета напряжения и тока для различных соединений. Сущность закона Ома для участка и для всей цепи.

    лабораторная работа , добавлен 12.01.2010

    Фундаментальные взаимодействия в природе. Взаимодействие электрических зарядов. Свойства электрического заряда. Закон сохранения электрического заряда. Формулировка закона Кулона. Векторная форма и физический смысл закона Кулона. Принцип суперпозиции.

Здравствуйте, уважаемые читатели сайта «Заметки электрика»..

Сегодня открываю новый раздел на сайте под названием .

В этом разделе я постараюсь в наглядной и простой форме объяснить Вам вопросы электротехники. Скажу сразу, что далеко углубляться в теоретические знания мы не будем, но вот с основами познакомимся в достаточном порядке.

Первое, с чем я хочу Вас познакомить, это с законом Ома для участка цепи. Это самый основной закон, который должен знать каждый .

Знание этого закона позволит нам беспрепятственно и безошибочно определять значения силы тока, напряжения (разности потенциалов) и сопротивления на участке цепи.

Кто такой Ом? Немного истории

Закон Ома открыл всем известный немецкий физик Георг Симон Ом в 1826 году. Вот так он выглядел.

Всю биографию Георга Ома я рассказывать Вам не буду. Про это Вы можете узнать на других ресурсах более подробно.

Скажу только самое главное.

Его именем назван самый основной закон электротехники, который мы активно применяем в сложных расчетах при проектировании, на производстве и в быту.

Закон Ома для однородного участка цепи выглядит следующим образом:

I – значение тока, идущего через участок цепи (измеряется в амперах)

U – значение напряжения на участке цепи (измеряется в вольтах)

R – значение сопротивления участка цепи (измеряется в Омах)

Если формулу объяснить словами, то получится, что сила тока пропорциональная напряжению и обратно пропорциональна сопротивлению участка цепи.

Проведем эксперимент

Чтобы понять формулу не на словах, а на деле, необходимо собрать следующую схему:

Цель этой статьи — это показать наглядно, как использовать закон Ома для участка цепи. Поэтому я на своем рабочем стенде собрал эту схему. Смотрите ниже как она выглядит.

С помощью ключа управления (избирания) можно выбрать, либо постоянное напряжение, либо переменное напряжение на выходе. В нашем случае используется постоянное напряжения. Уровень напряжения я меняю с помощью лабораторного автотрансформатора (ЛАТР).

В нашем эксперименте я буду использовать напряжение на участке цепи, равное 220 (В). Контроль напряжения на выходе смотрим по вольтметру.

Теперь мы полностью готовы провести самостоятельно эксперимент и проверить закон Ома в действительности.

Ниже я приведу 3 примера. В каждом примере мы будем определять искомую величину 2 методами: с помощью формулы и практическим путем.

Пример № 1

В первом примере нам нужно найти ток (I) в цепи, зная величину источника постоянного напряжения и величину сопротивления светодиодной лампочки.

Напряжение источника постоянного напряжения составляет U = 220 (В) . Сопротивление светодиодной лампочки равно R = 40740 (Ом) .

С помощью формулы найдем ток в цепи:

I = U/R = 220 / 40740 = 0,0054 (А)

Подключаем последовательно светодиодной лампочке , включенный в режиме амперметр, и замеряем ток в цепи.

На дисплее мультиметра показан ток цепи. Его значение равно 5,4 (мА) или 0,0054 (А), что соответствует току, найденному по формуле.

Пример № 2

Во втором примере нам нужно найти напряжение (U) участка цепи, зная величину тока в цепи и величину сопротивления светодиодной лампочки.

I = 0,0054 (А)

R = 40740 (Ом)

С помощью формулы найдем напряжение участка цепи:

U = I*R = 0,0054 *40740 = 219,9 (В) = 220 (В)

А теперь проверим полученный результат практическим путем.

Подключаем параллельно светодиодной лампочке мультиметр, включенный в режиме вольтметр, и замеряем напряжение.

На дисплее мультиметра показана величина измеренного напряжения. Его значение равно 220 (В), что соответствует напряжению, найденному по формуле закона Ома для участка цепи.

Пример № 3

В третьем примере нам нужно найти сопротивление (R) участка цепи, зная величину тока в цепи и величину напряжения участка цепи.

I = 0,0054 (А)

U = 220 (В)

Опять таки, воспользуемся формулой и найдем сопротивление участка цепи:

R = U/ I = 220/0,0054 = 40740,7 (Ом)

А теперь проверим полученный результат практическим путем.

Сопротивление светодиодной лампочки мы измеряем с помощью или мультиметра.

Полученное значение составило R = 40740 (Ом) , что соответствует сопротивлению, найденному по формуле.

Как легко запомнить Закон Ома для участка цепи!!!

Чтобы не путаться и легко запомнить формулу, можно воспользоваться небольшой подсказкой, которую Вы можете сделать самостоятельно.

Нарисуйте треугольник и впишите в него параметры электрической цепи, согласно рисунка ниже. У Вас должно получится вот так.

Как этим пользоваться?

Пользоваться треугольником-подсказкой очень легко и просто. Закрываете своим пальцем, тот параметр цепи, который необходимо найти.

Если оставшиеся на треугольнике параметры расположены на одном уровне, то значит их необходимо перемножить.

Если же оставшиеся на треугольнике параметры расположены на разном уровне, то тогда необходимо разделить верхний параметр на нижний.

С помощью треугольника-подсказки Вы не будете путаться в формуле. Но лучше все таки ее выучить, как таблицу умножения.

Выводы

В завершении статьи сделаю вывод.

Электрический ток — это направленный поток электронов от точки В с потенциалом минус к точке А с потенциалом плюс. И чем выше разность потенциалов между этими точками, тем больше электронов переместится из точки В в точку А, т.е. ток в цепи увеличится, при условии, что сопротивление цепи останется неизменным.

Но сопротивление лампочки противодействует протеканию электрического тока. И чем больше сопротивление в цепи (последовательное соединение нескольких лампочек), тем меньше будет ток в цепи, при неизменном напряжении сети.

P.S. Тут в интернете нашел смешную, но поясняющую карикатуру на тему закона Ома для участка цепи.

Соединенный проводами с различными электроприборами и потребителями электри-ческой энергии, образует электрическую цепь.

Электрическую цепь принято изображать с помощью схем, в которых элементы электрической цепи (сопротивления , источники тока, включатели, лампы, при-боры и т. д.) обозначены специальными значками.

Направление тока в цепи — это направление от положи-тельного полюса источника тока к отрицательному. Это пра-вило было установлено в XIX в. и с тех пор соблюдается. Перемещение реальных зарядов может не совпадать с ус-ловным направлением тока. Так, в металлах носителями тока являются отрицательно заряжен-ные электроны, и движутся они от отрицательного полюса к положительному, т. е. в обратном направлении. В электролитах реальное перемещение зарядов может совпадать или быть противоположным направлению тока, в зависимости от того, какие ионы являются носителями заря-да — положительные или отрицательные.

Включение элементов в электрическую цепь может быть последовательным или параллельным .

Закон Ома для полной цепи.

Рассмотрим электрическую цепь, состоящую из источника тока и ре-зистора R .

Закон Ома для полной цепи устанавливает связь между силой тока в цепи, ЭДС и полным сопротивлением цепи, состоя-щим из внешнего сопротивления R и внутреннего сопротивления источ-ника тока r .

Работа сторонних сил A ст источника тока, согласно определению ЭДС (ɛ ) равна A ст = ɛq , где q — заряд , перемещенный ЭДС. Согласно определе-нию тока q = It , где t — время, в течение которого переносился заряд. Отсюда имеем:

A ст = ɛ It .

Тепло, выделяемое при совершении работы в цепи, согласно закону Джоуля — Ленца , равно:

Q = I 2 Rt + I 2 rt .

Согласно закону сохранения энергии А = Q . Приравнивая (A ст = ɛ It ) и (Q = I 2 Rt + I 2 rt ), получим:

ɛ = IR + Ir.

Закон Ома для замкнутой цепи обычно записывается в виде:

.

Сила тока в полной цепи равна отношению ЭДС цепи к ее полному сопротивлению.

Если цепь содержит несколько последовательно соединенных ис-точников с ЭДС ɛ 1 , ɛ 2 , ɛ 3 и т. д., то полная ЭДС цепи равна алгебраической сумме ЭДС отдельных источников. Знак ЭДС источника определяется по отношению к направлению обхода контура, который выбирается произвольно, например, на рисунке ниже — против часовой стрелки.

Сторонние силы внутри источника совершают при этом по-ложительную работу . И наоборот, для цепи справедливо следующее уравнение:

ɛ = ɛ 1 + ɛ 2 + ɛ 3 = | ɛ 1 | — | ɛ 2 | -| ɛ 3 | .

В соответствии с сила тока положительна при положительной ЭДС — направление тока во внешней цепи совпадает с направлением обхода контура. Полное сопротивление цепи с несколькими источниками равно сумме внешнего и внутренних сопротивлений всех источников ЭДС, например, для рисунка выше:

R n = R + r 1 + r 2 + r 3 .

Если изолированный проводник поместить в электрическое поле \(\overrightarrow{E} \), то на свободные заряды \(q\) в проводнике будет действовать сила \(\overrightarrow{F} = q\overrightarrow{E}\) В результате в проводнике возникает кратковременное перемещение свободных зарядов. Этот процесс закончится тогда, когда собственное электрическое поле зарядов, возникших на поверхности проводника, скомпенсирует полностью внешнее поле. Результирующее электростатическое поле внутри проводника будет равно нулю.

Однако, в проводниках при определенных условиях может возникнуть непрерывное упорядоченное движение свободных носителей электрического заряда.

Направленное движение заряженных частиц называется электрическим током.

За направление электрического тока принято направление движения положительных свободных зарядов. Для существования электрического тока в проводнике необходимо создать в нем электрическое поле.

Количественной мерой электрического тока служит сила тока \(I\) — скалярная физическая величина, равная отношению заряда \(\Delta q\), переносимого через поперечное сечение проводника (рис. 1.8.1) за интервал времени \(\Delta t\), к этому интервалу времени:

$$I = \frac{\Delta q}{\Delta t} $$

Если сила тока и его направление не изменяются со временем, то такой ток называется постоянным .

В Международной системе единиц СИ сила тока измеряется в Амперах (А). Единица измерения тока 1 А устанавливается по магнитному взаимодействию двух параллельных проводников с током.

Постоянный электрический ток может быть создан только в замкнутой цепи , в которой свободные носители заряда циркулируют по замкнутым траекториям. Электрическое поле в разных точках такой цепи неизменно во времени. Следовательно, электрическое поле в цепи постоянного тока имеет характер замороженного электростатического поля. Но при перемещении электрического заряда в электростатическом поле по замкнутой траектории, работа электрических сил равна нулю. Поэтому для существования постоянного тока необходимо наличие в электрической цепи устройства, способного создавать и поддерживать разности потенциалов на участках цепи за счет работы сил неэлектростатического происхождения . Такие устройства называются источниками постоянного тока . Силы неэлектростатического происхождения, действующие на свободные носители заряда со стороны источников тока, называются сторонними силами .

Природа сторонних сил может быть различной. В гальванических элементах или аккумуляторах они возникают в результате электрохимических процессов, в генераторах постоянного тока сторонние силы возникают при движении проводников в магнитном поле. Источник тока в электрической цепи играет ту же роль, что и насос, который необходим для перекачивания жидкости в замкнутой гидравлической системе. Под действием сторонних сил электрические заряды движутся внутри источника тока против сил электростатического поля, благодаря чему в замкнутой цепи может поддерживаться постоянный электрический ток.

При перемещении электрических зарядов по цепи постоянного тока сторонние силы, действующие внутри источников тока, совершают работу.

Физическая величина, равная отношению работы \(A_{ст}\) сторонних сил при перемещении заряда \(q\) от отрицательного полюса источника тока к положительному к величине этого заряда, называется электродвижущей силой источника (ЭДС):

$$ЭДС=\varepsilon=\frac{A_{ст}}{q}. $$

Таким образом, ЭДС определяется работой, совершаемой сторонними силами при перемещении единичного положительного заряда. Электродвижущая сила, как и разность потенциалов, измеряется в Вольтах (В).

При перемещении единичного положительного заряда по замкнутой цепи постоянного тока работа сторонних сил равна сумме ЭДС, действующих в этой цепи, а работа электростатического поля равна нулю.

Цепь постоянного тока можно разбить на отдельные участки. Те участки, на которых не действуют сторонние силы (т. е. участки, не содержащие источников тока), называются однородными . Участки, включающие источники тока, называются неоднородными .

При перемещении единичного положительного заряда по некоторому участку цепи работу совершают как электростатические (кулоновские), так и сторонние силы. Работа электростатических сил равна разности потенциалов \(\Delta \phi_{12} = \phi_{1} — \phi_{2}\) между начальной (1) и конечной (2) точками неоднородного участка. Работа сторонних сил равна по определению электродвижущей силе \(\mathcal{E}\), действующей на данном участке. Поэтому полная работа равна

$$U_{12} = \phi_{1} — \phi_{2} + \mathcal{E}$$

Величину U 12 принято называть напряжением на участке цепи 1-2. В случае однородного участка напряжение равно разности потенциалов:

$$U_{12} = \phi_{1} — \phi_{2}$$

Немецкий физик Г. Ом в 1826 году экспериментально установил, что сила тока \(I\), текущего по однородному металлическому проводнику (т. е. проводнику, в котором не действуют сторонние силы), пропорциональна напряжению \(U\) на концах проводника:

$$I = \frac{1}{R} U; \: U = IR$$

где \(R\) = const.

Величину R принято называть электрическим сопротивлением . Проводник, обладающий электрическим сопротивлением, называется резистором . Данное соотношение выражает закон Ома для однородного участка цепи: сила тока в проводнике прямо пропорциональна приложенному напряжению и обратно пропорциональна сопротивлению проводника.

В СИ единицей электрического сопротивления проводников служит Ом (Ом). Сопротивлением в 1 Ом обладает такой участок цепи, в котором при напряжении 1 В возникает ток силой 1 А.

Проводники, подчиняющиеся закону Ома, называются линейными . Графическая зависимость силы тока \(I\) от напряжения \(U\) (такие графики называются вольт-амперными характеристиками , сокращенно ВАХ) изображается прямой линией, проходящей через начало координат. Следует отметить, что существует много материалов и устройств, не подчиняющихся закону Ома, например, полупроводниковый диод или газоразрядная лампа. Даже у металлических проводников при токах достаточно большой силы наблюдается отклонение от линейного закона Ома, так как электрическое сопротивление металлических проводников растет с ростом температуры.

Для участка цепи, содержащего ЭДС, закон Ома записывается в следующей форме:

$$IR = U_{12} = \phi_{1} — \phi_{2} + \mathcal{E} = \Delta \phi_{12} + \mathcal{E}$$
$$\color{blue}{I = \frac{U}{R}}$$

Это соотношение принято называть обобщенным законом Ома или законом Ома для неоднородного участка цепи .

На рис. 1.8.2 изображена замкнутая цепь постоянного тока. Участок цепи (cd ) является однородным.

Рисунок 1.8.2.

Цепь постоянного тока

По закону Ома

$$IR = \Delta\phi_{cd}$$

Участок (ab ) содержит источник тока с ЭДС, равной \(\mathcal{E}\).

По закону Ома для неоднородного участка,

$$Ir = \Delta \phi_{ab} + \mathcal{E}$$

Сложив оба равенства, получим:

$$I(R+r) = \Delta\phi_{cd} + \Delta \phi_{ab} + \mathcal{E}$$

Но \(\Delta\phi_{cd} = \Delta \phi_{ba} = -\Delta \phi_{ab}\).

$$\color{blue}{I=\frac{\mathcal{E}}{R + r}}$$

Эта формула выражает закон Ома для полной цепи : сила тока в полной цепи равна электродвижущей силе источника, деленной на сумму сопротивлений однородного и неоднородного участков цепи (внутреннего сопротивления источника).

Сопротивление r неоднородного участка на рис. 1.8.2 можно рассматривать как внутреннее сопротивление источника тока . В этом случае участок (ab ) на рис. 1.8.2 является внутренним участком источника. Если точки a и b замкнуть проводником, сопротивление которого мало по сравнению с внутренним сопротивлением источника (\(R\ \ll r\)), тогда в цепи потечет ток короткого замыкания

$$I_{кз}=\frac{\mathcal{E}}{r}$$

Сила тока короткого замыкания — максимальная сила тока, которую можно получить от данного источника с электродвижущей силой \(\mathcal{E}\) и внутренним сопротивлением \(r\). У источников с малым внутренним сопротивлением ток короткого замыкания может быть очень велик и вызывать разрушение электрической цепи или источника. Например, у свинцовых аккумуляторов, используемых в автомобилях, сила тока короткого замыкания может составлять несколько сотен ампер. Особенно опасны короткие замыкания в осветительных сетях, питаемых от подстанций (тысячи ампер). Чтобы избежать разрушительного действия таких больших токов, в цепь включаются предохранители или специальные автоматы защиты сетей.

В ряде случаев для предотвращения опасных значений силы тока короткого замыкания к источнику последовательно подсоединяется некоторое внешнее сопротивление. Тогда сопротивление r равно сумме внутреннего сопротивления источника и внешнего сопротивления, и при коротком замыкании сила тока не окажется чрезмерно большой.

Если внешняя цепь разомкнута, то \(\Delta \phi_{ba} = -\Delta \phi_{ab} = \mathcal{E}\), т. е. разность потенциалов на полюсах разомкнутой батареи равна ее ЭДС.

Если внешнее нагрузочное сопротивление R включено и через батарею протекает ток I , разность потенциалов на ее полюсах становится равной

$$\Delta \phi_{ba} = \mathcal{E} — Ir$$

На рис. 1.8.3 дано схематическое изображение источника постоянного тока с ЭДС равной \(\mathcal{E}\) и внутренним сопротивлением r в трех режимах: «холостой ход», работа на нагрузку и режим короткого замыкания (к. з.). Указаны напряженность \(\overrightarrow{E}\) электрического поля внутри батареи и силы, действующие на положительные заряды:\(\overrightarrow{F}_{э}\) — электрическая сила и \(\overrightarrow{F}_{ст}\) — сторонняя сила. В режиме короткого замыкания электрическое поле внутри батареи исчезает.

Для измерения напряжений и токов в электрических цепях постоянного тока используются специальные приборы — вольтметры и амперметры .

Вольтметр предназначен для измерения разности потенциалов, приложенной к его клеммам. Он подключается параллельно участку цепи, на котором производится измерение разности потенциалов. Любой вольтметр обладает некоторым внутренним сопротивлением \(R_{В}\). Для того, чтобы вольтметр не вносил заметного перераспределения токов при подключении к измеряемой цепи, его внутреннее сопротивление должно быть велико по сравнению с сопротивлением того участка цепи, к которому он подключен. Для цепи, изображенной на рис. 1.8.4, это условие записывается в виде:

$$R_{В} \gg R_{1}$$

Это условие означает, что ток \(I_{В} = \Delta \phi_{cd} / R_{В}\), протекающий через вольтметр, много меньше тока \(I = \Delta \phi_{cd} / R_{1}\), который протекает по тестируемому участку цепи.

Поскольку внутри вольтметра не действуют сторонние силы, разность потенциалов на его клеммах совпадает по определению с напряжением. Поэтому можно говорить, что вольтметр измеряет напряжение.

Амперметр предназначен для измерения силы тока в цепи. Амперметр включается последовательно в разрыв электрической цепи, чтобы через него проходил весь измеряемый ток. Амперметр также обладает некоторым внутренним сопротивлением \(R_{А}\). В отличие от вольтметра, внутреннее сопротивление амперметра должно быть достаточно малым по сравнению с полным сопротивлением всей цепи. Для цепи на рис. 1.8.4 сопротивление амперметра должно удовлетворять условию

$$R_{А} \ll (r + R_{1} + R{2})$$

чтобы при включении амперметра ток в цепи не изменялся.

Измерительные приборы — вольтметры и амперметры — бывают двух видов: стрелочные (аналоговые) и цифровые. Цифровые электроизмерительные приборы представляют собой сложные электронные устройства. Обычно цифровые приборы обеспечивают более высокую точность измерений.

Закон Ома простыми словами. Лекция № 6

Закон Ома для участка цепи

Со школьного курса физики всем хорошо известна классическая трактовка Закона Ома:

Сила тока в проводнике прямо пропорциональна напряжению на концах проводника и обратно пропорциональна его сопротивлению.

I = U/R

Это значит, если к концам проводника сопротивлением R = 1 Ом приложено напряжение U = 1 Вольт, тогда величина тока I в проводнике будет равна 1/1 = 1 Ампер.

Отсюда следуют ещё два полезных соотношения:

Если в проводнике, сопротивлением 1 Ом, протекает ток 1 Ампер, значит на концах проводника напряжение 1 Вольт (падение напряжения).

U = IR

Если на концах проводника есть напряжение 1 Вольт и по нему протекает ток 1 Ампер, значит сопротивление проводника равно 1 Ом.

R = U/I

Вышеописанные формулы в таком виде могут быть применимы для переменного тока лишь в том случае, если цепь состоит только из активного сопротивления R.
Кроме того, следует помнить, что Закон Ома справедлив только для линейных элементов цепи.

Предлагается простой Онлайн-калькулятор для практических расчётов.
Закон Ома. Расчёт напряжения, сопротивления, тока, мощности.
После сброса ввести два любых известных параметра.

I=U/R;   U=IR;   R=U/I;
P=UI   P=U²/R;   P=I²R;
R=U²/P;   R=P/I²   U=√(PR)   I= √(P/R)

Как звучит закон Ома для участка цепи

Есть говорить об официальной формулировке, то закон Ома можно озвучить так:

Сила тока имеет прямую зависимость от напряжения и обратную от сопротивления. Это высказывание справедливо для участка цепи с каким-то определенным и стабильным сопротивлением.

Формула этой зависимости на рисунке. Тут I — это сила тока, U — напряжение, R — сопротивление.

Формула закона Ома

  • Чем больше напряжение, тем больше ток.
  • Чем больше сопротивление, тем ток меньше.

Не так легко представить себе смысл этого выражения. Ведь электричество нельзя увидеть. Мы только приблизительно знаем что это такое. Попытаемся уяснить себе смысл этого закона при помощи аналогий.

Закон Ома для замкнутой цепи

Если к источнику питания подключить внешнюю цепь сопротивлением R, в цепи пойдёт ток с учётом внутреннего сопротивления источника:

I — Сила тока в цепи.

— Электродвижущая сила (ЭДС) — величина напряжения источника питания не зависящая от внешней цепи (без нагрузки). Характеризуется потенциальной энергией источника.
r — Внутреннее сопротивление источника питания.

Для электродвижущей силы внешнеее сопротивление R и внутреннее r соединены последовательно, значит величина тока в цепи определится значением ЭДС и суммой сопротивлений: I =
/(R+r) .

Напряжение на выводах внешней цепи определится исходя из силы тока и сопротивления R соотношением, которое уже рассматривалось выше: U = IR.
Напряжение U, при подключении нагрузки R, всегда будет меньше чем ЭДС на величину произведения I*r, которую называют падением напряжения на внутреннем сопротивлении источника питания.
С этим явлением мы сталкиваемся достаточно часто, когда видим в работе частично разряженные батарейки или аккумуляторы.
По мере разряда, увеличивается их внутреннее сопротивление, следовательно, увеличивается падение напряжение внутри источника, значит уменьшается внешнее напряжение U =
— I*r.
Чем меньше ток и внутреннее сопротивление источника, тем ближе по значению его ЭДС и напряжение на его выводах U.
Если ток в цепи равен нулю, следовательно,
= U. Цепь разомкнута, ЭДС источника равна напряжению на его выводах.

В случаях, когда внутренним сопротивлением источника можно пренебречь (r ≈ 0), напряжение на выводах источника будет равно ЭДС (
≈ U ) независимо от сопротивления внешней цепи R.
Такой источник питания называют источником напряжения.

Закон Ома для переменного тока

При наличии индуктивности или ёмкости в цепи переменного тока необходимо учитывать их реактивное сопротивление.
В таком случае запись Закона Ома будет иметь вид:

I = U/Z

Здесь Z — полное (комплексное) сопротивление цепи — импеданс. В него входит активная R и реактивная X составляющие.
Реактивное сопротивление зависит от номиналов реактивных элементов, от частоты и формы тока в цепи.
Более подробно ознакомится с комплексным сопротивлением можно на страничке импеданс.

С учётом сдвига фаз φ, созданного реактивными элементами, для синусоидального переменного тока обычно записывают Закон Ома в комплексной форме:
— комплексная амплитуда тока.
= Iampe jφ
— комплексная амплитуда напряжения.
= Uampe jφ
— комплексное сопротивление. Импеданс.
φ — угол сдвига фаз между током и напряжением.
e — константа, основание натурального логарифма.
j — мнимая единица.
Iamp , Uamp — амплитудные значения синусоидального тока и напряжения.

Для ЭДС

Перед тем как рассмотреть закон Ома для полной (замкнутой) цепи приведу правило знаков для ЭДС, которое гласит:
Если внутри источника ЭДС ток идет от катода (-) к аноду (+) (направление напряженности поля сторонних сил совпадает с направлением тока в цепи, то ЭДС такого источника считается положительной. В противном случае – ЭДС считается отрицательной.

Практическим применением этого правила является возможность приведения нескольких источников ЭДС в цепи к одному с величиной E=E1+E2+…+En, естественно, с учетом знаков, определяемых по вышеприведенному правилу. Например (рис.3.3) E=E1+E2-E3. При отсутствии встречно включенного источника E3 (на практике так почти никогда не бывает) имеем широко распространенное последовательное включение элементов питания, при котором их напряжения суммируются.

Для полной цепи

Закон Ома для полной цепи – его еще можно назвать закон ома для замкнутой цепи, имеет вид I=E/(R+r). Приведенная формула закона Ома содержит обозначение r, которое еще не упоминалось. Это внутреннее сопротивление источника ЭДС. Оно достаточно мало, в большинстве случаев при практических расчетах им можно пренебречь (при условии, что R>>r – сопротивление цепи много больше внутреннего сопротивления источника). Однако, когда они соизмеримы, пренебрегать величиной r нельзя.

Как вариант можно рассмотреть случай, при котором R=0 (короткое замыкание). Тогда приведенная формула закона Ома для полной цепи примет вид: I=E/r, то есть величина внутреннего сопротивления будет определять ток короткого замыкания. Такая ситуация вполне может быть реальной. Закон Ома рассмотрен здесь достаточно бегло, но приведенных формул достаточно для проведения большинства расчетов, примеры которых, по мере размещения других материалов я буду приводить.

Полноценную цепь составляет уже участок (участки), а также источник ЭДС. То есть, фактически к существующему резистивному компоненту участка цепи добавляется внутреннее сопротивление источника ЭДС. Поэтому логичным является некоторое изменение выше рассмотренной формулы:

I = U / (R + r)

Конечно, значение внутреннего сопротивления ЭДС в законе Ома для полной электрической цепи можно считать ничтожно малым, правда во многом это значение сопротивления зависит от структуры источника ЭДС. Тем не менее, при расчетах сложных электронных схем, электрических цепей с множеством проводников, наличие дополнительного сопротивления является важным фактором.

Как для участка цепи, так и для полной схемы следует учитывать естественный момент – использование тока постоянной или переменной величины. Если отмеченные выше моменты, характерные для закона Ома, рассматривались с точки зрения использования постоянного тока, соответственно с переменным током всё выглядит несколько иначе.

Нелинейные элементы и цепи

Закон Ома не является фундаментальным законом природы и может быть применим в ограниченных случаях, например, для большинства проводников.
Его невозможно использовать для расчёта напряжения и тока в полупроводниковых или электровакуумных приборах, где эта зависимость не является пропорциональной и её можно определять только с помощью вольтамперной характеристики (ВАХ). К данной категории элементов относятся все полупроводниковые приборы (диоды, транзисторы, стабилитроны, тиристоры, варикапы и т.д.) и электронные лампы.
Такие элементы и цепи, в которых они используются, называют нелинейными.

Напряжение, ток и сопротивление

Электрическая цепь образуется, когда создается проводящий путь, позволяющий электрическому заряду непрерывно перемещаться. Это непрерывное движение электрического заряда по проводникам цепи называется током, и о нем часто говорят как о «потоке», как о потоке жидкости через полую трубу.

Сила, побуждающая носители заряда «течь» по цепи, называется напряжением. Напряжение – это особая мера потенциальной энергии, которая всегда относительна между двумя точками. Когда мы говорим об определенной величине напряжения, присутствующего в цепи, мы имеем в виду измерение потенциальной энергии для перемещения носителей заряда из одной конкретной точки этой цепи в другую конкретную точку. Без упоминания двух конкретных точек термин «напряжение» не имеет значения.

Ток, как правило, проходит через проводники с некоторой степенью трения или противодействия движению. Это противодействие движению правильнее называть сопротивлением. Величина тока в цепи зависит от величины напряжения и величины сопротивления в цепи, препятствующего прохождению тока. Как и напряжение, сопротивление – это величина, измеряемая между двумя точками. По этой причине величины напряжения и сопротивления часто указываются как «между» двумя точками в цепи.

Единицы измерения: вольт, ампер и ом

Чтобы иметь возможность делать осмысленные утверждения об этих величинах в цепях, нам нужно уметь описывать их количества так же, как мы могли бы количественно определить массу, температуру, объем, длину или любые другие физические величины. Для массы мы можем использовать единицы «килограмм» или «грамм». Для температуры мы можем использовать градусы Фаренгейта или градусы Цельсия. В таблице ниже приведены стандартные единицы измерения электрического тока, напряжения и сопротивления:

Единицы измерения тока, напряжения, сопротивленияВеличинаСимволЕдиница измеренияСокращение единицы измерения
ТокIАмперА
НапряжениеVВольтВ
СопротивлениеRОмОм

«Символ», присвоенный каждой величине, представляет собой стандартную букву латинского алфавита, используемую для представления этой величины в формулах. Подобные стандартизированные буквы распространены во всех физических и технических дисциплинах и признаны во всем мире. «Сокращение единицы измерения» для каждой величины представляет собой алфавитный символ(ы), используемый в качестве сокращенного обозначения конкретной единицы измерения.

Каждая единица измерения названа в честь известного экспериментатора в области электричества: ампер в честь француза Андре М. Ампера, вольт в честь итальянца Алессандро Вольта, а ом в честь немца Георга Симона Ома.

Математический символ для каждой величины также имеет значение. «R» для сопротивления и «V» для напряжения говорят сами за себя («Resistance» и «Voltage», соответственно), тогда как «I» для тока кажется немного странным. Предполагается, что буква «I» должна представлять «интенсивность» («Intensity»)(потока заряда). Судя по исследованиям, которые мне удалось провести, кажется, что есть некоторые разногласия по поводу значения слова «I». Другой символ напряжения, «E», означает «электродвижущую силу» («Electromotive force»). Символы «E» и «V» по большей части взаимозаменяемы, хотя в некоторых текстах «E» зарезервировано для обозначения напряжения на источнике (таком как батарея или генератор), а «V»– для обозначения напряжения на любом другом элементе.

Все эти символы выражаются заглавными буквами, за исключением случаев, когда величина (особенно напряжение или ток) описывается в терминах короткого периода времени (так называемые «мгновенные» значения). Например, напряжение батареи, которое стабильно в течение длительного периода времени, будет обозначаться заглавной буквой «E», тогда как пиковое напряжения при ударе молнии в тот самый момент, когда она попадает в линию электропередачи, скорее всего, будет обозначаться строчной буквой «е» (или строчной буквой «v»), чтобы отметить это значение как имеющееся в один момент времени. Это же соглашение о нижнем регистре справедливо и для тока: строчная буква «i» представляет ток в некоторый момент времени. Однако большинство измерений в цепях постоянного тока, которые стабильны во времени, будут обозначаться заглавными буквами.

Кулон и электрический заряд

Одна из основных единиц электрических измерений, которую часто преподают в начале курсов электроники, но нечасто используют впоследствии, – это кулон – единица измерения электрического заряда, пропорциональная количеству электронов в несбалансированном состоянии. Один кулон заряда соответствует 6 250 000 000 000 000 000 электронов. Символом количества электрического заряда является заглавная буква «Q», а единица измерения кулонов обозначается «Кл». Единица измерения тока, ампер, равна 1 кулону заряда, проходящему через заданную точку в цепи за 1 секунду. В этом смысле, ток – это скорость движения электрического заряда через проводник.

Как указывалось ранее, напряжение – это мера потенциальной энергии на единицу заряда, доступная для стимулирования протекания тока из одной точки в другую. Прежде чем мы сможем точно определить, что такое «вольт», мы должны понять, как измерить эту величину, которую мы называем «потенциальной энергией». Общей метрической единицей измерения энергии любого вида является джоуль, равный количеству работы, совершаемой силой в 1 ньютон при движении на 1 метр (в том же направлении). В этих научных терминах 1 вольт равен 1 джоулю электрической потенциальной энергии на (деленному на) 1 кулон заряда. Таким образом, 9-вольтовая батарея выделяет 9 джоулей энергии на каждый кулон заряда, проходящего через цепь.

Эти единицы и символы электрических величин станут очень важны, когда мы начнем исследовать отношения между ними в цепях.

Формула Закона Ома

В 1827 году Георг Симон Ом открыл закон силы электрического тока. Его именем назвали Закон и единицу измерения величины сопротивления. Смысл закона в следующем.

Чем толще труба и больше давление воды в водопроводе (с увеличением диаметра трубы уменьшается сопротивление воде) – тем больше потечет воды. Если представить, что вода это электроны (электрический ток), то, чем толще провод и больше напряжение (с увеличением сечения провода уменьшается сопротивление току) – тем больший ток будет протекать по участку цепи.

Сила тока, протекающая по электрической цепи, прямо пропорциональна приложенному напряжению и обратно пропорциональна величине сопротивления цепи.

гдеI – сила тока, измеряется в амперах и обозначается буквой А;U – напряжение, измеряется в вольтах и обозначается буквой В;R – сопротивление, измеряется в омах и обозначается Oм.

Если известны напряжение питания U и сопротивление электроприбора R, то с помощью вышеприведенной формулы, воспользовавшись онлайн калькулятором, легко определить силу протекающего по цепи тока I.

С помощью закона Ома рассчитываются электрические параметры электропроводки, нагревательных элементов, всех радиоэлементов современной электронной аппаратуры, будь то компьютер, телевизор или сотовый телефон.

Анализ простых схем с помощью закона Ома

Давайте посмотрим, как эти формулы работают, чтобы помочь нам анализировать простые схемы:


Рисунок 1 – Пример простой схемы

В приведенной выше схеме есть только один источник напряжения (батарея слева) и только один источник сопротивления току (лампа справа). Это позволяет очень легко применить закон Ома. Если мы знаем значения любых двух из трех величин (напряжения, тока и сопротивления) в этой цепи, мы можем использовать закон Ома для определения третьей.

В этом первом примере мы вычислим величину тока (I) в цепи, учитывая значения напряжения (E) и сопротивления (R):


Рисунок 2 – Пример 1. Известны напряжение источника и сопротивление лампы

Какая величина тока (I) в этой цепи?

[I = frac{E}{R} = frac{12 В}{3 Ом} = 4 А]

Во втором примере мы вычислим величину сопротивления (R) в цепи, учитывая значения напряжения (E) и тока (I):


Рисунок 3 – Пример 2. Известны напряжение источника и ток в цепи

Какое сопротивление (R) оказывает лампа?

[R = frac{E}{I} = frac{36 В}{4 А} = 9 Ом]

В последнем примере мы рассчитаем величину напряжения, подаваемого батареей, с учетом значений тока (I) и сопротивления (R):


Рисунок 4 – Пример 3. Известны ток в цепи и сопротивление лампы

Какое напряжение обеспечивает батарея?

[E = IR = (2 А)(7 Ом) = 14 В]

Метода треугольника закона Ома

Закон Ома – очень простой и полезный инструмент для анализа электрических цепей. Он так часто используется при изучении электричества и электроники, что студент должен запомнить его. Если вы не очень хорошо умеете работать с формулами, то для его запоминания существует простой прием, помогающий использовать его для любой величины, зная две других. Сначала расположите буквы E, I и R в виде треугольника следующим образом:


Рисунок 5 – Треугольник закона Ома

Если вы знаете E и I и хотите определить R, просто удалите R с картинки и посмотрите, что осталось:


Рисунок 6 – Закон Ома для определения R

Если вы знаете E и R и хотите определить I, удалите I и посмотрите, что осталось:


Рисунок 7 – Закон Ома для определения I

Наконец, если вы знаете I и R и хотите определить E, удалите E и посмотрите, что осталось:


Рисунок 8 – Закон Ома для определения E

В конце концов, вам придется научиться работать с формуми, чтобы серьезно изучать электричество и электронику, но этот совет может облегчить запоминание ваших первых вычислений. Если вам удобно работать с формулами, всё, что вам нужно сделать, это зафиксировать в памяти E = IR и вывести из нее две другие формулы, когда они вам понадобятся!

Формула Закона Джоуля-Ленца

Величину резистора для изготовления блока нагрузки для блока питания компьютера мы рассчитали, но нужно еще определить какой резистор должен быть мощности? Тут поможет другой закон физики, который, независимо друг от друга открыли одновременно два ученых физика. В 1841 году Джеймс Джоуль, а в 1842 году Эмиль Ленц. Этот закон и назвали в их честь – Закон Джоуля-Ленца.

Потребляемая нагрузкой мощность прямо пропорциональна приложенной величине напряжения и протекающей силе тока. Другими словами, при изменении величины напряжения и тока будет пропорционально будет изменяться и потребляемая мощность.


гдеP – мощность, измеряется в ваттах и обозначается Вт;U – напряжение, измеряется в вольтах и обозначается буквой В;I – сила ток, измеряется в амперах и обозначается буквой А.

Зная напряжения питания и силу тока, потребляемую электроприбором, можно по формуле определить, какую он потребляет мощность. Достаточно ввести данные в окошки ниже приведенного онлайн калькулятора.

ЗаконДжоуля-Ленца позволяет также узнать силу тока, потребляемую электроприбором зная его мощность и напряжение питания. Величина потребляемого тока необходима, например, для выбора сечения провода при прокладке электропроводки или для расчета номинала.

Например, рассчитаем потребляемый ток стиральной машины. По паспорту потребляемая мощность составляет 2200 Вт, напряжение в бытовой электросети составляет 220 В. Подставляем данные в окошки калькулятора, получаем, что стиральная машина потребляет ток величиной 10 А.

Еще один пример, Вы решили в автомобиле установить дополнительную фару или усилитель звука. Зная потребляемую мощность устанавливаемого электроприбора легко рассчитать потребляемый ток и правильно подобрать сечение провода для подключения к электропроводке автомобиля. Допустим, дополнительная фара потребляет мощность 100 Вт (мощность установленной в фару лампочки), бортовое напряжение сети автомобиля 12 В. Подставляем значения мощности и напряжения в окошки калькулятора, получаем, что величина потребляемого тока составит 8,33 А.

Разобравшись всего в двух простейших формулах, Вы легко сможете рассчитать текущие по проводам токи, потребляемую мощность любых электроприборов – практически начнете разбираться в основах электротехники.

Преобразованные формулы Закона Ома и Джоуля-Ленца

Встретил в Интернете картинку в виде круглой таблички, в которой удачно размещены формулы Закона Ома и Джоуля-Ленца и варианты математического преобразования формул. Табличка представляет собой не связанные между собой четыре сектора и очень удобна для практического применения

По таблице легко выбрать формулу для расчета требуемого параметра электрической цепи по двум другим известным. Например, нужно определить ток потребления изделием по известной мощности и напряжению питающей сети. По таблице в секторе тока видим, что для расчета подойдет формула I=P/U.

А если понадобится определить напряжение питающей сети U по величине потребляемой мощности P и величине тока I, то можно воспользоваться формулой левого нижнего сектора, подойдет формула U=P/I.

Подставляемые в формулы величины должны быть выражены в амперах, вольтах, ваттах или Омах.

Применение закона Ома на практике

На практике часто приходится определять не силу тока I, а величину сопротивления R. Преобразовав формулу Закона Ома, можно рассчитать величину сопротивления R, зная протекающий ток I и величину напряжения U.

Величину сопротивления может понадобится рассчитать, например, при изготовлении блока нагрузок для проверки блока питания компьютера. На корпусе блока питания компьютера обычно есть табличка, в которой приведен максимальный ток нагрузки по каждому напряжению. Достаточно в поля калькулятора ввести данные величины напряжения и максимальный ток нагрузки и в результате вычисления получим величину сопротивления нагрузки для данного напряжения. Например, для напряжения +5 В при максимальной величине тока 20 А, сопротивление нагрузки составит 0,25 Ом.

Значение Закона Ома

     Закон Ома определяет силу тока в электрической цепи при заданном напряжении и известном сопротивлении. Он позволяет рассчитать тепловые, химические и магнитные действия тока, так как они зависят от силы тока.

   Закон Ома является чрезвычайно полезным в технике(электронной/электрической), поскольку он касается трех основных электрических величин: тока, напряжения и сопротивления. Он показывает, как эти три величины являются взаимозависимыми на макроскопическом уровне.

    Если бы было можно охарактеризовать закон Ома простыми словами, то наглядно это выглядело бы так:

       Из закона Ома вытекает, что замыкать обычную осветительную сеть проводником малого сопротивления опасно. Сила тока окажется настолько большой, что это может иметь тяжелые последствия.

Задача 1.1

  Рассчитать силу тока, проходящую по медному проводу длиной 100 м, площадью поперечного сечения 0,5 мм2, если к концам провода приложено напряжение 12 B.

  Задачка простая, заключается в нахождении сопротивления медной проволоки с последующим расчетом силы тока по формуле закона Ома для участка цепи. Приступим.

Параллельное и последовательное соединение

В электрике элементы соединяются либо последовательно — один за другим, либо параллельно — это когда к одной точке подключены несколько входов, к другой — выходы от тех же элементов.

Закон Ома для параллельного и последовательного соединения

Последовательное соединение

Как работает закон Ома для этих случаев? При последовательном соединении сила тока, протекающая через цепочку элементов, будет одинаковой. Напряжение участка цепи с последовательно подключенными элементами считается как сумма напряжений на каждом участке. Как можно это объяснить? Протекание тока через элемент — это перенос части заряда с одной его части в другую. То есть, это определенная работа. Величина этой работы и есть напряжение. Это физический смысл напряжения. Если с этим понятно, двигаемся дальше.

Последовательное соединение и параметры этого участка цепи

При последовательном соединении приходится переносить заряд по очереди через каждый элемент. И на каждом элементе это определенный «объем» работы. А чтобы найти объем работы на всем участке цепи, надо работу на каждом элементе сложить. Вот и получается, что общее напряжение — это сумма напряжений на каждом из элементов.

Точно так же — при помощи сложения — находится и общее сопротивление участка цепи. Как можно это себе представить? Ток, протекая по цепочке элементов, последовательно преодолевает все сопротивления. Одно за другим. То есть чтобы найти сопротивление, которое он преодолел, надо сопротивления сложить. Примерно так. Математический вывод более сложен, а так понять механизм действия этого закона проще.

Параллельное соединение

Параллельное соединение — это когда начала проводников/элементов сходятся в одной точке, а в другой — соединены их концы. Постараемся объяснить законы, которые справедливы для соединений этого типа. Начнем с тока. Ток какой-то величины подается в точку соединения элементов. Он разделяется, протекая по всем проводникам. Отсюда делаем вывод, что общий ток на участке равен сумме тока на каждом из элементов: I = I1 + I2 + I3.

Теперь относительно напряжения. Если напряжение — это работа по перемещению заряда, тоо работа, которая необходима на перемещение одного заряда будет одинакова на любом элементе. То есть, напряжение на каждом параллельно подключенном элементе будет одинаковым. U = U1=U2=U3. Не так весело и наглядно, как в случае с объяснением закона Ома для участка цепи, но понять можно.

Законы для параллельного соединения

Для сопротивления все несколько сложнее. Давайте введем понятие проводимости. Это характеристика, которая показывает насколько легко или сложно заряду проходить по этому проводнику. Понятно, что чем меньше сопротивление, тем проще току будет проходить. Поэтому проводимость — G — вычисляется как величина обратная сопротивлению. В формуле это выглядит так: G = 1/R.

Для чего мы говорили о проводимости? Потому что общая проводимость участка с параллельным соединением элементов равна сумме проводимости для каждого из участков. G = G1 + G2 + G3 — понять несложно. Насколько легко току будет преодолеть этот узел из параллельных элементов, зависит от проводимости каждого из элементов. Вот и получается, что их надо складывать.

Теперь можем перейти к сопротивлению. Так как проводимость — обратная к сопротивлению величина, можем получить следующую формулу: 1/R = 1/R1 + 1/R2 + 1/R3.

Что нам дает параллельное и последовательное соединение?

Теоретические знания — это хорошо, но как их применить на практике? Параллельно и последовательно могут соединяться элементы любого типа. Но мы рассматривали только простейшие формулы, описывающие линейные элементы. Линейные элементы — это сопротивления, которые еще называют «резисторы». Итак, вот как можно использовать полученные знания:

В общем, это наиболее распространенные варианты использования этих соединений.

Интегральная и дифференциальная формы закона

Все вышеизложенные моменты с расчетами применимы к условиям, когда в составе электрических схем используются проводники, так сказать, «однородной» структуры. Между тем на практике нередко приходится сталкиваться с построением схематики, где на различных участках структура проводников меняется. К примеру, используются провода большего сечения или, напротив, меньшего, сделанные на основе разных материалов.
Для учёта таких различий существует вариация, так называемого, «дифференциально-интегрального закона Ома». Для бесконечно малого проводника рассчитывается уровень плотности тока в зависимости от напряженности и величины удельной проводимости.

Под дифференциальный расчет берется формула: J = ό * E. Для интегрального расчета, соответственно, формулировка: I * R = φ1 – φ2 + έ Однако эти примеры скорее уже ближе к школе высшей математики и в реальной практике простого электрика фактически не применяются.

Источники

  • https://tel-spb.ru/ohm/
  • https://elektroznatok.ru/info/teoriya/zakon-oma
  • https://ElectroInfo.net/teorija/vse-o-zakone-oma-prostymi-slovami-s-primerami-dlja-chajnikov.html
  • https://radioprog.ru/post/920
  • https://YDoma.info/ehlektrotekhnika/electricity-zakon-oma.html
  • https://toe.1c-umi.ru/lekcii/lekciya_6_-_zakon_oma/

[свернуть]

Закон

Ома: теория, схема и применение

Мы слышим о законе Ома из наших базовых классов. Кроме того, мы ежедневно видим его применение в нашей жизни. В этой статье мы постараемся развеять все сомнения, которые могут возникнуть по этому поводу, с более практическим подходом.

В 1825 году немецкий физик Георг Симон Ом начал работать над сопротивлением. Два года спустя, в 1827 году, он представил свои результаты и опубликовал их в виде книги «Гальваническая цепь, исследованная математически».Наконец, в 1850 году работа Ома была признана доказанной и принята как закон Ома.

Этот закон говорит о соотношении между напряжением, током и сопротивлением. Основное определение закона Ома может быть дано как:

«При постоянных физических параметрах, таких как давление, температура и т. Д., Ток, протекающий по проводнику, прямо пропорционален напряжению на проводнике».

Основы напряжения, тока и сопротивления

Прежде чем углубиться в понятие закона Ома, давайте изучим основные термины, используемые в нем.

Электрическое напряжение

Напряжение в двух точках можно определить как работу, выполняемую при переносе заряда из одной точки в другую. Разность электрических потенциалов, электрическое напряжение, электрическое давление одинаковы и используются только для обозначения электрического напряжения.

Математически,

В = dw / dq

Итак, видя это, какой, по вашему мнению, должна быть единица измерения напряжения? Общая попытка !!

Да, вы правы! это должно быть джоули (для работы) на кулон (для заряда).И эти джоули на кулон обычно называют вольтами. Следовательно, единицей СИ для напряжения является вольт.

Что ж, в повседневной жизни вы много раз встречаете слово «напряжение». Не правда ли! Попробуйте вспомнить (см. Рис. Выше) !! Кроме того, вы знаете, какое напряжение мы получаем в нашем доме? Прокомментируйте, если знаете.

Вы когда-нибудь замечали, что мы развлекаемся с его приложениями, не зная, что это на самом деле означает.

Ха-ха !! Интересно, правда?

Электрический ток

Это еще один термин, с которым мы сталкиваемся ежедневно.Его можно определить как поток электронов в проводнике. Чтобы электроны текли, нам нужна энергия (в виде напряжения).

На практике вы можете представить поток воды (или любой жидкости) по трубе. Кроме того, вы наверняка видели электростанцию. Вы можете понять электростанцию ​​как электростанцию ​​тока.

Математически мы можем выразить ток как,

i = dq / dt

т.е. поток электрического заряда в единицу времени. Так какой же здесь должна быть единица СИ?

Да, это кулон в секунду, который также называют ампером (A).

Позвольте задать вам вопрос,

Знаете ли вы диапазон напряжения и тока мобильных зарядных устройств? Если да, то вы знаете, что делать, верно !! Ха-ха, да расскажи мне в комментариях.

Электрическое сопротивление

Сопротивление — это помеха / препятствие для потока электронов. Протекание тока зависит как от напряжения, так и от сопротивления. Очевидно, что чем больше напряжение, тем больше должен быть ток, но наличие сопротивления снижает ток.

Математически,

R = ρ (L / A), где ρ = удельное электрическое сопротивление (ом-метр), L = длина (метр), A = площадь проводника (квадратные метры), поэтому единицей СИ становится ом (Ом).

На практике представьте, что если в полой трубе застрянут камни или грязь, то это повлияет на поток воды через нее?

Да, будет. Они называются сопротивлениями на пути прохождения потока. Еще одна вещь, которую следует отметить, это то, что с увеличением сопротивления нагрев в системе также будет увеличиваться.

Закон Ома в цепях постоянного тока

Как обсуждалось выше, при постоянных физических параметрах ток, протекающий в проводнике, прямо пропорционален напряжению на нем.Следовательно, мы можем сказать, что сила тока и напряжение имеют линейную зависимость. То есть увеличение одного параметра увеличит другой и наоборот.

Уравнение закона Ома

Математически формулировку закона Ома можно записать как:

В ∝ I

Теперь эта пропорциональность заменена константой, называемой сопротивлением (R),

, следовательно, V = R * I

где, V = напряжение на проводе (в вольтах)

R = Сопротивление проводника (в Ом)

I = Ток, протекающий в проводнике (в амперах)

Из уравнения ясно видно линейную зависимость между напряжением и током.В своей лаборатории вы, должно быть, проводили эксперимент по закону Ома, не так ли?

Что вы заметили? Линейный график или нелинейный?

Конечно, линейный правильный!

График между током и напряжением

Наклон приведенного выше графика дает значение сопротивления в каждой точке.

Треугольник закона Ома

Это просто способ представить и вычислить три параметра, используемых в законе Ома. Напряжение, ток и сопротивление в одном треугольнике обычно называют техникой треугольника закона Ома.

С их помощью вы можете легко узнать стоимость одного, если указаны другие два. Не правда ли?

Например, когда указаны значения тока и сопротивления, формула остается как

В = I * R (в вольтах)

Если даны сопротивление и напряжение, то формулой можно манипулировать как

I = V / R (в амперах)

А если даны значения как напряжения, так и тока, и нам нужно узнать сопротивление, формулу можно изменить как

R = V / R (в омах)

Примеры закона Ома в цепях постоянного тока

Давайте узнаем все это на примере.

Пример 1

Итак, нам нужно вычислить здесь три вещи:

  • Полный ток в цепи
  • Индивидуальные токи в каждом резисторе
  • Эквивалентное сопротивление

Перейдем от последнего к первому, чтобы вычислить все параметры. Таким образом, эквивалентное сопротивление при параллельном подключении сначала определяется по формуле:

1 / R = 1 / R1 + 1 / R2 + 1 / R3

выставив значения, 1 / R = 1/2 + 1/2 + 1/2 = 3/2

, следовательно, R = 2/3 Ом

Во-вторых, индивидуальный ток можно найти как:

I = V / R, так как параллельное напряжение одинаково и значение сопротивления здесь такое же.

Следовательно, I1 = V / R1 = 12/2 = 6 A

Аналогично I2 = I3 = 6 A.

Наконец, общий ток можно найти, сложив все токи ответвления. То есть I = I1 + I2 + I3 = 6 + 6 + 6 = 18 А.

Закон Ома в цепи переменного тока

Закон

Ома в цепи переменного тока такой же, за исключением коэффициента импеданса, который заменяет сопротивление в цепи постоянного тока. Импеданс — это не что иное, как сопротивление переменному току, возникающее при протекании тока в цепи переменного тока.

Разница в сопротивлении постоянному и переменному току

Импеданс (Z) также влияет на индуктивность и емкость, а также на сопротивление. 2) + j (Xl-Xc))

, где Xl = индуктивное реактивное сопротивление, равное

.

Xl = 2Π (f * L)

и Xc = емкостное реактивное сопротивление и выражается как:

Xc = 2Π (f * C)

где, f = частота системы

Ну я знаю, что вам еще непонятно.У тебя есть сомнения?

Да, знаю. Фактор «j» — это не что иное, как термин, обозначающий мнимую часть уравнения. Обычно j называется «йота».

Еще одна вещь заключается в том, что знание расчета катушек индуктивности в последовательном, параллельном и последовательном, параллельном емкостях является необходимым условием.

Расчет мощности

Еще один важный фактор, связанный с законом Ома при расчете мощности как в цепях переменного, так и в цепях постоянного тока.

Итак, что такое мощность? Что вы под этим понимаете?

сила мышц !!! ха-ха, ну может быть !! Но поверьте мне, у вашего мозга лучшая сила в мире! Не правда ли?

Сила разума зависит от того, насколько вы его чистите.

В любом случае, мы говорим об электрической мощности цепей постоянного тока.Это может быть определено как работа, выполненная за единицу времени, и измеряется в ваттах (джоуль / сек).

Треугольник силы

Треугольник мощности показывает соотношение между мощностью, электрическим напряжением и током. Уравнение, управляющее этим треугольником:

P = V * I

В основном этот треугольник показывает, что если у нас есть два известных значения из трех, то третье можно легко вычислить.

  • Если заданы напряжение и ток и необходимо рассчитать мощность, то формула будет иметь вид:

P = V * I (Вт)

  • Если у нас есть значение тока и мощности, мы найдем напряжение в цепи по измененной формуле:

В = P / I (вольт)

  • Если указаны мощность и напряжение, мы рассчитаем ток как:

I = P / V (амперы)

Номинальная мощность резисторов

Ну, первый вопрос, который должен прийти в голову, — зачем нам оценивать резисторы?

Думай !! Считать !!!

Номинальная мощность говорит о максимальной мощности, допустимой для конкретного резистора. 2) R

Аналогично, если у нас есть параллельная цепь, тогда напряжения на всех ветвях одинаковы, а ток отличается.2) /

рэнд Различные уровни освещенности

Ограничения по закону Ома

Изменение температуры изменяет значение сопротивления.

Даже изменение любого другого физического параметра повлияет на сопротивление резистора.

Действительно только для двусторонних элементов, но не для односторонних элементов, таких как диоды, транзисторы и т. Д.

Также для нелинейных элементов закон Ома не действует. Это связано с тем, что для нелинейных элементов ток не течет линейно и, следовательно, изменяет значение сопротивления нелинейным образом.Тиристоры, электрическая дуга и т. Д. — вот несколько примеров.

Применение закона Ома

  • Этот закон используется в измерительных приборах постоянного тока, таких как амперметр и вольтметр постоянного тока.
  • Используется в схеме делителя напряжения для деления напряжения на выходном сопротивлении.
  • Есть одно приложение, которое мы ежедневно используем дома. Посмотрите на изображение и скажите, что это?
  • Да, это регулятор вентилятора. Ток в вентиляторе регулируется сопротивлением в цепи регулятора.
  • В различных электронных схемах требуется падение напряжения для достижения определенного значения напряжения. Это делается с помощью закона Ома.

Ну, есть еще тысячи приложений, где используется закон Ома.

Теперь у вас есть работа, прокомментируйте приложения закона Ома, которые мы ежедневно используем в нашей жизни.

Закон

Ома: что это такое и почему это важно?

Обновлено 28 декабря 2020 г.

Ли Джонсон

Электрические цепи повсеместно встречаются в нашей повседневной жизни.От сложных интегральных схем, управляющих устройством, которое вы читаете в этой статье, до проводки, которая позволяет вам включать и выключать лампочку в вашем доме, вся ваша жизнь была бы радикально другой, если бы вы не были окружены цепями повсюду. ты иди.

Но большинство людей на самом деле не изучают мельчайших деталей того, как работают схемы, и довольно простые уравнения, такие как закон Ома, которые объясняют взаимосвязь между ключевыми понятиями, такими как электрическое сопротивление, напряжение и электрический ток.Однако более глубокое погружение в физику электроники может дать вам гораздо более глубокое понимание основных правил, лежащих в основе большинства современных технологий.

Что такое закон Ома?

Закон Ома — одно из самых важных уравнений, когда дело доходит до понимания электрических цепей, но если вы собираетесь его понять, вам понадобится хорошее понимание основных понятий, которые он связывает: напряжение , ток и сопротивление . Закон Ома — это просто уравнение, которое описывает соотношение между этими тремя величинами для большинства проводников.

Напряжение — это наиболее часто используемый термин для обозначения разности электрических потенциалов между двумя точками, который обеспечивает «толчок», который позволяет электрическому заряду перемещаться по проводящей петле.

Электрический потенциал — это форма потенциальной энергии, подобная гравитационной потенциальной энергии, и определяется как электрическая потенциальная энергия на единицу заряда. Единицей измерения напряжения в системе СИ является вольт (В), а 1 В = 1 Дж / Кл, или один джоуль энергии на кулон заряда. Иногда ее также называют электродвижущей силой , или ЭДС.

Электрический ток — это скорость протекания электрического заряда через заданную точку в цепи, в системе СИ единицей измерения является ампер (А), где 1 А = 1 Кл / с (один кулон заряда в секунду). Он поставляется в форме постоянного (DC) и переменного (AC) тока, и хотя постоянный ток проще, цепи переменного тока используются для подачи энергии в большинство домашних хозяйств по всему миру, потому что его проще и безопаснее передавать на большие расстояния.

Последняя концепция, которую вам нужно понять, прежде чем приступить к рассмотрению закона Ома, — это сопротивление, которое является мерой сопротивления току, протекающему в цепи.Единицей измерения сопротивления в системе СИ является ом (в котором используется греческая буква омега, Ом), где 1 Ом = 1 В / А.

Уравнение закона Ома

Немецкий физик Георг Ом описал взаимосвязь между напряжением, током и сопротивлением в своем одноименном уравнении. Формула закона Ома:

В = IR

, где В, — напряжение или разность потенциалов, I — величина тока, а сопротивление R — конечная величина.

Уравнение можно легко переформулировать, чтобы получить формулу для расчета тока на основе напряжения и сопротивления или сопротивления на основе тока и напряжения. Если вам неудобно переставлять уравнения, вы можете найти треугольник закона Ома (см. Раздел «Ресурсы»), но это довольно просто для любого, кто знаком с основными правилами алгебры.

Ключевые моменты, которые показывает уравнение закона Ома, заключаются в том, что напряжение прямо пропорционально электрическому току (поэтому, чем выше напряжение, тем выше ток), и этот ток обратно пропорционален сопротивлению (поэтому чем выше сопротивление, тем ниже электрический ток).

Вы можете использовать аналогию с потоком воды, чтобы запомнить ключевые моменты, которые основаны на трубе с одним концом на вершине холма и одним концом внизу. Напряжение похоже на высоту холма (более крутой и высокий холм означает большее напряжение), текущий поток похож на поток воды (вода течет быстрее по крутому склону), а сопротивление похоже на трение между сторонами трубы. и вода (более тонкая труба создает большее трение и снижает скорость потока воды, как более высокое сопротивление для электрического тока).

Почему важен закон Ома?

Закон Ома жизненно важен для описания электрических цепей, поскольку он связывает напряжение с током, а значение сопротивления смягчает взаимосвязь между ними. Из-за этого вы можете использовать закон Ома для управления величиной тока в цепи, добавляя резисторы, чтобы уменьшить ток, и снимая их, чтобы увеличить величину тока.

Его также можно расширить, чтобы описать электрическую мощность (скорость потока энергии в секунду), потому что мощность P = IV, и поэтому вы можете использовать ее, чтобы гарантировать, что ваша схема обеспечивает достаточно энергии, например, для 60-ваттного прибора.

Для студентов-физиков наиболее важным в законе Ома является то, что он позволяет анализировать принципиальные схемы, особенно когда вы объединяете его с законами Кирхгофа, которые следуют из него.

Закон Кирхгофа по напряжению гласит, что падение напряжения вокруг любого замкнутого контура в цепи всегда равно нулю, а закон тока утверждает, что величина тока, протекающего в переходе или узле в цепи, равна величине, вытекающей из Это. Вы можете использовать закон Ома с законом напряжения, в частности, для расчета падения напряжения на любом компоненте схемы, что является общей проблемой, возникающей в классах электроники.

Примеры закона Ома

Вы можете использовать закон Ома, чтобы найти любую неизвестную величину из трех, при условии, что вам известны две другие величины для рассматриваемой электрической цепи. Работа с некоторыми базовыми примерами показывает, как это делается.

Во-первых, представьте, что у вас есть 9-вольтовая батарея, подключенная к цепи с общим сопротивлением 18 Ом. Сколько тока течет при подключении цепи? Изменив закон Ома (или используя треугольник), вы можете найти:

\ begin {align} I & = \ frac {V} {R} \\ & = \ frac {9 \ text {V}} {18 \ текст {Ω}} \\ & = 0.5 \ text {A} \ end {align}

Итак, 0,5 ампер тока течет по цепи. А теперь представьте, что это идеальная величина тока для компонента, который вы хотите запитать, но у вас есть только батарея на 12 В. Какое сопротивление вы должны добавить, чтобы убедиться, что компонент получает оптимальную силу тока? Опять же, вы можете переставить закон Ома и решить его, чтобы найти ответ:

\ begin {align} R & = \ frac {V} {I} \\ & = \ frac {12 \ text {V}} {0.5 \ text {A}} \\ & = 24 \ text {Ω} \ end {align}

Итак, вам понадобится резистор 24 Ом для завершения вашей схемы.Наконец, каково падение напряжения на резисторе 5 Ом в цепи с током 2 А, протекающим через нее? На этот раз стандартная форма закона V = IR работает нормально:

\ begin {align} V & = IR \\ & = 2 \ text {A} × 5 \ text {Ω} \\ & = 10 \ text {V} \ end {align}

Омические и неомические резисторы

Вы можете использовать закон Ома в огромном количестве ситуаций, но есть ограничения на его применимость — это не действительно фундаментальный закон физики. .Закон описывает линейную зависимость между напряжением и током, но это соотношение сохраняется только в том случае, если резистор или резистивный элемент цепи, с которым вы работаете, имеет постоянное сопротивление при различных значениях напряжения В и тока I .

Материалы, которые подчиняются этому правилу, называются омическими резисторами, и хотя большинство физических проблем связано с омическими резисторами, вы знакомы со многими неомическими резисторами из своей повседневной жизни.

Лампочка — прекрасный пример неомического резистора.Когда вы строите график зависимости В от I для омических резисторов, он показывает полностью прямолинейную зависимость, но если вы сделаете это для чего-то вроде лампочки, ситуация изменится. По мере того как нить накала в лампе нагревается, сопротивление лампы увеличивается на , что означает, что график становится кривой, а не прямой линией, и закон Ома не действует.

Основы закона

Ом — напряжение, ток и сопротивление

В предыдущем уроке мы обсудили применение тока, напряжения и важность закона Кулона в электричестве.Но без закона Ома работа электрической цепи становится неполной.

Для выполнения этого также задействован закон Ома . Немецкий физик Георг Симон Ом открыл закон Ома и обнаружил взаимосвязь между током, напряжением и сопротивлением.

Из этого руководства вы узнаете, как применить закон сопротивления к различным приложениям электротехники и электроники.

Как известно, электрический ток течет в виде заряженных электронов.Другими словами, меньший поток электронов означает высокое сопротивление в цепи. А высокий поток электронов означает низкое сопротивление.

Электронный ток — это количество электронов, движущихся за секунду. Однако для практических приложений нам нужно меньшее количество заряда электрона. Для упрощения использовались две единицы измерения, известные как ампер и кулонов .

Кулон, обозначенный «, выражает достаточное количество электрического заряда.

Электрический заряд равен 6 миллионам электронов, умноженным на 1 миллион электронов. Этот результирующий заряд снова умножается на один миллион электронов.

Ампер, представленный как ‘A’. — это сила единичного тока или количество электронов, перемещающихся в секунду, в данном случае один кулон в секунду.

Ампер слишком велик для некоторых приложений. Таким образом, он снова делится на части, известные как миллиампер ( мА), и микроампер ( мкА, ).

  1 А = 1000 мА = 1 000 000 мкА
1 мА = 1/1000 А = 1000 мкА
  

Теперь поговорим о сопротивлении. Поскольку состав различных материалов отличается, некоторые материалы обладают более сильным противодействием потоку электронов, чем другие металлы. Это электрическое явление известно как сопротивление .

Теперь, если мы приложим движущую силу или Электромагнитную силу (E.M.F) к проводнику, большое количество электронов потечет быстро.Это доказывает, что сопротивление проводника невелико.

С другой стороны, применение того же Э. М. Ф. к изолятору произведет меньше электронов. Следовательно, сопротивление изолятора высокое.

Сопротивление выражается в Ом и обозначается греческой заглавной буквой «Ω ». Единицей измерения E.M.F является вольт. Один вольт — это движущая сила, необходимая для создания силы тока 1 А в цепи с сопротивлением 1 Ом.

Электрический ток — это измерительный прибор, называемый амперметром, а электрическое сопротивление измеряется с помощью омметра.

Как работает закон Ома?

Закон

Ома связывает электрические величины, такие как ток, напряжение, мощность и сопротивление. Чтобы узнать о практическом применении закона Ома, приведу пример.

Подключите провод определенного сопротивления последовательно к источнику батареи 1,5 В и предположите, что амперметр показывает ток 0,2 А. Теперь, если мы увеличим напряжение до 3 В, измеритель тока покажет большее значение тока, скажем, 0,4 А.

Это означает, что при поддержании постоянного сопротивления и увеличении напряжения ток будет удвоен.Повторяя этот процесс увеличения и уменьшения напряжения, сохраняя неизменным сопротивление, напряжение будет пропорционально току.

То же самое происходит, если мы изменяем длину проводящего провода, сохраняя приложенное напряжение постоянным.

Если мы изменим длину провода на более короткую или более длинную, это будет иметь некоторый эффект из-за сопротивления провода.

Например, если приложить постоянный ЭДС 1,5 В и длину провода 2 м, потребляемый ток будет равен 0.3А.

Теперь, если мы изменим длину провода на 1 м, ток будет меньше 0,1 (но не 0,3) из-за меньшего расстояния, которое нужно преодолеть, и меньшего сопротивления, которое необходимо преодолеть.

Теория закона Ома

Когда вы берете металлический проводник и пропускаете через него ток, разность потенциалов между двумя концами проводника остается постоянной.

Определение закона Ома

Закон

Ом гласит, что «ток, протекающий через электрическую цепь, будет изменяться при приложении напряжения, но сопротивление обратно пропорционально сопротивлению материала проводника».

Формула закона Ома представлена ​​уравнением

В = ИК

‘V’ — падение потенциала (напряжения) на резисторе.

«I» — ток, протекающий в цепи через резистор

.

‘R’ — значение сопротивления резистора, выраженное в омах.

Приведенное выше уравнение I = V / R отражает следующие факты.

  • Ток меняется в зависимости от приложенного входного напряжения

Если сопротивление проводника остается постоянным, напряжение будет увеличиваться с увеличением тока и напряжение уменьшаться с уменьшением тока.

  • Ток и сопротивление противоположны друг другу

Теперь сохраните напряжение в цепи как постоянный параметр. Если вы измените сопротивление, ток также изменится.

Например, если сопротивление увеличивается, ток в цепи уменьшается, а если сопротивление уменьшается, ток увеличивается.

  • Соотношение напряжения и тока

Связь между напряжением и током линейная.то есть с большим напряжением ток будет выше, а с меньшим напряжением — меньше ток.

Аналогия закона Ома

Связь между напряжением, током и сопротивлением можно узнать, найдя третью величину из двух известных значений.

Двумя известными значениями могут быть напряжение, ток или сопротивление.

Расчет по закону Ома

Закон Ома можно представить в трех формах. Проще говоря, закон Ом, окружность или закон Ом. Треугольник используется в электрических цепях для определения третьей величины из двух других величин.

Метод круга или треугольника используется для запоминания закона Ома.

Здесь я использую круг закона Ома, чтобы узнать напряжение, ток и сопротивление.

  1. Чтобы рассчитать напряжение (В) , округлите напряжение (В), как показано ниже. Ток и сопротивление взаимосвязаны.

V = I x R

  1. Чтобы вычислить ток (в амперах) , округлите ток (I), как показано ниже. Это будет ток, протекающий в цепи.

I = V / R

  1. Аналогично, , чтобы узнать сопротивление (Ом) , округлив сопротивление (R), вы получите сопротивление проводника.

R = V / I

Комбинируя напряжение, ток и сопротивление, мы можем получить общее соотношение, чтобы нарисовать график закона Ом .

Из графика видно, что если ток в электрической цепи увеличивается, напряжение увеличивается линейно, и наоборот.

Для облегчения связи Закон Ома Таблица приведена для быстрого ознакомления.

Закон Ома Известные значения Отношение 1 Отношение 2
V = IR Ток и сопротивление Напряжение прямо пропорционально току Напряжение прямо пропорционально сопротивлению
I = V / R Напряжение и сопротивление Ток обратно пропорционален сопротивлению Ток прямо пропорционален напряжению
R = V / I Напряжение и ток Сопротивление прямо пропорционально напряжению Сопротивление обратно пропорционально току

Теперь дайте нам знать , как использовать формулы закона Ома на практике.

Примеры закона Ома

1 . Определение тока в цепи

Дано: напряжение = 5 В, сопротивление = 500 Ом, I =?

Формула:

I = V / R = 5/500 = 0,01 А.

Итак, при приложении потенциала 5 В через резистор на 500 Ом протекает ток 0,01 А.

2. Определение напряжения в цепи

Дано: Сопротивление = 100 Ом, I = 2 А, Напряжение =?

Формула:

В = ИК = 2 * 100 = 200 В

Итак, напряжение АКБ для схемы составляет 200В.

3. Определение сопротивления в цепи

Дано: I = 2 А, напряжение = 5 В, сопротивление =?

Формула:

R = V / I = 5/2 = 2,5 Ом

Таким образом, необходимо последовательно подключить к источнику батареи сопротивление 2,5 Ом.

Практическое применение закона Ома

1. Устройство блока питания (как делитель напряжения)

Закон

Ом полезен при проектировании источников питания для электронных схем.Делители напряжения определяют регулируемый выход для правильного функционирования схемы. Это достигается выбором правильного сопротивления по закону Ома.

2. Аналоговые датчики

Некоторые типы датчиков выдают текущее значение на выходе. Например, датчик радара дает выходной ток 4-20 мА.

Этот выходной ток должен быть преобразован в напряжение с помощью уравнения сопротивления. Полученное аналоговое напряжение затем обрабатывается через АЦП (аналого-цифровой преобразователь).

3. Контроль скорости

Закон

Ом широко используется в приложениях, регулирующих скорость. Он используется в потенциометре , также известном как « POT ». Сопротивление ручки изменяется медленно, что увеличивает напряжение и вращает двигатель или вентилятор.

4. Упрощение схем

Он также используется для сокращения сложных электрических цепей с использованием закона напряжения Кирхгофа и уравнения закона тока Кирхгофа .Последовательные и параллельные цепи могут быть реализованы просто с помощью закона Ома.

Заключение

В реальной жизни важно узнать ток и напряжение для любого приложения. Небольшое отклонение выходной нагрузки может привести к возгоранию или повреждению цепи. Чтобы этого избежать, необходимо применить принципы закона Ома и построить действующую электронную систему.

Закон

Ом — Заявление и формула закона Ома

Ом используется термопара, которая является стабильным источником напряжения, когда речь идет о внутреннем сопротивлении, и постоянным источником напряжения.Для измерения напряжения он использовал гальванометр. Ом знал о том, что температура спая пропорциональна напряжению на двух выводах термопары. Чтобы замкнуть цепь или сделать ее замкнутой, он использовал различные провода разной длины, свойств и диаметра. Полученные им результаты могут быть получены на основе уравнения:

X = a / (b + l), где x — показание гальванометра, a — постоянная, зависящая от температуры перехода соединения, b — постоянная величина, l — длина используемого проводника.

В свете приведенного выше уравнения он вывел закон пропорциональности как

I = E / (r + Rl), где E — ЭДС термопары, внутреннее сопротивление — r, а R — сопротивление испытательного провода на единицу. длина.

Произведения Ома, впечатленные его работами, которые привели к множеству новых открытий, были названы «паутиной обнаженных фантазий».

Согласно закону Ома величина тока, протекающего по проводнику, прямо пропорциональна величине разности потенциалов, приложенной к двум клеммам.Уравнение закона Ома: I = V / R, здесь коэффициент пропорциональности R — сопротивление, V — напряжение, а I — ток, протекающий по проводу. В этом законе сопротивление считается постоянным и не зависит от протекающего через него тока. Единица измерения R — ом, I — ампер, а V — вольт, в стандартных единицах. Закон также определяет проводимость материала, по которому течет ток.

Закон Ома часто обобщается в физике для многих других приложений.

Эмпирический закон, который представляет собой закон Ома, в его обобщенной форме утверждает, что ток пропорционален электрическому полю. Но это обобщение применимо не ко всем материалам. Некоторые материалы могут разрушаться в присутствии очень сильного электрического поля, а некоторые материалы не подчиняются этому закону в присутствии слабых электрических полей. Такие материалы, которые не подчиняются закону Ома, известны как неомические материалы. Но когда этот закон соблюдается, он применим даже к очень мелким масштабам, таким как атомный масштаб.

В электромагнетизме он используется в векторной форме, в которой указано, что J = σE. Здесь j рассматривается как плотность тока в определенном месте материала, E — электрическое поле, а σ — проводимость, которая является свойством используемого материала.

Чтобы понять закон Ома, нам нужно понять три основных принципа, а именно: напряжение, ток и сопротивление.

Напряжение: количество энергии, приходящейся на кулон, называется напряжением.Его также можно описать как количество потенциальной энергии между двумя терминалами. 1В определяется как разность потенциалов между двумя выводами, которая дает нам 1 Джоуль энергии на кулон зарядов, проходящих через него.

Сопротивление: свойство материала, благодаря которому он имеет тенденцию сопротивляться прохождению тока через него, известно как сопротивление. Таким образом, схема, в которой значение сопротивления больше, позволит меньшему количеству зарядов проходить через нее, а схема с меньшим значением сопротивления позволит протекать количеству электронов и, таким образом, поддерживать количество тока, протекающего через Это.

В проводе с равномерной площадью поперечного сечения значение сопротивления будет зависеть от величины площади поперечного сечения и длины провода. Он прямо пропорционален l / A.

R = ρ

Сопротивление также зависит от температуры проводника.

Ток: Скорость прохождения зарядов через определенную площадь поперечного сечения называется током. Другими словами, 1 А описывается как величина тока при заряде 1С или 6.24 x 10-19 электронов проходят через площадь поперечного сечения за единицу времени.

Вот некоторые другие важные параметры:

Скорость дрейфа: в проводнике присутствующие ионы подвижны и постоянно перемещаются беспорядочно. Для существования чистого потока заряда необходимо, чтобы частицы двигались вместе со средней скоростью. В металлических частицах электроны являются носителями заряда, которые движутся в направлении, противоположном направлению электрического тока, протекающего по проводнику.Движение идет в случайном направлении. Эта скорость или дрейф называется скоростью дрейфа. Значение скорости дрейфа можно рассчитать по уравнению:

I = nAvQ, где n — количество носителей заряда в единице объема, A — площадь поперечного сечения, через которое протекает ток, v — средняя скорость дрейфа, I — ток, а Q — величина заряда на каждой несущей. Обычно скорость или скорость дрейфа электронов в проводнике намного меньше. Например, возьмем медный провод, площадь сечения которого равна 0.5 мм2, а ток, протекающий через него, равен 5 A. Таким образом, скорость дрейфа частиц будет в миллиметрах, что и в секунду.

Удельное сопротивление: величина, обратная проводимости, называется удельным сопротивлением.

E = ρJ, где ρ называется удельным сопротивлением.

ОГРАНИЧЕНИЯ ЗАКОНА ОМА:

Закон Ома не лишен ограничений. Это:

В односторонних сетях или сетях, которые допускают протекание тока только в одном направлении и состоят из других электрических элементов, таких как диод, транзистор и т. Д., Закон Ома не работает и не может применяться в сети.

Нелинейные элементы или элементы, в которых ток не определенно пропорционален приложенному напряжению. В таких элементах сопротивление не является постоянной величиной и постоянно меняется с изменением величины приложенного напряжения и тока. Следовательно, такие элементы не подчиняются закону Ома, который изменяет значение сопротивления.

ПРИМЕНЕНИЕ ЗАКОНА ОМА:

Закон Ома помогает нам определять значения сопротивления, тока, протекающего по цепи, и приложенного напряжения.Следовательно, с помощью этих значений мы можем найти значения других факторов, таких как скорость дрейфа, удельное сопротивление и многие другие. Это также позволяет нам рассчитать значение потребляемой мощности.

ИСПОЛЬЗОВАНИЕ В ЕЖЕДНЕВНОЙ ЖИЗНИ:

Будучи фундаментальным законом, закон Ома имеет множество практических применений в электрических компонентах и, следовательно, в электроприборах. Давайте сосредоточимся на некоторых практических применениях закона Ома, с которыми мы сталкиваемся в нашей повседневной жизни.

Используется для управления скоростью вентиляторов: все мы знаем, что такое потенциометр.Электрический компонент, сопротивление которого имеет переменное значение, известен как потенциометр. Для управления скоростью стандартного вентилятора можно использовать потенциометр. Это достигается с помощью круглой ручки. Ручка фиксируется на компоненте. Эта ручка вращается и используется для достижения желаемого значения сопротивления на выходном компоненте. Следовательно, для конкретного значения сопротивления входа мы можем вычислить значение сопротивления, протекающего тока. Итак, это дает нам знание силы.Эти значения рассчитываются с помощью закона Ома.

Расчет требуемой мощности компонентов: для работы любых электроприборов, таких как утюг, электрический чайник и многие другие, используется огромное количество резисторов. Они необходимы для правильного функционирования этих приборов. Для обеспечения правильной работы этих резисторов требуется правильный выбор мощности в ваттах. Мощность рассчитывается по формуле P = VI.

Потребляемая мощность и мощность, подаваемая электронным устройством: Катушка, используемая в нагревателе, и приложенное напряжение помогают нам определить мощность электрического нагревателя.Когда это вычисляется, мощность умножается на продолжительность времени, в течение которого она использовалась, а также на количество дней, после чего мы получаем сумму, которую нам нужно заплатить в соответствии со счетом за электроэнергию.

Предохранители: Закон Ома также полезен при выборе предохранителей, которые будут использоваться. Для защиты цепи используются предохранители и автоматические выключатели. Они соединены последовательно с электрическими приборами. Закон Ома позволяет нам найти значение тока, который может протекать через предохранители.Если значение тока слишком велико, это может привести к повреждению цепи и даже к взрыву электронного устройства. Есть два случая, когда для выбора предохранителей можно использовать закон Ома. Первый — когда сопротивление известно, а второй — когда значение сопротивления неизвестно.

Что такое закон Ома? — Наука для детей

Определение закона Ома

Закон Ома — это математическая связь между электрическим током, сопротивлением и напряжением.

Закон

Ома гласит, что ток через проводник между двумя точками прямо пропорционален напряжению в этих двух точках.

Принцип назван в честь немецкого ученого Георга Симона Ома.

Формула закона Ома

Формула закона Ома выглядит следующим образом:

В = ИК

где

V — напряжение
I — ток
R — сопротивление

Как работает закон Ома?

Непрерывный поток свободных электронов через проводники цепи называется током.Электрическая цепь образуется, когда создается проводящий путь, позволяющий свободным электронам непрерывно перемещаться.

Сила, которая движет потоком электронов, называется напряжением. Это особая мера потенциальной энергии, которая всегда относительна между двумя точками.

Свободные электроны имеют тенденцию проходить через проводники с некоторой степенью трения или сопротивления движению. Это противодействие движению свободных электронов называется сопротивлением.

Используя этот Закон, мы можем анализировать электрические цепи.Если вам известны два любых значения, вы можете проанализировать третье. Иногда электрические цепи являются сложными, но это уравнение настолько важно, что оно также решает эти сложные значения схемы. Применяется практически во всех схемотехнических исследованиях.

Практическое применение закона Ома

  1. Закон Ома используется в электрических нагревателях для выработки тепла. Проводник предназначен для создания сопротивления потоку свободных электронов, поэтому сопротивление создает тепло.
  2. Потолочные и другие вентиляторы также используют закон Ома.Скорость регулируется с помощью приложения закона Ома.
  3. Лампочки излучают свет в соответствии с законом Ома

Вот простое и веселое упражнение. Обойдите свой дом и посмотрите, какой объект использует закон Ома! Составьте список и сравните его со своими друзьями. Повеселись!

Закон Ома

| Электрические цепи

Закон 11,2 Ома (ESBQ6)

Три основные величины для электрических цепей: ток, напряжение (разность потенциалов) и сопротивление .Резюме:

  1. Электрический ток, \ (I \), определяется как скорость прохождения заряда через цепь.

  2. Разность потенциалов или напряжение \ (В \) — это количество энергии на единицу заряда, необходимое для перемещения этого заряда между двумя точками в цепи.

  3. Сопротивление, \ (R \), является мерой того, насколько «трудно» протолкнуть ток через элемент схемы.

Теперь посмотрим, как эти три величины связаны друг с другом в электрических цепях.

Важная взаимосвязь между током, напряжением и сопротивлением в цепи была обнаружена Георгом Симоном Омом и называется законом Ома .

Закон Ома

Величина электрического тока через металлический проводник при постоянной температуре в цепи пропорциональна напряжению на проводнике и может быть описана как

. \ (I = \ frac {V} {R} \)

где \ (I \) — ток через проводник, \ (V \) — напряжение на проводнике, а \ (R \) — сопротивление проводника.Другими словами, при постоянной температуре сопротивление проводника постоянно, независимо от приложенного к нему напряжения или проходящего через него тока.

Закон Ома говорит нам, что если проводник имеет постоянную температуру, ток, протекающий через проводник, прямо пропорционален напряжению на нем. Это означает, что если мы нанесем напряжение на ось x графика, а ток — на ось y графика, мы получим прямую линию.

Наклон прямолинейного графика связан с сопротивлением проводника как \ [\ frac {I} {V} = \ frac {1} {R}.\] Это можно изменить с точки зрения постоянного сопротивления как: \ [R = \ frac {V} {I}. \]

Закон Ома

Цель

Для определения взаимосвязи между током, протекающим через резистор, и разностью потенциалов (напряжением) на том же резисторе.

Аппарат

4 ячейки, 4 резистора, амперметр, вольтметр, соединительные провода

Метод

Этот эксперимент состоит из двух частей. В первой части мы будем изменять приложенное к резистору напряжение и измерять результирующий ток в цепи.Во второй части мы будем изменять ток в цепи и измерять результирующее напряжение на резисторе. После получения обоих наборов измерений мы исследуем взаимосвязь между током и напряжением на резисторе.

  1. Изменение напряжения:

    1. Установите схему в соответствии со схемой 1), начиная с одной ячейки.

    2. Нарисуйте следующую таблицу в своем лабораторном журнале.

      Количество ячеек

      Напряжение, В (\ (\ text {V} \))

      Ток, I (\ (\ text {A} \))

      \ (\ text {1} \)

      \ (\ text {2} \)

      \ (\ text {3} \)

      0

      \ (\ text {4} \)

    3. Попросите учителя проверить электрическую цепь перед включением питания.

    4. Измерьте напряжение на резисторе с помощью вольтметра и ток в цепи с помощью амперметра.

    5. Добавьте в схему еще одну ячейку \ (\ text {1,5} \) \ (\ text {V} \) и повторите измерения.

    6. Повторяйте, пока не получите четыре ячейки и не заполните таблицу.

  2. Изменение тока:

    1. Установите схему в соответствии со схемой 2), начиная с одного резистора в цепи.

    2. Нарисуйте следующую таблицу в своем лабораторном журнале.

      Напряжение, В (\ (\ text {V} \))

      Ток, I (\ (\ text {A} \))

    3. Попросите учителя проверить вашу схему перед включением питания.

    4. Измерьте ток и напряжение на единственном резисторе.

    5. Теперь добавьте еще один резистор последовательно в схему и снова измерьте ток и напряжение только на исходном резисторе. Продолжайте добавлять резисторы, пока у вас не будет четырех последовательно, но не забывайте каждый раз измерять напряжение только на исходном резисторе. Введите измеренные вами значения в таблицу.

Анализ и результаты

  1. Используя данные, записанные в первой таблице, постройте график зависимости тока от напряжения.Поскольку напряжение — это переменная, которую мы изменяем напрямую, это независимая переменная, которая будет отложена по оси \ (x \). Ток является зависимой переменной и должен быть нанесен на ось \ (y \).

  2. Используя данные, записанные во второй таблице, постройте график зависимости напряжения от тока. В этом случае независимой переменной является ток, который должен быть нанесен на ось \ (x \), а напряжение является зависимой переменной и должно быть нанесено на ось \ (y \).

Выводы

  1. Изучите график, который вы построили из первой таблицы. Что происходит с током через резистор при увеличении напряжения на нем? т.е. увеличивается или уменьшается?

  2. Изучите график, который вы построили на основе второй таблицы. Что происходит с напряжением на резисторе, когда ток через резистор увеличивается? т.е. увеличивается или уменьшается?

  3. Подтверждают ли результаты ваших экспериментов закон Ома? Объяснять.

Вопросы и обсуждение

  1. Для каждого из ваших графиков вычислите градиент и по нему определите сопротивление исходного резистора. Получаете ли вы одно и то же значение, когда рассчитываете его для каждого из ваших графиков?
  2. Как вы можете найти сопротивление неизвестного резистора, используя только источник питания, вольтметр и известный резистор \ (R_0 \)?

Высокие оценки в науке — залог вашего успеха и будущих планов.Проверьте себя и узнайте больше о практике Сиявулы.

Зарегистрируйтесь и проверьте себя

Закон Ома

Упражнение 11.1

Постройте график напряжения (по оси X) и тока (по оси Y).

Какой тип графика вы получите (прямой, парабола, другая кривая)

прямая линия

Рассчитайте градиент графика.

Градиент графика (\ (m \)) — это изменение тока, деленное на изменение напряжения:

\ begin {align *} m & = \ frac {\ Delta I} {\ Delta V} \\ & = \ frac {(\ text {1,6}) — (\ text {0,4})} {(\ text {12}) — (\ text {3})} \\ & = \ текст {0,13} \ end {выровнять *}

Подтверждают ли результаты ваших экспериментов закон Ома? Объяснять.

Да. График с прямой линией получается, когда мы строим график зависимости напряжения от тока.

Как вы можете определить сопротивление неизвестного резистора, используя только источник питания, вольтметр и известный резистор \ (R_ {0} \)?

Вы начинаете с подключения известного резистора в цепь с источником питания. Теперь вы читаете напряжение источника питания и записываете его.

Затем вы последовательно подключаете два резистора.Теперь вы можете измерить напряжение для каждого из резисторов.

Итак, мы можем найти напряжения для двух резисторов. Теперь отметим, что:

\ [V = IR \]

Итак, используя это и тот факт, что для резисторов, включенных последовательно, ток одинаков во всей цепи, мы можем найти неизвестное сопротивление.

\ begin {align *} V_ {0} & = IR_ {0} \\ I & = \ frac {V_ {0}} {R_ {0}} \\ V_ {U} & = IR_ {U} \\ I & = \ frac {V_ {U}} {R_ {U}} \\ \ frac {V_ {U}} {R_ {U}} & = \ frac {V_ {0}} {R_ {0}} \\ \ поэтому R_ {U} & = \ frac {V_ {U} R_ {0}} {V_ {0}} \ end {выровнять *}

Омические и неомические проводники (ESBQ7)

Проводники, подчиняющиеся закону Ома, имеют постоянное сопротивление, когда на них изменяется напряжение или увеличивается ток через них.Эти проводники называются омическими проводниками . График зависимости тока от напряжения на этих проводниках будет прямолинейным. Некоторыми примерами омических проводников являются резисторы цепи и нихромовая проволока.

Как вы видели, когда мы говорим о законе Ома, есть упоминание о постоянной температуре . Это связано с тем, что сопротивление некоторых проводников изменяется при изменении их температуры. Эти типы проводников называются неомическими проводниками , потому что они не подчиняются закону Ома.Лампочка — типичный пример неомического проводника. Другими примерами неомических проводников являются диоды и транзисторы.

В лампочке сопротивление нити накала резко возрастает по мере того, как она нагревается от комнатной до рабочей температуры. Если мы увеличим напряжение питания в реальной цепи лампы, то увеличение тока приведет к увеличению температуры нити накала, что приведет к увеличению ее сопротивления. Это эффективно ограничивает увеличение тока.В этом случае напряжение и ток не подчиняются закону Ома.

Явление изменения сопротивления при изменении температуры присуще почти всем металлам, из которых сделано большинство проводов. Для большинства приложений эти изменения сопротивления достаточно малы, чтобы их можно было игнорировать. При применении металлических нитей накала ламп, температура которых сильно повышается (примерно до \ (\ text {1 000} \) \ (\ text {℃} \) и начиная с комнатной температуры), изменение довольно велико.

В общем, для неомических проводников график зависимости напряжения от тока не будет прямолинейным, что указывает на то, что сопротивление не является постоянным для всех значений напряжения и тока.

Включен рекомендуемый эксперимент для неформальной оценки. В этом эксперименте учащиеся получат данные о токе и напряжении для резистора и лампочки и определят, какой из них подчиняется закону Ома. Вам потребуются лампочки, резисторы, соединительные провода, источник питания, амперметр и вольтметр. Учащиеся должны обнаружить, что резистор подчиняется закону Ома, а лампочка — нет.

Омические и неомические проводники

Aim

Чтобы определить, подчиняются ли два элемента схемы (резистор и лампочка) закону Ома

Аппарат

4 ячейки, резистор, лампочка, соединительные провода, вольтметр, амперметр

Метод

Две схемы, показанные на схемах выше, одинаковы, за исключением того, что в первой есть резистор, а во второй — лампочка.Настройте обе схемы, указанные выше, начиная с 1 ячейки. Для каждой цепи:

  1. Измерьте напряжение на элементе схемы (резисторе или лампочке) с помощью вольтметра.

  2. Измерить ток в цепи с помощью амперметра.

  3. Добавьте еще одну ячейку и повторяйте измерения, пока в вашей цепи не будет 4 ячейки.

Результаты

Нарисуйте в своей книге две таблицы, которые выглядят следующим образом.У вас должна быть одна таблица для измерений первой цепи с резистором и другая таблица для измерений второй цепи с лампочкой.

Количество ячеек

Напряжение, В (\ (\ text {V} \))

Ток, I (\ (\ text {A} \))

\ (\ text {1} \)

\ (\ text {2} \)

\ (\ text {3} \)

0

\ (\ text {4} \)

Анализ

Используя данные в ваших таблицах, нарисуйте два графика \ (I \) (\ (y \) — ось) vs.\ (V \) (\ (x \) — ось), один для резистора и один для лампочки.

Вопросы и обсуждение

Внимательно изучите свои графики и ответьте на следующие вопросы:

  1. Как должен выглядеть график зависимости \ (I \) от \ (V \) для проводника, подчиняющегося закону Ома?

  2. Один или оба ваших графика выглядят так?

  3. Какой можно сделать вывод о том, подчиняются ли резистор и / или лампочка закону Ома?

  4. Имеет ли лампочка омический или неомический провод?

Использование закона Ома (ESBQ8)

Теперь мы готовы увидеть, как закон Ома используется для анализа схем.

Рассмотрим схему с ячейкой и омическим резистором R. Если резистор имеет сопротивление \ (\ text {5} \) \ (\ text {Ω} \), а напряжение на резисторе равно \ (\ text { 5} \) \ (\ text {V} \), то мы можем использовать закон Ома для расчета тока, протекающего через резистор. Наша первая задача — нарисовать принципиальную схему. При решении любой проблемы с электрическими схемами очень важно составить схему схемы перед тем, как производить какие-либо расчеты. Принципиальная схема для этой проблемы выглядит следующим образом:

Уравнение закона Ома: \ [R = \ frac {V} {I} \]

, который можно преобразовать в: \ [I = \ frac {V} {R} \]

Ток, протекающий через резистор:

\ begin {align *} I & = \ frac {V} {R} \\ & = \ frac {\ text {5} \ text {V}} {\ text {5} \ Omega} \\ & = \ текст {1} \ текст {А} \ end {align *}

Рабочий пример 1: Закон Ома

Изучите принципиальную схему ниже:

Сопротивление резистора равно \ (\ text {10} \) \ (\ text {Ω} \), а ток, проходящий через резистор, равен \ (\ text {4} \) \ (\ text {A} \ ).Какова разность потенциалов (напряжение) на резисторе?

Определите, как подойти к проблеме

Нам задают сопротивление резистора и ток, проходящий через него, и просят вычислить напряжение на нем. Мы можем применить закон Ома к этой проблеме, используя: \ [R = \ frac {V} {I}. \]

Решить проблему

Измените приведенное выше уравнение и замените известные значения на \ (R \) и \ (I \), чтобы найти \ (V \). \ begin {align *} R & = \ frac {V} {I} \\ R \ times I & = \ frac {V} {I} \ times I \\ V & = I \ раз R \\ & = \ текст {10} \ times \ text {4} \\ & = \ текст {40} \ текст {V} \ end {align *}

Напишите окончательный ответ

Напряжение на резисторе равно \ (\ text {40} \) \ (\ text {V} \).

Высокие оценки в науке — залог вашего успеха и будущих планов. Проверьте себя и узнайте больше о практике Сиявулы.

Зарегистрируйтесь и проверьте себя

Закон Ома

Упражнение 11.2

Рассчитайте сопротивление резистора, разность потенциалов которого составляет \ (\ text {8} \) \ (\ text {V} \), когда ток равен \ (\ text {2} \) \ (\ text {A} \) протекает через него. Перед расчетом нарисуйте принципиальную схему.

Сопротивление неизвестного резистора составляет:

. \ begin {align *} R & = \ frac {V} {I} \\ & = \ frac {8} {2} \\ & = \ текст {4} \ текст {Ω} \ end {выровнять *}

Какой ток будет проходить через резистор \ (\ text {6} \) \ (\ text {Ω} \) при разности потенциалов \ (\ text {18} \) \ (\ text {V} \) на концах? Перед расчетом нарисуйте принципиальную схему.

Сопротивление неизвестного резистора составляет:

. \ begin {align *} I & = \ frac {V} {R} \\ & = \ frac {18} {6} \\ & = \ текст {3} \ текст {А} \ end {выровнять *}

Какое напряжение на резисторе \ (\ text {10} \) \ (\ text {Ω} \) при токе \ (\ text {1,5} \) \ (\ text {A} \) течет хоть это? Перед расчетом нарисуйте принципиальную схему.

Сопротивление неизвестного резистора составляет:

. \ begin {align *} V & = I \ cdot R \\ & = (\ текст {1,5}) (10) \\ & = \ текст {15} \ текст {V} \ end {выровнять *}

Резисторы последовательно и параллельно (ESBQ9)

В 10 классе вы узнали о резисторах и познакомились со схемами, в которых резисторы подключены последовательно и параллельно.В последовательной цепи есть один путь, по которому течет ток. В параллельной цепи есть несколько путей, по которым течет ток.

Когда в цепи более одного резистора, мы обычно можем рассчитать общее суммарное сопротивление всех резисторов. Это известно как сопротивление , эквивалентное .

Эквивалентное последовательное сопротивление

В цепи, в которой резисторы включены последовательно, эквивалентное сопротивление — это просто сумма сопротивлений всех резисторов.

Эквивалентное сопротивление в последовательной цепи,

Для последовательно подключенных n резисторов эквивалентное сопротивление составляет:

. \ [R_ {s} = R_ {1} + R_ {2} + R_ {3} + \ ldots + R_ {n} \]

Применим это к следующей схеме.

Резисторы включены последовательно, следовательно:

\ begin {align *} R_ {s} & = R_ {1} + R_ {2} + R_ {3} \\ & = \ text {3} \ text {Ω} + \ text {10} \ text {Ω} + \ text {5} \ text {Ω} \\ & = \ текст {18} \ текст {Ω} \ end {align *}
Эквивалентное параллельное сопротивление

В цепи, в которой резисторы включены параллельно, эквивалентное сопротивление определяется следующим определением.

Эквивалентное сопротивление в параллельной цепи

Для параллельных резисторов \ (n \) эквивалентное сопротивление составляет:

\ [\ frac {1} {R_ {p}} = \ frac {1} {R_ {1}} + \ frac {1} {R_ {2}} + \ frac {1} {R_ {3}} + \ ldots + \ frac {1} {R_ {n}} \]

Применим эту формулу к следующей схеме.

Какое полное (эквивалентное) сопротивление в цепи?

\ begin {align *} \ frac {1} {R_ {p}} & = \ left (\ frac {1} {R_ {1}} + \ frac {1} {R_ {2}} + \ frac {1} {R_ {3}) } \верно) \\ & = \ left (\ frac {1} {\ text {10} \ text {Ω}} + \ frac {1} {\ text {2} \ text {Ω}} + \ frac {1} {\ text { 1} \ text {Ω}} \ right) \\ & = \ left (\ frac {\ text {1} \ text {Ω} + \ text {5} \ text {Ω} + \ text {10} \ text {Ω}} {\ text {10} \ text { Ω}} \ right) \\ & = \ left (\ frac {\ text {16} \ text {Ω}} {\ text {10} \ text {Ω}} \ right) \\ R_ {p} & = \ text {0,625} \ text {Ω} \ end {align *}

Последовательное и параллельное сопротивление

Упражнение 11.3

Два резистора \ (\ text {10} \) \ (\ text {kΩ} \) соединены последовательно. Рассчитайте эквивалентное сопротивление.

Поскольку резисторы включены последовательно, мы можем использовать:

\ [R_ {s} = R_ {1} + R_ {2} \]

Эквивалентное сопротивление:

\ begin {align *} R_ {s} & = R_ {1} + R_ {2} \\ & = \ text {10} \ text {kΩ} + \ text {10} \ text {kΩ} \\ & = \ текст {20} \ текст {кОм} \ end {выровнять *}

Два резистора соединены последовательно.Эквивалентное сопротивление равно \ (\ text {100} \) \ (\ text {Ω} \). Если один резистор равен \ (\ text {10} \) \ (\ text {Ω} \), вычислите номинал второго резистора.

Поскольку резисторы включены последовательно, мы можем использовать:

\ [R_ {s} = R_ {1} + R_ {2} \]

Эквивалентное сопротивление:

\ begin {align *} R_ {s} & = R_ {1} + R_ {2} \\ R_ {2} & = R_ {s} — R_ {1} \\ & = \ text {100} \ text {Ω} — \ text {10} \ text {Ω} \\ & = \ текст {90} \ текст {Ω} \ end {выровнять *}

Два резистора \ (\ text {10} \) \ (\ text {kΩ} \) подключены параллельно.Рассчитайте эквивалентное сопротивление.

Поскольку резисторы включены параллельно, мы можем использовать:

\ [\ frac {1} {R_ {p}} = \ frac {1} {R_ {1}} + \ frac {1} {R_ {2}} \]

Эквивалентное сопротивление:

\ begin {align *} \ frac {1} {R_ {p}} & = \ frac {1} {R_ {1}} + \ frac {1} {R_ {2}} \\ & = \ frac {1} {\ text {100}} + \ frac {1} {\ text {10}} \\ & = \ frac {1 + 10} {\ text {100}} \\ & = \ frac {11} {\ text {100}} \\ R_ {p} & = \ text {9,09} \ text {kΩ} \ end {выровнять *}

Два резистора подключены параллельно.Эквивалентное сопротивление равно \ (\ text {3,75} \) \ (\ text {Ω} \). Если сопротивление одного резистора равно \ (\ text {10} \) \ (\ text {Ω} \), каково сопротивление второго резистора?

Поскольку резисторы включены параллельно, мы можем использовать:

\ [\ frac {1} {R_ {p}} = \ frac {1} {R_ {1}} + \ frac {1} {R_ {2}} \]

Эквивалентное сопротивление:

\ begin {align *} \ frac {1} {R_ {p}} & = \ frac {1} {R_ {1}} + \ frac {1} {R_ {2}} \\ \ frac {1} {R_ {2}} & = \ frac {1} {R_ {p}} — \ frac {1} {R_ {1}} \\ & = \ frac {1} {\ text {3,75}} — \ frac {1} {\ text {10}} \\ & = \ frac {\ text {10} — \ text {3,75}} {\ text {37,5}} \\ & = \ frac {\ text {6,25}} {\ text {37,5}} \\ R_ {2} & = \ текст {6} \ текст {Ω} \ end {выровнять *}

Рассчитайте эквивалентное сопротивление в каждой из следующих цепей:

a) Резисторы включены параллельно, поэтому мы используем:

\ [\ frac {1} {R_ {p}} = \ frac {1} {R_ {1}} + \ frac {1} {R_ {2}} \]

Эквивалентное сопротивление:

\ begin {align *} \ frac {1} {R_ {p}} & = \ frac {1} {R_ {1}} + \ frac {1} {R_ {2}} \\ & = \ frac {1} {\ text {3}} + \ frac {1} {\ text {2}} \\ & = \ frac {\ text {2} + \ text {3}} {\ text {6}} \\ & = \ frac {\ text {5}} {\ text {6}} \\ R & = \ текст {1,2} \ текст {Ω} \ end {выровнять *}

b) Резисторы включены параллельно, поэтому мы используем:

\ [\ frac {1} {R_ {p}} = \ frac {1} {R_ {1}} + \ frac {1} {R_ {2}} + \ frac {1} {R_ {3}} + \ frac {1} {R_ {4}} \]

Эквивалентное сопротивление:

\ begin {align *} \ frac {1} {R_ {p}} & = \ frac {1} {R_ {1}} + \ frac {1} {R_ {2}} + \ frac {1} {R_ {3}} + \ гидроразрыв {1} {R_ {4}} \\ & = \ frac {1} {\ text {2}} + \ frac {1} {\ text {3}} + \ frac {1} {\ text {4}} + \ frac {1} {\ text { 1}} \\ & = \ frac {\ text {6} + \ text {4} + \ text {3} + \ text {12}} {\ text {12}} \\ & = \ frac {\ text {25}} {\ text {12}} \\ R & = \ text {0,48} \ text {Ω} \ end {выровнять *}

c) Резисторы включены последовательно, поэтому мы используем:

\ [R_ {s} = R_ {1} + R_ {2} \]

Эквивалентное сопротивление:

\ begin {align *} R_ {s} & = R_ {1} + R_ {2} \\ & = \ text {2} \ text {Ω} + \ text {3} \ text {Ω} \\ & = \ текст {5} \ текст {Ω} \ end {выровнять *}

d) Резисторы включены последовательно, поэтому мы используем:

\ [R_ {s} = R_ {1} + R_ {2} + R_ {3} + R_ {4} \]

Эквивалентное сопротивление:

\ begin {align *} R_ {s} & = R_ {1} + R_ {2} + R_ {3} + R_ {4} \\ & = \ text {2} \ text {Ω} + \ text {3} \ text {Ω} + \ text {4} \ text {Ω} + \ text {1} \ text {Ω} \\ & = \ текст {10} \ текст {Ω} \ end {выровнять *}

Применение закона Ома в последовательных и параллельных цепях (ESBQB)

Используя определения эквивалентного сопротивления для резисторов, включенных последовательно или параллельно, мы можем проанализировать некоторые схемы с этими установками.

Последовательные цепи

Рассмотрим схему, состоящую из трех резисторов и одного одиночная ячейка соединена последовательно.

Первый принцип, который нужно понять в отношении последовательных цепей, заключается в том, что величина тока одинакова через любой компонент в цепи. Это потому, что существует только один путь для движения электронов в последовательной цепи. По способу подключения батареи мы можем сказать, в каком направлении будет течь ток. Мы знаем, что ток по условию течет от положительного к отрицательному.Обычный ток в этой цепи будет течь по часовой стрелке от точки A к B, от C к D и обратно к A.

Мы знаем, что в последовательной цепи ток должен быть одинаковым во всех компонентах. Итак, мы можем написать:

\ [I = I_ {1} = I_ {2} = I_ {3}. \]

Мы также знаем, что полное напряжение цепи должно быть равно сумме напряжений на всех трех резисторах. Итак, мы можем написать:

\ [V = V_ {1} + V_ {2} + V_ {3} \]

Используя эту информацию и то, что мы знаем о вычислении эквивалентного сопротивления последовательно соединенных резисторов, мы можем решить некоторые проблемы схемы.

Рабочий пример 2: Закон Ома, последовательная цепь

Вычислите ток (I) в этой цепи, если оба резистора имеют омическую природу.

Определите, что требуется

Нам необходимо рассчитать ток, протекающий в цепи.

Определите, как подойти к проблеме

Поскольку резисторы имеют омическую природу, мы можем использовать закон Ома. Однако в цепи два резистора, и нам нужно найти полное сопротивление.

Найти полное сопротивление в цепи

Поскольку резисторы включены последовательно, общее (эквивалентное) сопротивление R составляет:

\ [R = R_ {1} + R_ {2} \]

Следовательно,

\ begin {align *} R & = \ текст {2} + \ текст {4} \\ & = \ текст {6} \ текст {Ω} \ end {align *}

Применить закон Ома

\ begin {align *} R & = \ frac {V} {I} \\ R \ times \ frac {I} {R} & = \ frac {V} {I} \ times \ frac {I} {R} \\ I & = \ frac {V} {R} \\ & = \ frac {12} {6} \\ & = \ текст {2} \ текст {А} \ end {align *}

Напишите окончательный ответ

В цепи протекает ток \ (\ text {2} \) \ (\ text {A} \).

Рабочий пример 3: Закон Ома, последовательная цепь

Два омических резистора (\ (R_ {1} \) и \ (R_ {2} \)) соединены последовательно с ячейкой. Найдите сопротивление \ (R_ {2} \), учитывая, что ток, протекающий через \ (R_ {1} \) и \ (R_ {2} \), равен \ (\ text {0,25} \) \ ( \ text {A} \) и что напряжение на ячейке равно \ (\ text {1,5} \) \ (\ text {V} \). \ (R_ {1} \) = \ (\ text {1} \) \ (\ text {Ω} \).

Нарисуйте схему и введите все известные значения.

Определите, как подойти к проблеме.

Мы можем использовать закон Ома, чтобы найти полное сопротивление R в цепи, а затем вычислить неизвестное сопротивление, используя:

\ [R = R_ {1} + R_ {2} \]

, потому что он находится в последовательной цепи.

Найдите полное сопротивление

\ begin {align *} R & = \ frac {V} {I} \\ & = \ frac {\ text {1,5}} {\ text {0,25}} \\ & = \ текст {6} \ текст {Ω} \ end {align *}

Найдите неизвестное сопротивление

Мы знаем, что:

\ [R = \ text {6} \ text {Ω} \]

и что

\ [R_ {1} = \ text {1} \ text {Ω} \]

С

\ [R = R_ {1} + R_ {2} \] \ [R_ {2} = R — R_ {1} \]

Следовательно,

\ [R_ {1} = \ text {5} \ text {Ω} \]

Рабочий пример 4: Закон Ома, последовательная цепь

Для следующей схемы рассчитайте:

  1. падение напряжения \ (V_1 \), \ (V_2 \) и \ (V_3 \) на резисторах \ (R_1 \), \ (R_2 \) и \ (R_3 \)

  2. сопротивление \ (R_3 \).

Определите, как подойти к проблеме

Нам даны напряжение на ячейке и ток в цепи, а также сопротивления двух из трех резисторов. Мы можем использовать закон Ома для расчета падения напряжения на известных резисторах. Поскольку резисторы включены в последовательную цепь, напряжение равно \ (V = V_1 + V_2 + V_3 \), и мы можем вычислить \ (V_3 \). Теперь мы можем использовать эту информацию, чтобы найти напряжение на неизвестном резисторе \ (R_3 \).

Рассчитать падение напряжения на \ (R_1 \)

Используя закон Ома: \ begin {align *} R_1 & = \ frac {V_1} {I} \\ I \ cdot R_1 & = I \ cdot \ frac {V_1} {I} \\ V_1 & = {I} \ cdot {R_1} \\ & = 2 \ cdot 1 \\ V_1 & = \ текст {2} \ текст {V} \ end {align *}

Рассчитать падение напряжения на \ (R_2 \)

Снова используя закон Ома: \ begin {align *} R_2 & = \ frac {V_2} {I} \\ I \ cdot R_2 & = I \ cdot \ frac {V_2} {I} \\ V_2 & = {I} \ cdot {R_2} \\ & = 2 \ cdot 3 \\ V_2 & = \ текст {6} \ текст {V} \ end {align *}

Рассчитать падение напряжения на \ (R_3 \)

Поскольку падение напряжения на всех резисторах, вместе взятых, должно быть таким же, как падение напряжения на ячейке в последовательной цепи, мы можем найти \ (V_3 \), используя: \ begin {align *} V & = V_1 + V_2 + V_3 \\ V_3 & = V — V_1 — V_2 \\ & = 18-2-6 \\ V_3 & = \ текст {10} \ текст {V} \ end {align *}

Найдите сопротивление \ (R_3 \)

Нам известно напряжение на \ (R_3 \) и ток через него, поэтому мы можем использовать закон Ома для вычисления значения сопротивления: \ begin {align *} R_3 & = \ frac {V_3} {I} \\ & = \ frac {10} {2} \\ R_3 & = \ text {5} \ Omega \ end {align *}

Напишите окончательный ответ

\ (V_1 = \ text {2} \ text {V} \)

\ (V_2 = \ text {6} \ text {V} \)

\ (V_3 = \ text {10} \ text {V} \)

\ (R_1 = \ text {5} \ Omega \)

Параллельные цепи

Рассмотрим схему, состоящую из одной ячейки и трех резисторов, соединенных параллельно.

Первый принцип, который нужно понять в отношении параллельных цепей, заключается в том, что напряжение одинаково на всех компонентах в цепи. Это связано с тем, что в параллельной цепи есть только два набора электрически общих точек, и напряжение, измеренное между наборами общих точек, всегда должно быть одинаковым в любой момент времени. Итак, для показанной схемы верно следующее:

\ [V = V_ {1} = V_ {2} = V_ {3}. \]

Второй принцип параллельной схемы состоит в том, что все токи, проходящие через каждый резистор, должны в сумме равняться общему току в цепи:

\ [I = I_ {1} + I_ {2} + I_ {3}.\]

Используя эти принципы и наши знания о том, как рассчитать эквивалентное сопротивление параллельных резисторов, мы можем теперь подойти к некоторым проблемам схемы, связанным с параллельными резисторами.

Рабочий пример 5: Закон Ома, параллельная цепь

Вычислите ток (I) в этой цепи, если оба резистора имеют омическую природу.

Определите, что требуется

Нам необходимо рассчитать ток, протекающий в цепи.

Определите, как подойти к проблеме

Поскольку резисторы имеют омическую природу, мы можем использовать закон Ома.Однако в цепи два резистора, и нам нужно найти полное сопротивление.

Найдите эквивалентное сопротивление в цепи

.

Поскольку резисторы включены параллельно, общее (эквивалентное) сопротивление R составляет:

\ [\ frac {1} {R} = \ frac {1} {R_ {1}} + \ frac {1} {R_ {2}}. \] \ begin {align *} \ frac {1} {R} & = \ frac {1} {R_1} + \ frac {1} {R_2} \\ & = \ frac {1} {2} + \ frac {1} {4} \\ & = \ frac {2 + 1} {4} \\ & = \ frac {3} {4} \\ \ text {Следовательно,} R & = \ text {1,33} \ Omega \ end {align *}

Применить закон Ома

\ begin {align *} R & = \ frac {V} {I} \\ R \ cdot \ frac {I} {R} & = \ frac {V} {I} \ cdot \ frac {I} {R} \\ I & = \ frac {V} {R} \\ I & = V \ cdot \ frac {1} {R} \\ & = (12) \ left (\ frac {3} {4} \ right) \\ & = \ текст {9} \ текст {А} \ end {align *}

Напишите окончательный ответ

В цепи протекает ток \ (\ text {9} \) \ (\ text {A} \).

Рабочий пример 6: Закон Ома, параллельная цепь

Два омических резистора (\ (R_1 \) и \ (R_2 \)) подключены параллельно ячейке. Найдите сопротивление \ (R_2 \), учитывая, что ток, протекающий через ячейку, равен \ (\ text {4,8} \) \ (\ text {A} \) и что напряжение на ячейке равно \ (\ текст {9} \) \ (\ text {V} \).

Определите, что требуется

Нам нужно рассчитать сопротивление \ (R_2 \).

Определите, как подойти к проблеме

Поскольку резисторы омические и нам даны напряжение на ячейке и ток в ячейке, мы можем использовать закон Ома, чтобы найти эквивалентное сопротивление в цепи.\ begin {align *} R & = \ frac {V} {I} \\ & = \ frac {9} {\ text {4,8}} \\ & = \ text {1,875} \ \ Omega \ end {align *}

Рассчитайте значение для \ (R_2 \)

Поскольку мы знаем эквивалентное сопротивление и сопротивление \ (R_1 \), мы можем использовать формулу для параллельных резисторов, чтобы найти сопротивление \ (R_2 \). \ begin {align *} \ frac {1} {R} & = \ frac {1} {R_1} + \ frac {1} {R_2} \ end {выровнять *} Переставляем решение для \ (R_2 \): \ begin {align *} \ frac {1} {R_2} & = \ frac {1} {R} — \ frac {1} {R_1} \\ & = \ frac {1} {\ text {1,875}} — \ frac {1} {3} \\ & = \ текст {0,2} \\ R_2 & = \ frac {1} {\ text {0,2}} \\ & = \ текст {5} \ \ Omega \ end {align *}

Напишите окончательный ответ

Сопротивление \ (R_2 \) равно \ (\ text {5} \) \ (\ Omega \)

Рабочий пример 7: Закон Ома, параллельная цепь

Ячейка на 18 В подключена к двум параллельным резисторам \ (\ text {4} \) \ (\ Omega \) и \ (\ text {12} \) \ (\ Omega \) соответственно.Рассчитайте ток через ячейку и через каждый из резисторов.

Сначала нарисуйте схему перед выполнением любых расчетов

Определите, как подойти к проблеме

Нам нужно определить ток через ячейку и каждый из параллельных резисторов. Нам дана разность потенциалов на ячейке и сопротивления резисторов, поэтому мы можем использовать закон Ома для расчета тока.

Рассчитать ток через ячейку

Чтобы рассчитать ток через элемент, нам сначала нужно определить эквивалентное сопротивление остальной части цепи.Резисторы включены параллельно и поэтому: \ begin {align *} \ frac {1} {R} & = \ frac {1} {R_1} + \ frac {1} {R_2} \\ & = \ frac {1} {4} + \ frac {1} {12} \\ & = \ frac {3 + 1} {12} \\ & = \ frac {4} {12} \\ R & = \ frac {12} {4} = \ text {3} \ \ Omega \ end {выровнять *} Теперь, используя закон Ома, чтобы найти ток через ячейку: \ begin {align *} R & = \ frac {V} {I} \\ I & = \ frac {V} {R} \\ & = \ frac {18} {3} \\ I & = \ text {6} \ text {A} \ end {align *}

Теперь определим ток через один из параллельных резисторов

Мы знаем, что для чисто параллельной схемы напряжение на ячейке такое же, как напряжение на каждом из параллельных резисторов.Для этой схемы: \ begin {align *} V & = V_1 = V_2 = \ text {18} \ text {V} \ end {выровнять *} Начнем с расчета тока через \ (R_1 \) по закону Ома: \ begin {align *} R_1 & = \ frac {V_1} {I_1} \\ I_1 & = \ frac {V_1} {R_1} \\ & = \ frac {18} {4} \\ I_1 & = \ text {4,5} \ text {A} \ end {align *}

Рассчитайте ток через другой параллельный резистор

Мы можем снова использовать закон Ома, чтобы найти ток в \ (R_2 \): \ begin {align *} R_2 & = \ frac {V_2} {I_2} \\ I_2 & = \ frac {V_2} {R_2} \\ & = \ frac {18} {12} \\ I_2 & = \ text {1,5} \ text {A} \ end {выровнять *} Альтернативный метод вычисления \ (I_2 \) заключался бы в использовании того факта, что токи через каждый из параллельных резисторов должны составлять суммарный ток через ячейку: \ begin {align *} I & = I_1 + I_2 \\ I_2 & = I — I_1 \\ & = 6 — 4.5 \\ I_2 & = \ text {1,5} \ text {A} \ end {align *}

Напишите окончательный ответ

Ток через ячейку равен \ (\ text {6} \) \ (\ text {A} \).

Ток через резистор \ (\ text {4} \) \ (\ Omega \) равен \ (\ text {4,5} \) \ (\ text {A} \).

Ток через резистор \ (\ text {12} \) \ (\ Omega \) равен \ (\ text {1,5} \) \ (\ text {A} \).

Закон Ома в последовательной и параллельной цепях

Упражнение 11.4

Рассчитать номинал неизвестного резистора в цепи:

Сначала мы используем закон Ома для расчета полного последовательного сопротивления:

\ begin {align *} R & = \ frac {V} {I} \\ & = \ frac {9} {1} \\ & = \ текст {9} \ текст {Ω} \ end {выровнять *}

Теперь мы можем найти неизвестное сопротивление:

\ begin {align *} R_ {s} & = R_ {1} + R_ {2} + R_ {3} + R_ {4} \\ R_ {4} & = R_ {s} — R_ {1} — R_ {2} — R_ {3} \\ & = 9 — 3 — 3 — 1 \\ & = \ текст {2} \ текст {Ω} \ end {выровнять *}

Рассчитайте значение тока в следующей цепи:

Сначала находим общее сопротивление:

\ begin {align *} R_ {s} & = R_ {1} + R_ {2} + R_ {3} \\ & = \ text {1} + \ text {2,5} + \ text {1,5} \\ & = \ текст {5} \ текст {Ω} \ end {выровнять *}

Теперь мы можем рассчитать текущую:

\ begin {align *} I & = \ frac {V} {R} \\ & = \ frac {9} {5} \\ & = \ текст {1,8} \ текст {А} \ end {выровнять *}

Три резистора с сопротивлением \ (\ text {1} \) \ (\ text {Ω} \), \ (\ text {5} \) \ (\ text {Ω} \) и \ (\ text {10} \) \ (\ text {Ω} \) соответственно соединены последовательно с батареей \ (\ text {12} \) \ (\ text {V} \).Рассчитайте значение тока в цепи.

Рисуем принципиальную схему:

Теперь мы находим общее сопротивление:

\ begin {align *} R_ {s} & = R_ {1} + R_ {2} + R_ {3} \\ & = \ текст {1} + \ текст {5} + \ текст {10} \\ & = \ текст {16} \ текст {Ω} \ end {выровнять *}

Теперь мы можем рассчитать текущую:

\ begin {align *} I & = \ frac {V} {R} \\ & = \ frac {12} {16} \\ & = \ текст {0,75} \ текст {A} \ end {выровнять *}

Рассчитайте ток через ячейку, если оба резистора омические по своей природе.

Сначала находим общее сопротивление:

\ begin {align *} \ frac {1} {R_ {p}} & = \ frac {1} {R_ {1}} + \ frac {1} {R_ {2}} \\ & = \ frac {1} {\ text {1}} + \ frac {1} {\ text {3}} \\ & = \ frac {3 + 1} {\ text {3}} \\ & = \ frac {4} {\ text {3}} \\ & = \ текст {0,75} \ текст {Ω} \ end {выровнять *}

Теперь мы можем рассчитать текущую:

\ begin {align *} I & = \ frac {V} {R} \\ & = \ frac {9} {\ text {0,75}} \\ & = \ текст {12} \ текст {А} \ end {выровнять *}

Рассчитайте номинал неизвестного резистора \ (R_ {4} \) в цепи:

Сначала находим общее сопротивление:

\ begin {align *} R & = \ frac {V} {I} \\ & = \ frac {24} {\ text {2}} \\ & = \ текст {12} \ текст {Ω} \ end {выровнять *}

Теперь мы можем рассчитать неизвестное сопротивление:

\ begin {align *} \ frac {1} {R_ {p}} & = \ frac {1} {R_ {1}} + \ frac {1} {R_ {2}} + \ frac {1} {R_ {3}} + \ гидроразрыв {1} {R_ {4}} \\ \ frac {1} {R_ {4}} & = \ frac {1} {R_ {p}} — \ frac {1} {R_ {1}} — \ frac {1} {R_ {2}} — \ гидроразрыв {1} {R_ {3}} \\ & = \ frac {1} {\ text {12}} — \ frac {1} {\ text {120}} — \ frac {1} {\ text {40}} — \ frac {1} {\ text { 60}} \\ & = \ frac {10 — 1 — 3 — 2} {\ text {120}} \\ & = \ frac {4} {\ text {120}} \\ & = \ текст {30} \ текст {Ω} \ end {выровнять *}

значение тока через аккумулятор

Рисуем принципиальную схему:

Чтобы вычислить значение тока через батарею, нам сначала нужно вычислить эквивалентное сопротивление:

\ begin {align *} \ frac {1} {R_ {p}} & = \ frac {1} {R_ {1}} + \ frac {1} {R_ {2}} + \ frac {1} {R_ {3}} \\ & = \ frac {1} {\ text {1}} + \ frac {1} {\ text {5}} + \ frac {1} {\ text {10}} \\ & = \ frac {10 + 2 + 1} {\ text {10}} \\ & = \ frac {13} {\ text {10}} \\ & = \ текст {0,77} \ текст {Ω} \ end {выровнять *}

Теперь можем посчитать ток через батарею:

\ begin {align *} I & = \ frac {V} {R} \\ & = \ frac {20} {\ text {0,77}} \\ & = \ текст {26} \ текст {А} \ end {выровнять *}

значение тока в каждом из трех резисторов.

Для параллельной схемы напряжение на ячейке такое же, как напряжение на каждом из резисторов. Для этой схемы:

\ [V = V_ {1} = V_ {2} = V_ {3} = \ text {20} \ text {V} \]

Теперь мы можем рассчитать ток через каждый резистор. Начнем с \ (R_ {1} \):

\ begin {align *} I & = \ frac {V} {R} \\ & = \ frac {20} {\ text {1}} \\ & = \ текст {20} \ текст {А} \ end {выровнять *}

Затем мы вычисляем ток через \ (R_ {2} \):

\ begin {align *} I & = \ frac {V} {R} \\ & = \ frac {20} {\ text {5}} \\ & = \ текст {4} \ текст {А} \ end {выровнять *}

И наконец вычисляем ток через \ (R_ {3} \):

\ begin {align *} I & = \ frac {V} {R} \\ & = \ frac {20} {\ text {10}} \\ & = \ текст {2} \ текст {А} \ end {выровнять *}

Вы можете проверить, что они в сумме составляют общий ток.

Последовательные и параллельные сети резисторов (ESBQC)

Теперь, когда вы знаете, как работать с простыми последовательными и параллельными цепями, вы готовы заняться цепями, которые объединяют эти две схемы, например, следующую схему:

Рисунок 11.1: Пример последовательно-параллельной сети. Пунктирными прямоугольниками обозначены параллельные участки цепи.

Проработать такие схемы относительно легко, потому что вы используете все, что вы уже узнали о последовательных и параллельных схемах.Единственная разница в том, что вы делаете это поэтапно. На рисунке 11.1 схема состоит из двух параллельных частей, которые затем включены последовательно с ячейкой. Чтобы вычислить эквивалентное сопротивление для схемы, вы начинаете с вычисления общего сопротивления каждой из параллельных частей, а затем последовательно складываете эти сопротивления. Если бы все резисторы на рисунке 11.1 имели сопротивление \ (\ text {10} \) \ (\ text {Ω} \), мы можем вычислить эквивалентное сопротивление всей цепи.

Начнем с расчета общего сопротивления параллельной цепи 1 .{-1} \\ & = \ текст {5} \, \ Omega \ end {align *}

Теперь вы можете рассматривать схему как простую последовательную схему следующим образом:

Следовательно, эквивалентное сопротивление: \ begin {align *} R & = R_ {p1} + R_ {p2} \\ & = 5 + 5 \\ & = 10 \, \ Omega \ end {align *}

Эквивалентное сопротивление цепи на рисунке 11.1 равно \ (\ text {10} \) \ (\ text {Ω} \).

Последовательные и параллельные сети

Упражнение 11.5

Начнем с определения эквивалентного сопротивления параллельной комбинации:

\ begin {align *} \ frac {1} {R_ {p}} & = \ frac {1} {R_ {1}} + \ frac {1} {R_ {2}} \\ & = \ frac {1} {4} + \ frac {1} {2} \\ & = \ frac {3} {4} \\ R_ {p} & = \ text {1,33} \ text {Ω} \ end {выровнять *}

Теперь у нас есть цепь с двумя последовательно включенными резисторами, поэтому мы можем вычислить эквивалентное сопротивление:

\ begin {align *} R_ {s} & = R_ {3} + R_ {p} \\ & = \ текст {2} + \ текст {1,33} \\ & = \ текст {3,33} \ текст {Ω} \ end {выровнять *}

Начнем с определения эквивалентного сопротивления параллельной комбинации:

\ begin {align *} \ frac {1} {R_ {p}} & = \ frac {1} {R_ {1}} + \ frac {1} {R_ {2}} \\ & = \ frac {1} {1} + \ frac {1} {2} \\ & = \ frac {3} {2} \\ R_ {p} & = \ text {0,67} \ text {Ω} \ end {выровнять *}

Теперь у нас есть цепь с тремя последовательно включенными резисторами, поэтому мы можем вычислить эквивалентное сопротивление:

\ begin {align *} R_ {s} & = R_ {3} + R_ {4} + R_ {p} \\ & = \ текст {4} + \ текст {6} + \ текст {0,67} \\ & = \ текст {10,67} \ текст {Ω} \ end {выровнять *}

Начнем с определения эквивалентного сопротивления параллельной комбинации:

\ begin {align *} \ frac {1} {R_ {p}} & = \ frac {1} {R_ {1}} + \ frac {1} {R_ {2}} + \ frac {1} {R_ {3}} \\ & = \ frac {1} {3} + \ frac {1} {5} + \ frac {1} {1} \\ & = \ frac {23} {15} \\ R_ {p} & = \ text {0,652} \ text {Ω} \ end {выровнять *}

Теперь у нас есть цепь с двумя последовательно включенными резисторами, поэтому мы можем вычислить эквивалентное сопротивление:

\ begin {align *} R_ {s} & = R_ {4} + R_ {p} \\ & = \ текст {2} + \ текст {0,652} \\ & = \ текст {2,652} \ текст {Ω} \ end {выровнять *}

ток \ (I \) через ячейку.

Чтобы найти ток \ (I \), нам сначала нужно найти эквивалентное сопротивление. Начнем с расчета эквивалентного сопротивления параллельной комбинации:

\ begin {align *} \ frac {1} {R_ {p}} & = \ frac {1} {R_ {1}} + \ frac {1} {R_ {2}} + \ frac {1} {R_ {3}} \\ & = \ frac {1} {3} + \ frac {1} {5} + \ frac {1} {1} \\ & = \ frac {23} {15} \\ R_ {p} & = \ text {0,652} \ text {Ω} \ end {выровнять *}

Теперь у нас есть цепь с двумя последовательно включенными резисторами, поэтому мы можем вычислить эквивалентное сопротивление:

\ begin {align *} R_ {s} & = R_ {4} + R_ {p} \\ & = \ текст {2} + \ текст {0,652} \\ & = \ текст {2,652} \ текст {Ω} \ end {выровнять *}

Итак, ток через ячейку:

\ begin {align *} I & = \ frac {V} {R} \\ & = \ frac {\ text {12}} {\ text {2,652}} \\ & = \ текст {4,52} \ текст {А} \ end {выровнять *}

ток через резистор \ (\ text {5} \) \ (\ text {Ω} \).

Ток через параллельную комбинацию резисторов равен \ (\ text {4,52} \) \ (\ text {A} \). (Ток одинаков при последовательном соединении резисторов, и мы можем рассматривать весь параллельный набор резисторов как один последовательный резистор.)

Используя это, мы можем найти напряжение через параллельную комбинацию резисторов (не забудьте использовать эквивалентное параллельное сопротивление, а не эквивалентное сопротивление цепи):

\ begin {align *} V & = I \ cdot R \\ & = (\ текст {4,52}) (\ текст {0,652}) \\ & = \ текст {2,95} \ текст {V} \ end {выровнять *}

Поскольку напряжение на каждом резисторе в параллельной комбинации одинаково, это также напряжение на резисторе \ (\ text {5} \) \ (\ text {Ω} \).

Итак, теперь мы можем рассчитать ток через резистор:

\ begin {align *} I & = \ frac {V} {R} \\ & = \ frac {\ text {2,95}} {\ text {5}} \\ & = \ текст {0,59} \ текст {А} \ end {выровнять *}

Если ток, протекающий через ячейку, равен \ (\ text {2} \) \ (\ text {A} \), и все резисторы омические, рассчитайте напряжение на ячейке и на каждом из резисторов, \ (R_1 \ ), \ (R_2 \) и \ (R_3 \) соответственно.

Чтобы найти напряжение, нам сначала нужно найти эквивалентное сопротивление.Начнем с расчета эквивалентного сопротивления параллельной комбинации:

\ begin {align *} \ frac {1} {R_ {p}} & = \ frac {1} {R_ {2}} + \ frac {1} {R_ {3}} \\ & = \ frac {1} {2} + \ frac {1} {4} \\ & = \ frac {3} {4} \\ R_ {p} & = \ text {1,33} \ text {Ω} \ end {выровнять *}

Теперь у нас есть цепь с двумя последовательно включенными резисторами, поэтому мы можем вычислить эквивалентное сопротивление:

\ begin {align *} R_ {s} & = R_ {1} + R_ {p} \\ & = \ text {4,66} + \ text {1,33} \\ & = \ текст {5,99} \ текст {Ω} \ end {выровнять *}

Итак, напряжение на ячейке:

\ begin {align *} V & = I \ cdot R \\ & = (\ текст {2}) (\ текст {5,99}) \\ & = \ текст {12} \ текст {V} \ end {выровнять *}

Ток через параллельную комбинацию резисторов равен \ (\ text {2} \) \ (\ text {A} \).(Ток одинаков при последовательном соединении резисторов, и мы можем рассматривать весь параллельный набор резисторов как один последовательный резистор.)

Используя это, мы можем найти напряжение на каждом из резисторов. Начнем с нахождения напряжения на \ (R_ {1} \):

\ begin {align *} V & = I \ cdot R \\ & = (\ текст {2}) (\ текст {4,66}) \\ & = \ текст {9,32} \ текст {V} \ end {выровнять *}

Теперь находим напряжение на параллельной комбинации:

\ begin {align *} V & = I \ cdot R \\ & = (\ текст {2}) (\ текст {1,33}) \\ & = \ текст {2,66} \ текст {V} \ end {выровнять *}

Поскольку напряжение на каждом резисторе в параллельной комбинации одинаково, это также напряжение на резисторах \ (R_ {2} \) и \ (R_ {3} \).

ток через ячейку

Чтобы найти ток, нам сначала нужно найти эквивалентное сопротивление. Начнем с расчета эквивалентного сопротивления параллельной комбинации:

\ begin {align *} \ frac {1} {R_ {p}} & = \ frac {1} {R_ {2}} + \ frac {1} {R_ {3}} \\ & = \ frac {1} {1} + \ frac {1} {1} \\ & = 2 \\ R_ {p} & = \ text {0,5} \ text {Ω} \ end {выровнять *}

Теперь у нас есть цепь с двумя последовательно включенными резисторами, поэтому мы можем вычислить эквивалентное сопротивление:

\ begin {align *} R_ {s} & = R_ {1} + R_ {4} + R_ {p} \\ & = \ text {2} + \ text {1,5} + \ text {0,5} \\ & = \ текст {4} \ текст {Ω} \ end {выровнять *}

Итак, ток через ячейку:

\ begin {align *} I & = \ frac {V} {R} \\ & = \ frac {\ text {10}} {\ text {4}} \\ & = \ текст {2,5} \ текст {А} \ end {выровнять *}

падение напряжения на \ (R_4 \)

Ток через все резисторы равен \ (\ text {2,5} \) \ (\ text {A} \).(Ток одинаков при последовательном соединении резисторов, и мы можем рассматривать весь параллельный набор резисторов как один последовательный резистор.)

Используя это, мы можем найти напряжение через \ (R_ {4} \):

\ begin {align *} V & = I \ cdot R \\ & = (\ text {2,5}) (\ text {1,5}) \\ & = \ текст {3,75} \ текст {V} \ end {выровнять *}

ток через \ (R_2 \)

Ток через все резисторы равен \ (\ text {2,5} \) \ (\ text {A} \).(Ток одинаков при последовательном соединении резисторов, и мы можем рассматривать весь параллельный набор резисторов как один последовательный резистор.)

Используя это, мы можем найти ток через \ (R_ {2} \).

Сначала нам нужно найти напряжение на параллельной комбинации:

\ begin {align *} V & = I \ cdot R \\ & = (\ text {2,5}) (\ text {0,5}) \\ & = \ текст {1,25} \ текст {V} \ end {выровнять *}

Теперь мы можем найти ток через \ (R_ {2} \), используя тот факт, что напряжение одинаково на всех резисторах в параллельной комбинации:

\ begin {align *} I & = \ frac {V} {R} \\ & = \ frac {\ text {1,25}} {\ text {1}} \\ & = \ текст {1,25} \ текст {А} \ end {выровнять *}

Как применять закон Ома — Jade Learning

Как применять закон Ома

Автор: Вес Губиц | 07 августа 2019 г.

Электроэнергия работает в предсказуемых пределах.Мы пришли к определению этих границ как закон Ома. Закон Ома был разработан как средство объяснения того, как электричество работает в замкнутой цепи. Формула закона Ома помогает установить взаимосвязь между различными свойствами в электрической цепи. Мы можем использовать закон Ома, чтобы объяснить, что произошло, а также что произойдет, когда на электрическую цепь накладываются определенные условия.

Основные характеристики электрической схемы: Напряжение, ток и сопротивление .Они специфичны, определены и не меняются — при условии, что все свойства остаются постоянными. Однако измените значение только одного из этих свойств, и все свойства изменят значение соответствующим образом.

Закон Ома — это самая основная из электрических формул, он был разработан путем простого наблюдения за свойствами электричества в электрической цепи. Электричество ведет себя иначе из-за ограничений, налагаемых формулой закона Ома; формула просто представляет наши наблюдения за поведением, уже происходящим в электрической цепи.

Хотя закон Ома — всего лишь вводная ступенька на лестнице электротехники, для понимания того, как закон Ома как формула применяется к простой цепи, необходимо базовое понимание электрической цепи. Простая схема состоит из источника питания, нагрузки, проводов, устройства максимального тока и устройства управления. Ток будет течь в этой простой цепи, если имеется достаточное напряжение, чтобы преодолеть любое сопротивление цепи.
Напряжение считается давлением в электрической цепи; это уместно называется электродвижущей силой.Это давление или «сила» вызывается разными электрическими полюсами, которые хотят уравновесить себя. Толчок и притяжение, наложенные на электроны в проводнике, подключенном к этим разным полюсам, заставят электроны двигаться, если для них существует полный путь. Единственное, что может остановить движение электронов, — это приложенное сопротивление сверх того напряжения, которое заставляет их двигаться, или разрыв цепи, который нарушает поток этих электронов. Требуется один вольт (В) этой электродвижущей силы, чтобы протолкнуть один ампер (А) тока через один ом (Ом) сопротивления — это закон Ома.Напряжение (E или V) равно току (I), умноженному на сопротивление (R). Или, другими словами, E (или V) = IR.

Обозначения

  • Вольт (E или V) = электродвижущая сила, опять же, это давление, которое заставляет электроны перемещаться по проводнику (и через нагрузку) в замкнутой цепи.
  • Ток (I) = интенсивность, представляет ток, протекающий в цепи. Помните, что «интенсивность» тока в цепи измеряется в амперах.
  • Сопротивление (R) = Ом, сопротивление току.Сопротивление может быть преднамеренным или случайным, но в любом случае оно является противодействием свободному току в цепи и отображается на вашем электрическом счетчике в виде Ом. Нулевое сопротивление или близкое к нему означает буквально отсутствие сопротивления току. Медь имеет очень низкое значение сопротивления на фут и является высококачественным материалом для создания эффективных проводников.

Давайте посмотрим на символы закона Ома внутри треугольника закона Ома.

Помните, что вольт (E или V) равняется току (I), умноженному на сопротивление (R)

Использование треугольника закона Ома в качестве наглядного пособия при запоминании трех уравнений закона Ома — не редкость.

Чтобы найти пропущенное значение в реальном уравнении закона Ома, просто закройте букву, представляющую пропущенное значение в треугольнике, и используйте оставшиеся два значения для вычисления этого пропущенного значения.

Например: если вы знаете, что лампа на 120 В (Е или В) измеряет при использовании ток 0,625 А (I), какое сопротивление оказывает лампа?

120 В (E), деленное на 0,625 А (I), равняется 192 Ом (R) сопротивления.

Что делать, если вам известны измеряемые амперы (I) протекающего тока и сопротивление (R) нити накала лампы? Можете ли вы затем рассчитать напряжение, подаваемое на эту лампу? Посмотрите на треугольник закона Ома ниже, чтобы определить свой ответ.

Добавить комментарий

Ваш адрес email не будет опубликован. Обязательные поля помечены *